SlideShare una empresa de Scribd logo
1 de 195
Descargar para leer sin conexión
INFECTOLOGIA
ENARM
MENINGITIS
• Cual es la presión de apertura a la toma de LCR en meningitis bacteriana?
• R = Normal <180 o elevada > 180, siendo común 200-500
• Cuales son los datos del LCR de la meningitis bacteriana aguda?
I. Presión de abertura: Normal o > 180 siendo común observar valores entre 2 –
500
II. Apariencia: Leucocitos y bacterias turban el LCR
III. Glucorraquia: Concentración < 40 (NORMAL > 45)
IV. Proteinorraquia: Valores arriba de 50 (NORMAL 40)
• Cual es el tratamiento de elección en los contactos de pacientes con enfermedad
meningocococica?
• R = RIFAMPICINA
• Que constituye el síndrome de austrian?
• R = MENINGITIS, ENDOCARDITIS y NEUMONÍA por estreptococo pneumoniae
en PACIENTE ALCOHÓLICOS
MENINGITIS
• Tratamiento de elección para meningitis por NEUMOCOCO?
• R = CEFTRIAXONA
• Tratamiento de elección para meningitis por S. AUREUS METICILINO SENSIBLE?
• R = Nafcilina, DICLOXACILINA
• Tratamiento de elección para meningitis por S. AUREUS METICILINO RESISTENTE?
• R = VANCOMICINA o TEICOPLANINA
• Tratamiento de elección para meningitis por LISTERIA?
• R = AMPICILINA
• Tratamiento de elección para meningitis por H. INFLUENZA?
• R = CEFTRIAXONA o cefotaxima
• Tratamiento de elección para meningitis por ANAEROBIOS?
• R = METRONIDAZOL
• Cual es el tratamiento de elección en la meningitis CRIPTOCOCOCICA?
• R = ANFOTERICINA B + 5-fluocitosina
MENINGITIS
• Que medida se debe tomar en los contactos de un paciente con meningitis
por Neisseria Meningitidis?
• R = RIFAMPICINA oral a contactos cercanos o CIPROFLOXACINO.
• - Two students from a university dormitory building have contracted
meningitis due to Neisseria meningitides. Which of the following students in
the dormitory are most likely to benefit from chemoprophylaxis?
• (A) everybody in the dormitory, with oral amoxicillin
• (B) close contacts only, with oral amoxicillin
• (C) everybody in the dormitory, with oral rifampin
• (D) close contacts only, with oral rifampin
• (E) everybody in the dormitory, with meningococcal vaccine
- Although only close contacts need
chemoprophylaxis, it is sometimes given more
widely than recommended because of
community concern. Meningococcal vaccine is
effective against serotype A and C, and will
prevent late secondary infection in close
contacts. Ciprofloxacin or ofloxacin are
alternatives to rifampin
MENINGITIS
• Característica de meningoencefalitis subaguda?
• Meningoencefelatis por tuberculosis
• Agente etiológicos en meningitis en recién nacidos?
• E.COLI (en el trabajo de parto) y estreptococos del grupo B (AGALACTIAE)
• Agente etiológico de meningitis en adultos?
• 1.Streptococo pneumoniae 2.Neisseria Meningitidis 3.Streptococo del grupo B
• CASO CLÍNICO: Recién nacido, con fontanela abombadas, petequias en la cara anterior
del tórax, esta en cunero y alado entro por deshidratación. Actualmente con fiebre,
rigidez de nuca, Kerney y Brusinski, punción lumbar con diplococos GRAM NEGATIVOS
en LCR
• Agente etiológico?:
• Meningitis por Neisseria Meningitidis (MENINGOCOCO)
• Tx:
• PGSC a dosis altas, al personal involucrado: cipro o rifampicina
• Medida implantada por la OMS para detección rápida de MENINGITIDIS?
• DETERMINACIÓN DE ANTÍGENOS CAPSULARES: Streptococo pneumone, Neisseria
Meningitidis, Hemophilus Influenza se detecta en 3 horas –aunque no los solicites-
MENINGITIS
• Meningitis POR LISTERIA (BACILO GRAM +) esta relacionado con:
• LECHE O DERIVADOS LACTEOS, CARNES MAL COCIDAS, INMUNODEPRIMIDOS y px
de la tercera edad…. Tx :AMPICILINA como primera opción
• Con respecto del Streptococo Pneumoniae caract de infección?:
• Se da principalmente en pacientes de los 2 a los 20 años, px ALCOHÓLICOS, con
OTITIS PREVIA.
• Meningitis por Bacilos GRAM NEGATIVOS caract de infección?
• Se presenta en diabéticos, cirróticos, alcohólicos px con IVU, px con cáncer y/o
inmunodeprimidos. El bacilo GRAM POSITIVO y entérico mas importante la
listeria monocitogena, otro bacilo que no es entérico es la hemophilus Influenza
(disminución en su incidencia actualmente)
• Agente etiológico de la Meningitis en un px que previamente se le realizo
procedimiento neuroquirurgico?
• STAPHILOCOCOS AUREUS
• Citocinas proinflamatorias que producen daño a nivel SNC, en la barrera
hematoencefalica, plexos coroides?
• IL 1 Y FNT que aumentan en las primeras 2 horas
MENINGITIS
• Cantidad normal de proteínas en liquido cefalorraquídeos?
• Normal: 40 nos pueden poner 45
• La meningitis es el prototipo de los tres tipos de edema cerebral, cuales son?:
1) EDEMA VASOGENICO (FNT: aumenta la permeabilidad dela Barrera
hematopencefalica),
2) EDEMA INTERSTICIAL (a nivel del espacio subaracnoideo hay Exudado, leucocitos
producen obstrucción del flujo LCR produciendo este tipo de edema),
3) EDEMA CITOTOXICO (degranulación de los neutrófilos libera metabolitos tóxicos)
• Como obtenemos la presión perfusión intracerebral?
• Presión arterial media menos la presión intracerebral… una disminución de una
presión arterial media puede provocar isquemia e infarto cerebral (p/ej sepsis)
• Cuadro clínico de px con meningitis bacteriana?
1) FIEBRE , CEFALEA Y RIGIDEZ DE NUCA (hasta en el 90% de los casos) leucocitosis,
alteración del edo. De la conciencia.
2) Nausea , vomito, si tiene RASH EN LA CARA ANTERIOR DEL TORAX (casi
seguro MENINGOCOCCEMIA). Datos de hipertensión intracraneal (papiledema,
pupilas hiporreactivas, postura de descerebración, tiene reflejo de cushing)
MENINGITIS
• Laboratorio de LCR?
• PUNCIÓN LUMBAR: 1.-PRESION NORMAL DEL LCR 180mm de Agua, 2.-POLIMORFONUCLEARES:
menos de 5 polimorfos 3.-GLUCOSA: es el 50% dela glucosa central del px entre 45 y 55 4.-
PROTEINAS: 40mg/dl 5.-AGUA DE ROCA 6.-CLORUROS: 90 a 110 en LCR
• En quienes se realiza el cultivo en TINTA CHINA?
• CRIPTOCOCO NEOFORMANS EN PX CON VIH
• Caso clinico
• Px con rigidez de nuca, brusinski +, irritado Cefalea, fiebre, sin traumatismo craneoencefálico,
ANTECEDENTE DE NADAR EN UN MANANTIAL LANZÁNDOSE CLAVADOS, a la punción lumbar: aspecto
turbio, presión:200mmhg glucosa:20mg/dl proteínas:85 mg/dl Tinción de GRAM fue NEGATIVA, el
cultivo se realizo, que tipo de meningitis tiene este px:
• a.-M aseptica b.-M purulenta c.-M Tuberculosa (Agente etiológico: NAEGLERIA o gardenelas son
amibas de vida silvestre) muy agresivas, con pronostico mortal,
• Tx?:
• ANFOTERICINA B
• La meningitis Viral caract del LCR:
• CELULARIDAD: NORMAL PROTEÍNAS: ligeramente AUMENTADAS ASPECTO: AGUA DE ROCA
GLUCOSA: NORMAL PRESIÓN: nl o ligeramente NORMAL CLORUROS: NORMALES
MENINGITIS
• CASO CLÍNICO: Px con fiebre intensa (40), ingresa al servicio de urg, con trastorno
del edo. De conciencia, con 2 crisis convulsivas.. de ocupación CUIDADOR DE
CABALLOS (antecedente de haber enterrado a su caballo y después fue atacado
por aves de rapiña)
• LCR: Glucosa: 100 liquido transparente, presión: 185mm de agua, celularidad:
linfocitos cloruros normales
• Cual es la impresión dx:
• MENINGOENCEFALITIS VIRAL EQUINA VENEZOLANA
• CASO CLÍNICO: Niña Indígena procedente de AREA RURAL, se presenta al servicio
de urg. Con 3 semanas de evolución con fiebre persistente, vomito, trastorno del
estado de conciencia, ala exploración física presenta papiledema ligero, con
afectación del 3er, 4to, 6to par craneal, con rigidez de nuca. LCR: ASPECTO
XANTOCROMICO, Presión: 220mm de agua Glucosa: 30 Proteínas: de 75mg/dl ,
Celularidad por linfocitos CLORUROS: 70 Meningitis tuberculosa
• Tx de meningitis en Recién Nacidos?
• AMPICILINA (e.coli)- CEFOTAXIMA (streptococo)
MENINGITIS
• Tx de meningitis en niños de 1 a 3 meses?
• AMPI + CEFO o CEFTRIAXONA + (valorar administración de DEXAMETASONA) para evitar
VENTRICULITIS
• Tx de meningits en px de 3 meses y menos de 50 años?
• CEFOTAXIMA + VANCOMICINA o CEFTRIAXONA + VANCOMICINA (cubrir cocos gram +, Y
bacilos gram -)
• Tx de meningitis en un px mas de 50 años con antecedente de alcoholismo o tiene DM
• AMPICILINA + VANCOMICINA + CEFALOSPORINA (tx p cocos gram + bacilos gram -)
• Px neutropenico, o px con meningitis con TRAUMATISMO CRANEOENCEFALICO, o que le
hicieron un procedimiento neuroquirurgico que tx se le dara?
• Cubrir Gram - (ceftazidil), cubrir Gram + meticilino resistente (Vancomicina)
• px que en el cultivo presenta meningococo que tx se da?
• PENICILINA G si es resistente: CEFTRIAXONA O CEFOTAXIMA y si es alérgico
VANCOMICINA (2 gr x dia)
• Px que le hicieron punción lumbar y obtuvieron en el LCR un cultivo Gram – que tx:
• Pseudomona aeroginosa: Ceftazidine(6gr x dia) Kliebsella, e coli: CEFOTAXIMA O
CEFTRIAZONA (4gr x dia)
• Tx para: Listeria: AMPICILINA(12 gr al dia) H. Influenza: CEFTRIAXONA o cefotaxima
Anaerobios: METRONIDAZOL (2 grx dia)
FOD
• Cual es la definición de FOD?
• R = Elevación de la temperatura por arriba de 38 GRADOS en varias determinaciones durante
mas de 3 SEMANAS sin llegar al diagnostico DESPUÉS DE 1 SEMANA de estudio hospitalario
• Cual es la definición de FOD en paciente hospitalizados?
• R = Paciente hospitalizado 24 HRS CON FIEBRE y 2 CULTIVOS NEGATIVOS después DE 2 DÍAS.
• Define FOD en paciente inmunodeficiente?
• R = FIEBRE >38.3 en paciente con NEUTROPENIA < 500 durante > 3 DÍAS y cultivos negativos
después DE 2 DÍAS.
• Define FOD asociado a VIH?
• R = Fiebre por encima de 38.3 GRADOS y que permanece sin diagnostico por MAS DE 3
SEMANAS
• Cuales son las principales causas de FOD?
1. Infección por TB y CMV
2. Neoplasia por linfoma o leucemia
3. Idiopática si después de 6m de estudio no hay causa
FOD
• Que posibilidades diagnosticas te sugieren los estudios de
BH en FOD?
1. Neutrofilia: Infecciones bacterianas
2. Linfocitosis: Virus, micobacterias, Brucella, Salmonella,
Rickettsia y Leishmania
3. Eosinofilia: Neoplasias, parasitosis y vasculitis.
4. Leucopenia: Neoplasia hematológica o inmunosupresión.
5. > VSG: Sepsis, polimialgia reumática y AR.
• Cual es el pronostico del FOD?
• R = Bueno aunque no se identifique la causa
FOD
• Cual es el mecanismo de acción por el cual las citocinas generan fiebre?
• Se liberan por los monocitos llegan por la sangre al SNC a nivel del
hipotálamo y estimulan la síntesis de PROSTAGLANDINAS E2 Y LA
PROTAGLANDINA S2 ALFA que a su vez estimula el centro termorregulador
para producir fiebre…
• Mecanismo por el cual los corticoides disminuyen la fiebre?
• Son inmunosupresores inhiben la fagocitosis entonces no hay liberación de
citocinas y x lo tanto no hay fiebre
• Que diferencia hay entre hipertermia y fiebre?
• En la hipertermia hay un daño a nivel hipotalámico (gralmente fármacos:
haloperidol, anestésicos inhalados p ej) y provocar una reacción
idiosincrática… en la hipertermia hay temperaturas de 40-41 grados y no
esta mediada por citocinas
• Efecto perjudicial de la fiebre en el embarazo?
• En los primeros meses puede provocar anencefalia..
FOD
• Mecanismo por el cual el paciente de larga evolución padece de anemia?
• Las CITOCINAS PROINFLAMATORIAS compite por la TRANSFERRINA
• Características del síndrome febril agudo?
• Paciente con fiebre menos de dos semanas de evolución, o fiebre cuantificada con
sintomatología inespecífica, por lo general debida principalmente a enf. Biliares son auto
limitadas
• Caract de la fiebre de origen a determinar?
• Fiebre mayor de 3 semanas que no cede, con fiebre mayor de 38 grados
• Etiología de la fiebre de origen indeterminado en un px con vih?
• Pneumocistis jirovecci, Micobacterium avium Intracelular, Linfomas
• Cáncer mas frecuente en los niños?
• Leucemia
• Y en los adultos jóvenes?
• Linfoma
• Características del síndrome de horner?
• Paciente con cáncer de pulmón, con ptosis y miosis, facies hipocráticas, EL TUMOR SE
ENCUENTRA DEL LADO DE LA CARA DONDE NO ESTA SUDANDO.
SEPSIS
• Como se diagnostica el síndrome de respuesta inflamatoria sistémica?
• R = Cuando se presentan 2 o mas de los siguientes parámetros:
1. FIEBRE corporal > 38 o hipotermia < 36
2. TAQUIPNEA con FR > 24 x minuto
3. TAQUICARDIA > 90 x minuto
4. LEUCOCITOSIS > 12,000 o leucopenia < 4,000 o 10% de bandas
• Como diagnosticas sepsis grave?
• R = Sepsis asociada a disfunción de un órgano con los siguientes datos:
1. Cardiovascular: TAS < 90 mm Hg que responde a fluidoterapia
2. Respiratoria: Relación FiO2/PaO2 < 200
3. Renal: Diuresis < 0.5 ML/KG/HR durante 1 hr a pesar de fluidoterapia
4. Trombocitopenia: < 80,000 o su reducción a la mitad con respecto a la obtenida
72 hrs antes
5. Acidosis metabólica: Con PH < 7.30
6. Choque séptico: Hipotensión con TAS < 90 durante 1 hr y que no responde a
fluidoterapia necesitando vasopresores
SEPSIS
• En caso de bacteriemia o sepsis cuales son algunas características
distintivas de los agentes patológicos?
A. Neisseria meningitidis: Se acompaña de PURPURA O PETEQUIAS
B. Rikettsia: Cuando aparecen LESIONES PETEQUIALES por mordida
de garrapata en zona endémica
C. Pseudomona aureoginosa: ECTIMA GANGRENOSO
D. S. Aureus o S. Pyogenes: ERITRODERMIA GENERALIZADA.
• Cual es la complicación mas frecuente de la bacteriemia?
• R = SIRPA caracterizado por HIPOXEMIA E INFILTRADOS
PULMONARES DIFUSOS.
FASCITIS NECROSANTE
• Cual es la secuencia de eventos para que sea manifiesta la fascitis
necrosante?
• R = Dolor – Fiebre – aumento de volumen local con eritema e hiperestesia –
epidermis se indura adquiriendo un color rojo/café donde se forman
vesículas - piel se necrosa y desprende
• Cuales son las variables de presentación de la fascitis necrosante?
A. GANGRENA DE FOURNIER: Afecta escroto, pene, perineo con la probable
extensión a muslos y abdomen
B. GANGRENA SINERGICA DE MELENEY: Da como resultado por la
combinación de S. Aureus + anaerobios.
• Cual es el agente causal de la fascitis necrosante?
• R = ESTREPTOCOCO GRUPO A
ABSCESO CEREBRAL
• Cual es la causa mas común de absceso cerebral?
• R = INFECCIONES CRÓNICAS de los oídos y senos paranasales, en pacientes
trasplantados es debido a hongos- Aspergillus
• Cual es el cuadro clínico del absceso cerebral?
• R = Similar al de un tumor endocraneal mas síndrome febril
• Localización mas frecuente del absceso cerebral?
• R = Frontal
• Cual es el manejo del absceso cerebral?
• R = CRANEOTOMÍA, PUNCIÓN O ASPIRACIÓN + PENICILINA G +
METRONIDAZOL considerando que el agente puede ser estreptococo,
neumococo o anaerobios mixtos.
ENCEFALITIS
• Cual es el agente etiológico de la encefalitis de San Luis?
• R = Mosquitos en EUA
• Cual es el agente etiológico de la encefalitis equina oriental?
• R = Afecta a caballos y a veces a humanos, se desconoce agente
• Cual es el agente etiológico de la encefalitis de La Crosse?
• R = ARDILLAS en el bosque.
• Cual es el agente etiológico de la encefalitis del Nilo Occidental?
• R = Esta se manifiesta en dichas regiones epidemiológicas.
• Cual es el manejo de las encefalitis virales?
• R = SOPORTE, no dar esteroides.
NEUROCISTICERCOSIS
• Cual es el agente infeccioso en neurocisticercosis?
• R = Tenia SOLIUM
• Cual es el cc de la neurocisticercosis?
1) Forma activa: Sin evidencia del parasito con hallazgos en la TAC de
calcificaciones o hidrocefalia
2) Parenquimatosa: Larvas en corteza y ganglios basales
3) Forma subaracnoidea: Con fibrosis leptomeningea provocando
neuropatías por atrapamiento o hidrocefalia
• Cual es el método diagnostico de elección para le
neurocisticercosis?
• R = TAC de elección e IRM en casos dudosos.
• Cual es el manejo de la neurocisticercosis?
• R = ALBENDAZOL 400 mg c/12 1 SEMANA, PRAZICUANTEL X 15 DÍAS.
SE RECOMIENDA EL USO DE PREDNISONA 1 DÍA ANTES DEL FÁRMACO
Y CONTINUAR CON DISMINUCIÓN DE LA DOSIS 14 DÍAS DESPUÉS.
TETANOS
• Cual es el cuadro clínico del tétanos?
• R = Comienza con ESPASMOS LEVES EN LA MANDÍBULA (trismo), el
cuello y la cara. La RIGIDEZ se desarrolla rápidamente en el TÓRAX,
ESPALDA, MÚSCULOS ABDOMINALES y en ocasiones la laringe
interfiriendo con la respiración. Los espasmos musculares son
contracciones súbitas, fuertes y dolorosas.
• Cual es el tratamiento de elección para tétanos?
• R = PENICILINA SÓDICA CRISTALINICA 20, 000 000. Se debe
administrar globulina inmunitaria antitetánica 5000 U/IM y una vez
recuperado el paciente se da esquema completo.
CASO CLINICO
• -A 22-year-old recent immigrant to the United States has never
been vaccinated for tetanus. He sustains a minor, but soil-
contaminated, injury. Which of the following statements is correct?
• (A) tetanus usually develops within 2 weeks following exposure
• (B) tetanus always develops within 4 hours following exposure in
patients who have not been previously immunized
• (C) tetanus may develop many months or years following exposure
in susceptible individuals
• (D) the usual incubation period for tetanus is 48 hours
• (E) tetanus may be prevented with penicillin
-In tetanus, an acute onset is usual. The median
onset is 7 days, and 90% present within 14 days
of injury. The organism is an anaerobic, motile
Gram-positive rod. It has the ability to survive for
years in the form of spores, which are resistant to
disinfectants and heat. Tetanus can occur in
nonimmunized individuals, or those who have
neglected their booster shots. Penicillin, or
metronidazole, is used in treatment, but their
efficacy is not clear
CARBUNO O ANTRAX
• Cual es el agente etiológico del Carbunco o ANTRAX?
• R = BACILLUS ANTHRACIS, bacilo GRAM +
• La forma cutánea de carbunco o ántrax que es la mas común como se manifiesta?
• R = Desarrollo de PÁPULA EN ZONA DE INOCULACIÓN, RODEADA DE VESÍCULAS, LA ZONA CENTRAL SE ULCERA
Y SECA DESARROLLÁNDOSE ZONA DEPRIMIDA DE COLOR NEGRO, NO DOLOROSO. Se resuelve
espontáneamente pero 20% puede ser fatal.
• Como se manifiesta la forma intestinal de carbunco o ántrax?
• R = FIEBRE, DISNEA, CIANOSIS, desorientación y signos de septicemia. Evoluciona rápidamente a choque, coma
y muerte.
• Como se manifiesta el carbunco o ántrax respiratorio y quienes lo PADECEN FRECEUNTEMENTE?
• R = Se manifiesta en personas que MANIPULAN PIEL Y LANAS. Se produce FIEBRE CON TOS NO PRODUCTIVA y
mal estar general, con buena evolución a 2-3 días y a continuación SÚBITAMENTE PRESENTA DIFICULTAD
RESPIRATORIA GRAVE, CIANOSIS Y SEPTICEMIA FATAL CON MUERTE EN <24 HRS.
• Como realizas el diagnostico de carbunco o ántrax?
• R = Identificación del bacilo con TINCIÓN GRAM, PCR.
• Cual es el manejo de carbunco o ántrax?
• R = CIPROFLOXACINO, levofloxacino o PENICILINA G.
• Como previenes el carbunco o ántrax?
• R = PROTECCIÓN DE PIEL, MUCOSAS Y LA VACUNACIÓN
SALMONELOSIS
• Cuantos antígenos tiene la salmonella typhi?
• R = 3: H FLAGELAR, K CAPSULAR y O SOMÁTICO
• Cuales son los sitios frecuentes de infección secundaria por salmonella typhi?
• R = Hígado, bazo, la medula ósea, PLACAS DE PEYER DEL ÍLEON TERMINAL y la vesícula biliar.
• Cual es el cuadro clínico característico de salmonella typhi?
• R = ROSÉOLA TIFOIDICA que se caracteriza por MACULAS ERITEMATOSAS DE 2-4 MM QUE BLANQUEAN A
LA PRESIÓN, localizadas en la parte superior del abdomen y tórax anterior y que por lo general dura de 2-3
días.
• Cual es la complicación mas frecuente de la salmonella typhi?
• R = La mas común es la HEMORRAGIA GI que resulta de la erosión de un vaso de la pared intestinal secundaria
a la necrosis de las placas de peyer en el íleon terminal
• Cual es el medio diagnostico mas sensible en la primer semana?
• R = HEMOCULTIVO, después el mielocultivo.
• Semana en la que se presentan las complicaciones de fiebre tifoidea?
• R = Fines de SEGUNDA Y TERCERA SEMANA
• Con que inoculo de S. Tiphy se desarrolla la enfermedad?
• R = 100,000
SALMONELOSIS
• A que semana aparece la roséola tifoidica, la cual aparece a la digito presión?
• R = 2da semana
• Como dx a un portador crónico de salmonella?
• R = COPROCULTIVOS + DURANTE 1ª
• En que semana los px se perforan o tienen hemorragias con choque hipovolemico?
• R = 3ERA semana
• En que semanas en la fiebre tifoidea se realizan los cultivos específicos?
• R = 1era HEMOCULTIVO, 2da MIELOCULTIVO y 3era COPROCULTIVO/urocultivo
• Cual es el cuadro clínico de la salmonelosis?
• R = Fiebre elevada 2-3 semanas, dolor abdominal, DIARREA EN SOPA DE
CHICHARO, ROSÉOLA, DELIRIO, ESPLENOMEGALIA.
• Cual es la prueba de laboratorio confirmatoria de fiebre tifoidea?
• R = 1:640, LEUCOPENIA y cuadro clínico característico
SALMONELOSIS
• Cual es el sitio donde suele albergarse la salmonella?
• R = Vesícula biliar
• Que se presenta en la primera semana de la salmonelosis?
• R = FIEBRE, HIPERSENSIBILIDAD ABDOMINAL, TIFLITIS (CUANDO HAY DOLOR EN FOSA ILIACA
DERECHA POR INVASIÓN DE LA PLACA DE PEYER Y CREPITA AL TACTO)
• Que se presenta en la segunda semana en la fiebre tifoidea?
• R = DIARREA, estreñimiento, delirio, EXANTEMA EN CARA ANTERIOR DE TÓRAX QUE SEDE A
LA DIGITO PRESIÓN. El sistema retículo-endotelial se hipertrofia con
HEPATO/ESPLENOMEGALIA, hiperplasia de las PLACAS DE PEYER. Bazo e hígado desarrollan
nódulos tifoideos.
• Cuales son las complicaciones habituales de la fiebre tifoidea en la 3era semana?
• R = HEMORRAGIA Y PERFORACIÓN INTESTINAL, peritonitis con placa simple de abd con aire
libre. Orquitis, meningitis, nefritis.
• Cual es el tratamiento de la salmonella multiresistente o en lugares o zonas endémicas?
• R = CIPROFLOXACINO, levofloxacino, ceftriaxona
SALMONELOSIS
• Cual es el mejor método diagnostico en un paciente que se
automedico en fiebre tifoidea?
• R = MIELOCULTIVO por que el medicamento no penetra a esa zona
• Tratamiento para fiebre tifoidea?
• R = CLORAMFENICOL 50/mg/kg por 2 SEMANAS
• Tratamiento para anemia + fiebre tifoidea?
• R = AMOXICILINA o ampicilina, NO DARLE CLORAMFENICOL por ke
causa aplasia medular.
• Tx fiebre tifoidea en embarazada?
• R = AMOXICILINA o ampicilina
SALMONELOSIS
• Tx fiebre tifoidea en niño con resistencia?
• R = QUINOLONAS NO, cefotaxima o CEFTRIAXONA
• En que casos se utiliza la dexametasona en fiebre tifoidea?
• R = En el ESTADO TIFOIDICO CON SHOQUE
• Que se le da al portador de salmonella?
• R = CIPROFLOXACINA por 3 MESES
• Que tx das a un px portador crónico que no respondió a
ciprofloxacino?
• R = COLECISTECTOMÍA, por que ahí vive la salmonella.
BRUCELOSIS
• Cuales son los principales vectores de la brucelosis y a quienes afecta
comúnmente?
• R= ORDEÑADORES, rastros, CARNICERO, veterinarios son vectores.
DERIVADOS DE LA LECHE como el queso o tejidos del animal afectado.
• Cuales son las cepas de brucella que produce enf en humano/ FIEBRE
ONDULANTE?
• R = MELITERSIS DE CABRA + frecuente a nivel mundial, SUIZ DEL CERDO +,
ABORTUS DE GANADO VOVINO +++.
• Microbiologicamente que es la brucella?
• R = COCOBACILO GRAM -, crece a 37 grados, es inmóvil a pesar de TENER
FLAGELO.
• Cual es la prueba de laboratorio para brucelosis?
• R = 2- MERCAPTOETANOL, es una inmunoglubulina IgG Vs BRUCELLA
BRUCELOSIS
• Cual es el medio de cultivo de elección para brucelosis?
• R = RUIZ CASTAÑEDA medio doble y ROSA DE BENGALA (fines
epidemiológicos)
• Que tratamiento utilizas de primera elección contra brucella?
• R = Combinado DOXICICLINA + GENTAMICINA de 3-6 semanas.
• Que tx utilizas en brucelosis en hueso y SNC?
• R = DOXICICLINA Y RIFAMPICINA
• Cual es el tratamiento de brucelosis en embarazada?
• R = TMP/SMZ+ RIFAMPICINA + ACIDO FÓLICO
• Cual es el tratamiento de niños menores de 12ª con brucelosis?
• R = TMP/SMZ + RIFAMPICINA
• Cual es el tratamiento en pacientes alérgicos a sulfas en brucelosis?
• R = RIFAMPICINA Y CEFTRIAXONA.
BOTULISMO
• Cual es el cuadro clínico del CLOSTRIDIUM BOTULINUM
transmitido por alimentos?
• R = Los síntomas comienzan en 6 HRS DESPUÉS A 2
SEMANAS, se manifiesta con DIPLOPÍA, VISIÓN BORROSA,
PTOSIS, disfagia, sensación de sequedad de mucosa oral,
DEBILIDAD MUSCULAR que afecta únicamente a los
hombros, miembros torácicos y mas tarde a los pélvicos.
• Cual es el manejo?
• R = Administración de TOXINA BOTULÍNICA con PREVENCIÓN
DE INSUFICIENCIA RESPIRATORIA que amerite ventilación
mecánica
CASO CLINICO
• - An 18-year-old woman has eaten homemade
preserves. Eighteen hours later, she develops
diplopia, dysarthria, and dysphagia. Which of the
following is the most likely causative organism?
• (A) Clostridium botulinum toxin
• (B) staphylococcal toxin
• (C) salmonellosis
• (D) brucellosis
• (E) shigellosis
PESTE BUBONICA
• Cual es el agente causal por la peste bubónica?
• R = YERSINIA PESTIS
• Cual es el cuadro clínico de la peste bubónica?
• R = Es la forma mas frecuente, se transmite por la PICADURA DE PULGA. Inicio con FIEBRE
ALTA, nauseas, MIALGIAS y ADENOPATÍAS DOLOROSAS REGIONALES, el PACIENTE
EVOLUCIONA A ESTUPOR, COMA Y MUERTE.
• Que formas clínicas tiene la peste?
• R = Peste bubónica, septicémica y neumónica
• Como diagnosticas peste?
• R = BIOPSIA DE GANGLIOS o EXUDADO FARÍNGEO
• Cual es el tratamiento de la peste?
• R = ESTREPTOMICINA, CLORANFENICOL O TETRACICLINAS.
• Cual es la prevención de la peste?
• R = Hay una vacuna con CEPA INACTIVADA F1
NAEGLERIA
• Cual es el agente causal de la enfermedad
transmitida en los manantiales?
• R = Naegleria
• Cuale es el tratamiento de naegleria?
• R = Anfotericina B
• Cual es el pronostico de naegleria?
• R = Muerte
RABIA
• Cual es el agente causal de la rabia?
• R = Virus de la familia Rabdovirus
• Cual es el cuadro clínico de la rabia?
• R = Dolor en el sitio de la mordedura, fiebre, malestar general, nausea y vomito. Diez días
mas tarde hay manifestaciones del SNC pudiendo ser encefálica o paralitica y las 2 formas
evolucionan a coma - muerte.
A. ENCEFALICA: Se caracteriza por delirio, somnolencia, hidrofobia (espasmos laríngeos
dolorosos al beber agua)
B. PARALITICA: Da una parálisis ascendente.
• Cual es el manejo de la rabia?
I. Es necesario observar al animal los siguientes 10 DÍAS tomando UNA BIOPSIA DEL CUERO
CABELLUDO y analizarla con Ac fluorescente o PCR.
II. Atención de la LESIÓN LAVANDO CON AGUA Y JABÓN ABUNDANTE A CHORRO DURANTE
10 MINUTOS, para MUCOSA ORAL O NASAL CON SOLUCIÓN FISIOLÓGICA DURANTE 5
MINUTOS. Desinfectar la herida con agua oxigenada o tintura de yodo.
III. Si se requiere SUTURAR LA HERIDA se debe APLICAR PRIMERO INMUNOGLOBULINA
ANTIRRÁBICA HUMANA y se aproximan los bordes o se dan puntos temporales en caso
que la herida sea profunda,
IV. En exposición leve se administra vacunación antirrábica en la región deltoidea los días 0-3-
7-14 Y 28.
V. Se APLICARA INMUNOGLOBULINA INTRALESIONAL LA PRIMER ½ Y LA OTRA MITAD VÍA IM.
VI. Se usara SUERO HETEROLOGO EN CASO EXTREMO DE NO CONTAR CON
INMUNOGLOBULINA.
ADENOVIRUS
• Donde se replica el adenovirus?
• R = Faringe, conjuntivas, intestino delgado, ganglios
linfáticos cervicales, preauriculares o mesentéricos.
• Cual es el cuadro clínico de adenovirus?
• R = CONJUNTIVITIS aguda, QUERATOCONJUNTIVITIS,
FIEBRE CONJUNTIVAL, FARINGITIS LARINGOTRAQUEITIS
(CRUP), bronquiolitis, neumonía, gastroenteritis,
linfadenitis mesentérica y CISTITIS HEMORRÁGICA.
INFLUENZA
INFLUENZA
• Cuales son las proteínas que distinguen al virus de
la influenza?
• R = H: HEMAGLUTININA y N: NEURAMINIDASA
• La influenza aviar que nomenclatura viral la
representa?
• R = H5N1 y es altamente mortal
DENGUE Y FIEBRE AMARILLA
• Cual es el mosquito transmisor del dengue?
• R = AEDES AEPYPTI.
• Cual es el cc del dengue?
1) El DENGUE CLÁSICO se inicia con FIEBRE, BRADICARDIA, pulso lento, EXANTEMA
MACULAR PUNTIFORME, tos, ardor faríngeo, MIALGIAS, ARTRALGIAS.
2) El DENGUE HEMORRÁGICO se caracteriza por aumento en la permeabilidad vascular con
clasificación del I-IV.
• Que es lo que distingue a la fiebre amarilla del dengue laboratorialmente?
• R = CUERPOS DE INCLUSIÓN VIRAL llamados CUERPOS DE COUNCILMAN EN LA SANGRE.
• Cual es el cuadro clínico de la fiebre amarilla?
• R = FIEBRE, cefalea, mialgias, DOLOR LUMBOSACRO, ERITEMA OCULAR Y FACIAL,
PETEQUIAS EN ENCIAS Y MUCOSA NASAL, HEMATEMESIS, MELENA, ALBUMINURIA,
encefalitis o meningoencefalitis.
• Cual es el manejo de la fiebre amarilla y del dengue?
• R = SINTOMÁTICO
VIH
• Que células ataca el VIH?
• R = CD4
• Cual es el ciclo vital del VIH?
• R = A través de la proteína gp41, se realiza la fusión, posteriormente la RNA del VIH se
descubre e interna en la célula afectada, la enzima transcriptasa inversa del virion cataliza la
transcripción inversa del RNA en DNA, este se transfiere hacia el núcleo en el cual se integra
en los cromosomas por medio de la integrasa. Los macrófagos actúan como reservorio del
VIH y lo diseminan a otros sistemas
• Que se debe realizar como prevención en una persona que halla sido puncionada de
manera accidental con una aguja utilizada previamente en un paciente con VIH?
• R = INHIBIDORES NUCLEOSIDOS DE LA TRANSCRIPTASA REVERSA
• Como se manifiesta el síndrome retroviral agudo SRA en VIH?
• R = SÍNTOMAS SIMILARES AL RESFRIADO O MONONUCLEOSIS, fiebre, escalofríos, sudores
nocturnos y erupciones en la piel que dura de 1-3 SEMANAS.
• De acuerdo a la CDC como se realiza dx de SIDA?
• R = Recuento <200/MM DE CD4 + la presencia de ENFERMEDAD OPORTUNISTA
VIH
• Como dx VIH?
• R = ELISA y confirmatorio con WESTERN BLOOD
• Cuales son parte del grupo de fármacos que inhiben la TRANSCRIPTASA INVERSA en VIH?
• R = ZIDOVUDINA, didanosina, zalcitabina.
• Cuales son las indicaciones actuales para uso de tratamiento retro vírico en VIH?
1. Síndrome de infección aguda
2. INFECCIÓN CRÓNICA: Enfermedad sintomática o asintomática con T CD4 <350/NL o RNA
DEL VIH CON > 50, 000 COPIAS
3. PROFILAXIS POST- EXPOSICIÓN: Se recomienda utilizar combinaciones como ZIDOVUDINA
+ LAMIVUDINA + NEVERIPINA
• Cuales son las indicaciones para cambiar el tratamiento antirretrovirico?
1. Disminución <1 log del RNA del VIH con plasma a las 4 semanas de haber iniciado el
tratamiento
2. Incremento DEL TRIPLE O MAS DEL VALOR PLASMÁTICO DEL RNA NO ATRIBUIBLE A
INFECCIÓN concomitantes o vacuolas.
3. DISMINUCIÓN DEL PORCENTAJE DE CÉLULAS TCD4
4. Deterioro clínico
5. Efectos adversos
VIH
• Como se manifiesta la neumonía por Pneumocystis Jiroveci?
• R = CD4 < 200/MM mas AUMENTO DE LA DHL, el diagnostico definitivo se obtiene por la TINCIÓN DE ESPUTO CON
WRIGHT-GIEMSA
• Cual es la causa mas frecuente de retinitis en pacientes con VIH?
• R = CITOMEGALOVIRUS
• Mecanismo por el que el VIH penetra a la célula?
• R = Pinocitosis
• Cual es la neoplasia maligna mas común en pacientes con VIH?
• R = Sarcoma de Kaposi
• Cual es el manejo del Sarcoma de Kaposi?
• R = Para la FORMA CUTÁNEA SE DA DOXORRUBICINA, para el INTESTINAL O VISCERAL SE DA
DOXORRUBICINA, BLEOMICINA Y VINBLASTINA.
• Como se maneja la profilaxis en pacientes con VIH de acuerdo al conteo de las células CD4?
1) CD4 < 200: VS P. JIROVECI con TMP/SMZ 1Tab/24 hrs C/3 DIA
2) CD4 75-100: VS M. AVIUM con CLARITROMICINA y se SUSPENDE CUANDO AUMENTAN LOS CD4
3) CD4 < 50: VS CITOMEGALOVIRUS con GANCICLOVIR
4) PPD con induración > 5 MM iniciar profilaxis VS M. TUBERCULOSIS con ISONIACIDA a 300 mg al día +
PIRIDOXINA durante 9-12 MESES.
VIH
• Tratamiento de primera elección para la retinitis por CITOMEGALOVIRUS?
• GANCICLOVIR
• Tratamiento para el herpes tipo 2 en pacientes con VIH?
• ACICLOVIR
• De que manera se puede inhibir el síndrome de desgaste?
• TALIDOMIDA
• Agente infeccioso de la lengua vellosa?
• Virus de EPSTEIN BARR ( relacionada con virus de mononucleosis y linfomas)
VIRUS TIPO 4
• Sitio mas común de lesión dermatológica, “SARCOMA DE KAPOSI” manchas color de piel?
• El sitio de afectación mas frecuente es la CARA EN PARPADOS Y PUENTE NASAL
RELACIONADO CON EL VIRUS TIPO 8
• Tratamiento de toxoplasmosis?
• PIREMITAMINA o sulfas + CLINDAMICINA en dosis altas
VIH
• Posibilidad de contagio por coito anal receptivo en el VIH?
• 1:100 1:30
• Zona donde se alojan el virus de la inmunodeficiencia es alta y útil para seguimiento del tx
de VIH?
• Biopsia de mucosa rectal otra opción es PCR
• Transmisión perinatal del VIH?:
• 13% al 40%
• Coito con inserción anal: 1: 1,000
• Transfusión: 1: 100,000
• Coito inserción vaginal: 1: 10,000 varia si la mujer se encuentra en periodo menstrual o si
tiene una ETS
• Aguja infectada?:
• 1-300
• Compartir agujas?:
• 1-150 (drogadictos)
• A que se refiere la regla de los 3 con relación a aguja infectada?
• Hepatitis B : 30% Hepatitis C: 3% HIV: .3%
VIH
• Diagnostico para valorar el tratamiento “respuesta terapéutica”
• RCP se realiza al mes y medio y a los 3 meses y posteriormente CADA 6 MESES (no se usa para diagnostico)
• Que pasa con una persona que tiene VIH y las siguientes enfermedades Concomitantes?
• Hepatitis B: Indiferente (no pasa nada)
• Hepatitis C: Agresividad
• HEPATITIS G: FRENA LA PROGRESIÓN DEL VIH
• Con respecto a la estatificación de etapas A-B-C y grados I al III?
• A: asintomáticos B: Síntomas Constitucionales (fiebre, perdida de peso, diarrea) C: Infecciones Oportunistas
(neumocistis jirovecci, linfoma del SNC, Criptosporidiasis, Toxoplasma del SNC, Micovacterium Avium intra
celular)
• I.- 500 copias II.-MAS DE 200 Y MENOS DE 500 III.-MENOS DE 200 copias
• INHIBIDORES DE TRANSCRIPTASA INVERSA -no análogo de los Nucleosidos- NNRTI
• De la ZIDOVUDINA AZT cual es el efecto indeseable? PROBLEMAS MEDULARES, anemia severa,
trombocitopenia, leucopenia
• DE LA EFEVIRENZ cual es el efecto indeseable?
• Insomnio y pesadillas (DEPRESIÓN)
• INHIBIDORES DE PROTEASAS
• Del Indinavir efecto indeseable mas frecuente: FORMADOR DE CÁLCULOS cólico renoureteral
• Ritonavir + lopinavir : Es el caletra efecto sinérgico
VIH
• Profilaxis (exposición en trabajadores de la salud)(exposición SEXUAL) que maniobras o
procedimientos?
• Tiene una posibilidad de infección de 1:300 SE REALIZA UN ELISA con fines legales, ( AL
MES)… si se considera EXPOSICIÓN DE BAJO RIESGO dar AZT+3TC X UN MES se REALIZA
ELISA SI ES NEGATIVO SE SUSP… si es exposición de alto riesgo AZT+3TC +UN TERCER
MEDICAMENTO…. NO DAR TX A LAS 72 POSTEXPOSICION
• Copias de CD4?
• Entre 50 y 100 copias en el momento del dx: MVAIC, CMV, Linfomas del SNC
• MENOS DE 200: Criptocococis, toxoplasmosis, NEUMOCISTIS JIROVECCI
• Entre 200 y 500: LENGUA VELLOSA, Sarcoma de Kaposi, TB Pulmonar, Infección por virus
herpes, candidiosis
• Px con HIV y tiene exposición con px con tb?
• Profilaxis: PPD MAS DE 5MM ISONIACIDA + RIFAMPICINA + PIRAZINAMIDA POR 12 MESES
+ PIRIDOXINA
• En que consiste el síndrome retroviral agudo?
1) Paciente con infección de VIH con SÍNTOMAS A CORTO PLAZO (de 2 A 3 SEMANAS y
sucede en el 40%de los casos) Y CONSISTE EN FIEBRE ADENOPATÍA, RASH CUTÁNEO
COLOR SALMON Y HEPATOMEGALIA… CUADRO MUY PARECIDO AL DE LA
MONONUCLEOSIS INFECCIOSA,
2) DX ELISA A LAS 6 SEMANAS Y 3 MESES si sale positivo se realiza el WESTERN BLOT PARA
CONFIRMARLO
VIH
• Combinaciones aprobadas:
I. Tenofovir (TDF)+ emtricitabina (FTC) y efavirenz (EFV)
II. Atazanavir (ATV) + ritonavir (rtv)+tenofovir (TDF) / emtricitabina (FTC) (si no funciona el primer
esquema)
III. Caletra +tenofovir (TDF) / emtricitabina (FTC) (El caletra se puede cambiar con AZT + 3TC)
• Efectos colaterales?
I. Zidovudina….. ANEMIA por hipoplasia medular
II. Nerfinavir…. DIARREA
III. Didanosina…. PANCREATITIS
IV. Zalcitabina….. NEUROPATÍA PERIFÉRICA
V. Estaduvidina…. NEUROPATÍA PERIFÉRICA (lamivudina +zidovudina) 3TC….. Neuropatia periférica
VI. Abacavir….. FIEBRE RASH, HIPERSENSIBILIDAD FATAL
VII. Tenofovir…. TOXICIDAD RENAL
VIII. Saquinavir…. LÍPIDOS ELEVADOS
• Px con embarazo + VIH Cual es el tx?
I. ZIDOVUDINA (2do y 3er trimestre)… se continua en el trabajo de parto (para evitar la
infección vertical)
II. POSPARTO (NO LACTANCIA 10-20 tiene posibilidad de infectar) 2da opción LAMIVUDINA
VIH
• Cuales son parte del grupo de fármacos INHIBIDORES DE
PROTEASA en VIH?
• R = SaquinAVIR, ritonAVIR, indinAVIR.
• Cuales son parte del grupo de fármacos INHIBIDORES DE LA
FUSIÓN?
• R = Enfubitida, que entre sus efectos adversos se encuentra la
hipersensibilidad y neumonía bacteriana
• Cuales son parte del grupo de fármacos que inhiben la
TRANSCRIPTASA INVERSA en VIH?
• R = ZIDOVUDINA, didanosINA, zalcitabINA.
CASO CLINICO
-A 19-year-old man has donated blood for the first time. Despite having
no risk factors for human immunodeficiency virus (HIV) infection, his
blood tests positive for HIV by enzyme immunoassay (EIA). Which of
the following statements is correct?
• (A) EIA is currently the most specific test for HIV
• (B) he might have a false-positive secondary to an unsuspected
collagen-vascular disease
• (C) he has a 75% chance of truly being infected with HIV
• (D) EIA is an excellent screening test
• (E) a Western blot test would be more sensitive
-EIA is an excellent screening test for HIV infection
as it is positive in over 99.5% of cases. However, it
lacks specificity, and in low risk populations, only
about 10% of EIA positive results are true
positives. Recent influenza vaccination, acute
viral infections, and liver disease are common
causes for false positives. The Western blot test is
more specific and is the usual confirmatory test,
although even more specific tests are now
available.
ASPERGILOSIS
• Como se adquiere la aspergilosis?
• R = Por inhalación de ESPORAS DEL HONGO, las cuales se encuentran en HOJAS SECAS,
GRANEROS DE MAIZ, ESTIÉRCOL Y VEGETALES EN DESCOMPOSICIÓN.
• En los pacientes con rinitis crónica por aspergilus, que datos de laboratorio encuentras en
el moco?
• R = Rico en EOSINOFILOS y cristales de CHARCOT-LEYDEN
• En que consiste la aspergilosis pulmonar endobronquial saprofita o aspergiloma?
• R = En el CRECIMIENTO DEL HONGO dentro de las cavidades pulmonares, SECUNDARIAS
GENERALMENTE A TB, SARCOIDOSIS, HISTOPLASMOSIS O BRONQUIECTASIAS.
• Como dx aspergilus?
• R = CULTIVO DE SABOURAD, detección de ANTICUERPOS, galactomanano en suero (chbts
en su pared celular) y biopsia
• Cual es el tratamiento de aspergilus?
• R = Esteroides, ANFOTERICINA B, LOBECTOMÍA EN CASO DE ASPERGILOMA o debridacion de
tejido infectado.
• Cual es el tratamiento de elección en caso de toxicidad secundaria al DESOXICOLATO de
ANFOTERICINA B en caso de aspergilosis invasiva?
• R = VORICONAZOL 6mg/kg/dia.
BLASTOMICOSIS
• Cual es el agente causal de blastomicosis?
• R = BLASTOMYCES DERMATITIDIS
• Cuales son las manifestaciones clínicas de blastomicosis?
• R = PULMONAR Y CUTÁNEA. NEUMONÍA aguda que no responde al tratamiento y
CURA ESPONTÁNEAMENTE. Las LESIONES CUTÁNEAS predominan en las ZONAS
EXPUESTAS, característicamente son PAPULAS, NÓDULOS O PLACAS BIEN
DELIMITADAS, únicas o múltiples, NO DOLOROSAS NI PRURIGINOSAS, que
evolucionan a lesiones verrugosas, costras o ulceras.
• Como diagnosticas blastomicosis?
• R = De ELECCIÓN CULTIVO pero tarda mucho, se pueden identificar organismos con
la TINCIÓN DE PLATA, METENAMINA O ACIDO PERIODICO DE SCHIFF en la biopsia
de los tejidos infectados o en la citología de esputo. Donde se observan CÉLULAS
LEVADURIFORMES CON YEMAS DE BASE AMPLIA.
• Cual es el tratamiento de blastomicosis?
• R = En la forma LEVE - ITRACONAZOL, en la forma GRAVE - ANFOTERICINA B
CANDIDOSIS
• Como se manifiesta la CANDIDIASIS DIGESTIVA y como se trata?
• R = Se ASOCIA a lesiones ORALES Y PERIANALES, produce DIARREA CON MOCO, el
diagnostico se realiza con el EXAMEN DIRECTO DE LAS HECES CON KOH y se trata
con NISTATINA ORAL.
• Como se manifiesta la CANDIDIOSIS URINARIA y como se trata?
• R = Se presenta como CISTITIS, pielonefritis, BOLAS FÚNGICAS RENALES, ABSCESOS
RENALES o necrosis de las papilas. Se trata con fluconazol o ANFOTERICINA B.
• Como se diagnostica CANDIDIOSIS PULMONAR y como se trata?
• R = Cuando realizas HEMOCULTIVOS POSITIVOS o ANTÍGENO MANAN DEL
SUERO. El tratamiento se realiza con ANFOTERICINA B, en los pacientes que no la
toleran se utiliza FLUCONAZOL.
• Como se manifiesta la ENDOCARDITIS por candidiosis?
• R = FIEBRE persistente, SOPLOS CARDIACOS y ESPLENOMEGALIA, en algunos casos
hay embolización a grandes arterias, como la iliaca o femoral. LA VÁLVULA MAS
AFECTADA ES LA MITRAL.
CANDIDOSIS
• Como diagnosticas ENDOCARDITIS por cándida?
• R = Ecocardiagrafia, hemocultivos seriados y detección de
ANTÍGENO MANAN en suero?
• Cual es el tratamiento de endocarditis por cándida?
• R = RESECCIÓN DE VÁLVULA afectada y ANFOTERICINA B
• Como se presenta la candidiasis diseminada aguda o sepsis?
• R = Se presenta en PACIENTES NEUTROPENICOS, CON FIEBRE,
LEUCOCITOSIS, TROMBOCITOPENIA, inestabilidad
hemodinámica y choque.
CRIPTOCOCOCIS (AVES, PALOMAS)
• Cual es el agente causal de la criptococosis y donde se
alberga?
• R = CRYPTOCOCCUS NEOFORMANS, se encuentra en HECES
DE PALOMAS (CUIDADORES DE AVES) O NIDOS
• Como diagnosticas criptococosis?
• ANTÍGENO MANAN en suero o biopsia.
• En LCR con TINCIÓN DE TINTA CHINA
• Como se trata la criptococosis?
• R = ANFOTERICINA B y 5-fluorocitosina
HISTOPLASMOSIS (CAVERNAS)
• Donde se alberga el Histoplasma capsulatum?
• R = En forma de moho en suelo RICO EN NITRÓGENO, como el de
los GALLINEROS, o donde duermen las aves y CAVERNAS DE
MURCIÉLAGOS
• Como diagnosticas histoplasmosis?
• R = El método de elección es el CULTIVO, pero tarda mucho y
haces frotis de los fluidos o tejidos infectados, las muestras para
cultivo en HISTOPLASMOSIS DISEMINADA SE OBTIENEN DE
MEDULA ÓSEA, HÍGADO, GANGLIOS LINFÁTICOS O LESIONES
MUCOCUTANEAS.
• Cual es el manejo de la hispoplasmosis?
• R = Anfotericina B
MUCORMICOSIS
• Cual es el agente etiológico de la mucormicosis?
• R = MUCOR, ABSIDIA, RIZOPUS denominado ficomicetos.
• Como se manifiesta la infección de los senos paranasales en mucormicosis?
• R = Predominantemente a PACIENTES CON DM mal tratada, con FIEBRE,
congestión nasal, SECRECIÓN NASAL SANGUINOLENTA, DOLOR SINUSAL Y
DIPLOPÍA, a la exploración física se observa DISMINUCIÓN DE LOS MOVIMIENTOS
OCULARES, quemosis y PROPTOSIS y SI INVADE EL SENO FRONTAL PRODUCE COMA.
• Como se manifiesta la mucormicosis gastrointestinal?
• R = Se disemina al peritoneo produciendo la muerte en 70 días
• Como se diagnostica mucormicosis?
• R = FROTIS EN FRESCO donde se ven HIFAS HIALINAS o en BIOPSIAS donde se
observa NECROSIS ISQUÉMICA o hemorrágica
• Cual es el manejo de la mucormicosis?
• R = El tratamiento consiste en altas dosis de ANFOTERICINA B, ASEO QUIRÚRGICO
DE LOS TEJIDOS.
COCCIDIOIDOMICOSIS
• Cuales son los signos y síntomas de la coccidioidomicosis?
• R = TOS, RINORREA, febricula, dolores musculares y en la RADIOGRAFÍA DE
TÓRAX CON CAVITACIÓN.
• - A 43-year-old man developed a cough shortly after returning from a 1-
month hiking trip in California. While there, he was hiking in the central
California valleys. During his trip, he had developed a “flu-like” illness
consisting of fever, cough, and muscle pains, which resolved spontaneously.
A CXR shows a thin-walled cavity in the right upper lobe, and the sputum
reveals fungal elements. Which of the following is the most likely causative
organism?
• (A) ringworm
• (B) Cryptococcus neoformans
• (C) Candida albicans
• (D) mycobacteria
• (E) coccidioidomycosis
- Coccidioidomycosis is the usual cause of
pulmonary cavitation resulting from fungal
infection. A rarefaction may be demonstrable
in a pneumonic lesion within 10 days of onset
PALUDISMO
• Cual es el agente causal de paludismo o malaria?
• R = PARASITOSIS intracelular eritrocitaria por PLASMODIUM, que se transmite
por el MOSQUITO ANOPHELES
• Cuantas especies de plasmodium existen?
• R = 4, P. falciparum, P. vivax, P. ovale y P. malariae
• Cual es el ciclo biológico del plasmodium?
• R = Los mosquitos se infectan al ingerir sangre infectada con micro y
macrigametocitos. Dentro del mosquito, se multiplica sexualmente en su intestino,
produciendo los ooquistes, y al madurar se rompen liberando ESPOROZOITOS, que
migran a las glándulas salivales del mosquito y son la forma infectante en los seres
humanos. Ya en el TORRENTE SANGUÍNEO viajan a hígado donde se reproducen y
forman los MEROZOITOS que tienen la capacidad de infectar los eritrocitos.
• Que produce la lisis eritrocitaria al contraer paludismo?
• R = La PLASMEPSINA, que es una proteasa del acido aspartico que DEGRADA LA
HEMOGLOBINA.
PALUDISMO
• Cual es el tipo de genero en paludismo mas frecuente?
• R = P. FALCIPARUM, es la mas frecuente y de peor pronostico con
fiebre > 40
• Como se diagnostica el paludismo?
• R = OBSERVACIÓN DE PARASITO EN FROTIS SANGUÍNEO O GOTA
GRUESA. La TINCIÓN DE GIEMSA permite observar las
granulaciones de Schuffner.
• Como se trata el paludismo?
• R = QUININA Y CLOROQUINA. El SULFATO DE QUININA se usa en
RESISTENCIA A CLOROQUINA
BORRELIOSIS O ENFERMEDAD DE
LYME
• Cual es el agente causal de la enfermedad de Lyme?
• R = Espiroqueta BORRELIA BURGDORFERI GRAM (-) de color violeta con
tinción Giemsa que se CULTIVA EN MEDIO NOGUCHI. Se adquiere por
MORDEDURA DE GARRAPATA del genero IXODES.
• Cual es el cuadro clínico de la enfermedad de Lyme?
• R = Se divide en 3 etapas
I. ERITEMA CRÓNICO O MIGRANS, pápulas eritematosas que se EXTIENDEN
DE FORMA CENTRIFUGA DEJANDO LA PARTE CENTRAL DEL CUERPO
SANA, acompañada de síntomas generales como astenia, adinamia,
fiebre, mialgias y cefalea.
II. Alteraciones neurológicas, cardiacas y reumáticas
III. POLIARTRITIS MIGRATORIA, alteraciones dermatológicas y neurológicas
• Como se diagnostica la enfermedad de Lyme?
• R = ELISA o PCR
BORRELIOSIS O ENFERMEDAD DE
LYME
• Tratamiento de elección para enf de Lyme?
• R = DOXICICLINA por mes y medio
• Cual es el tratamiento para borreliosis en su etapa
precoz?
• R = DOXICICLINA.
• Cual es el tratamiento para borreliosis en su etapa
tardía?
• R = CEFTRIAXONA
LEPTOSPIROSIS
• Cual es el vector de la leptospirosis, que enzimas produce y que nutrientes necesita?
• R = Leptospira interrogans en ORINA DE RATA. PRODUCE CATALASA Y HIALURODINASA y requieren
TIAMINA B1 Y VITAMINA B12 PARA SU DESARROLLO
• Cuales son las manifestaciones clínicas de la Leptospirosis?
• R = De inicio súbito, con FIEBRE, derrame conjuntival, epistaxis, MIALGIAS, cefalea intensa, EXANTEMA Y
HEPATOMEGALIA.
• Que es el síndrome de Weil en Leptospirosis?
• R = Al cursar la primera fase, viene una ASINTOMÁTICA para posteriormente PRESENTARSE
LEPTOSPIROSIS ANICTERICA.
• Como se manifiesta el síndrome de Weil?
• R = CON FIEBRE, CEFALEA, MIALGIAS, EXANTEMA, miocarditis, ICTERICIA, insuficiencia hepática,
INSUFICIENCIA RENAL, FENÓMENOS HEMORRÁGICOS, LEPTOSPIURIA y daño al SNC (MENINGITIS
ASEPTICA).
• Cual es la triada clásica del síndrome de Weil?
• R = Daño en HÍGADO, RIÑÓN y SNC.
• Cual es el tratamiento de la Leptospirosis?
• R = PENICILINA, estreptomicina o tetraciclinas
TULAREMIA
• Cual es el agente causal de Tularemia?
• R = FRANCISELLA TULLARENSIS POR PICADURA DE GARRAPATA
• Cual es el cuadro clínico de Tularemia?
• R = De inicio súbito con FIEBRE, ASTENIA, ADINAMIA, hiporexia, ataque al
estado general, posteriormente hay diversas formas clínicas:
ULCEROGANGLIONAR, manifestada con adenomegalias y ulceras cutáneas;
TIFOIDE GLANDULAR, donde se afecta el pulmón, ganglios, hígado y bazo.
SE PRESENTA CON NEUMONÍA, NECROSIS VISCERAL Y SEPSIS.
• Como diagnosticas Tularemia?
• R = En medio de CULTIVO DE AGAR SANGRE cisteína o ELISA.
• Como tratas la Tularemia?
• R = ESTREPTOMICINA, tetraciclina.
RICKETSIA
FIEBRE Q
LEISHMANIASIS
• Que vectores transmiten leishmaniasis?
• R = MOSQUITOS del genero LUTZOMYA Y PHLEBOTOMUS, LEISHMANIA pertenece a la familia
del tripanosoma
• Cual es el ciclo biológico de la leishmaniasis?
• R = Al picar el mosquito a un mamífero infectado, ingiere los macrófagos parasitados, que
posteriormente se destruyen, dejando libre al parasito en su forma flagelar e infectante
(promastigote), se reproducen por fision binaria longitudinal, y migran hacia la porción anterior
del aparato digestivo, hasta la probocide (boca). La infección se produce al alimentarse de un
nuevo huésped, inocula los PROMASTIGOTES, que al ser fagocitados se transforman en
AMASTIGOTES, donde se multiplican, causan la lisis de las células que los contienen e infectan a
otras células.
• En la leishmaniasis viceral o Kala-Azar a que células afecta el parasito?
• R = A los MACRÓFAGOS del sistema retículo endotelial
• Como se manifiesta la leishmaniosis cutánea?
• R = En el sitio de inoculación, PAPULA ERITEMATOSA/PRURIGINOSA – NODULO – ULCERA PIEL/TEJ.
CEL. SUB – EXUDA LIQ. SEROHEMATICO – ADENOPATÍA REGIONAL.
• Como se manifiesta la leishmaniosis mucocutanea o espundia?
• R = ULCERA CUTÁNEA – pasan years – LESIÓN OBSTRUCTIVA ÚNICA/MULTIPLE –
LARINGE/TABIQUE NASAL/ANO Y VULVA.
LEISHMANIASIS
• Como se manifiesta la leishmaniasis viceral o Kala-Azar (fiebre negra)?
• R = Hay HIPERPIGMENTACIÓN CUTÁNEA DE LAS EXTREMIDADES, mal estar general, DOLOR ABDOMINAL, HEPATO-
ESPLENOMEGALIA, LINFADENOPATIAS, DIARREA es fatal si no se trata adecuadamente.
• Como diagnosticas leishmaniasis?
• R = VER EL AMASTIGOTE EN BIOPSIAS DE PIEL, lesiones mucosas, hígado, ganglios linfáticos, medula ósea tenidas con
Giemsa. En los CULTIVOS de los tejidos se busca el PROMASTIGOTE.
• Con que prueba de laboratorio diagnosticas leishmaniasis?
• R = INTRADERMORREACCIÓN DE MONTENEGRO y ELISA.
• Cual es el tratamiento de la leishmaniasis viceral?
• R = Para la FORMA VISCERAL el tratamiento DE ELECCIÓN ES ESTIBOGLUCONATO DE SODIO O ANTIMONIATO DE
GLUTAMINA IM POR 50 DÍAS.
• Que medicamentos empleas en caso de que la leishmaniasis visceral no responda a los antiamoniales?
• R = ANFOTERICINA LIPOSOMAL POR 5 DÍAS Y EL ISETHIONATO DE PENTAMIDINA.
• Que medicamento utilizas en leishmaniasis cutánea?
• R = ISETIONATO DE PENTAMIDINA, paromomicina APLICACIÓN TÓPICA 2 VECES AL DÍA POR 15 DÍAS.
GIARDIASIS
• Cual es el agente causal de giardiasis y a que gpo de edad afecta comúnmente?
• R = GIARDIA INTESTINALIS de la clase MASTIGOPHORA, de la FAMILIA HEXANITIDAE y afecta a
menores de 5 años
• Cual es el ciclo biológica de la giardiasis?
• R = Los TROFOZOITOS presentes en el duodeno, avanzan por el intestino y debido a la escasa
cantidad de agua en el colon, cambian de forma a su estado quístico. El quiste permanece como la
forma infectante y es ELIMINADO CON LA MATERIA FECAL. Al ser INGERIDO POR EL HUÉSPED,
pasa por el tubo digestivo y los ácidos gástricos rompen la pared liberando al trofozoito, que
nuevamente pasa al duodeno, donde madura, se fija a la pared intestinal y se replica por fision
binaria longitudinal y es arrastrado con la materia fecal.
• Que déficit nutricional puede ocasionar la giardiasis?
• R = Déficit de vitaminas KADE, B12 Y ACIDO FÓLICO.
• Como se manifiesta la giardiasis?
• R = En niños cursa asintomática. NAUSEA , VOMITO, DIARREA CON MOCO Y ESTEATORREA.
GIARDIASIS
• Como diagnosticas giardiasis?
• R = Observación de QUISTE EN HECES, ELISA.
• Que componente de los trofozoitos de giardiasis te causa
hipotrofia de las vellosidades intestinales
• R = El DISCO SUCTOR
• Cual es el tratamiento de elección para la giardiasis?
• R = El TINIDAZOL con dosis única de 2 gr. METRONIDAZOL. La
furazolidona causa hemolisis en pacientes con déficit de 6-
GDP.
AMEBOSIS
• Cual es el agente causal de la Amebosis?
• R = Entamoeba dispar y ENTAMOEBA HYSTOLITICA.
• Como se desplaza el quiste de la E. Hystolitica?
• R = Pseudopodos
• Cual es el ciclo biológico de la entamoeba hystolitica?
• R = Ingestión de quistes maduros, desenquistamiento en intestino delgado y
los TROFOZOITOS liberados tetranucleados multiplican sus núcleos para
formar una ameba de ocho núcleos, que posteriormente se fragmenta en
ocho amebas, llamadas amébulas meta quísticas, que maduran a
trofozoitos y migran a colon, donde se alimentan de bacterias y restos
celulares; se reproducen por fisión binaria y producen quistes que se
eliminan por las heces.
• Como se unen los parásitos del intestino grueso de E. Hystolitica?
• R = Mediante la lectina
AMEBOSIS
• Cual es el cuadro clínico abdominal de la amebiasis por E. Hystolitica?
1. COLITIS AMEBIANA AGUDA manifestada por dolor abd tipo cólico, tenesmo y
EVACUACIONES DIARREICAS CON MOCO Y SANGRE.
2. COLITIS FULMINANTE con mortalidad mayor al 50% se presenta con diarrea sanguinolenta
intensa, ataque al estado general, fiebre, dolor abdominal intenso y PERFORACIÓN
INTESTINAL con choque séptico secundarios a la extensión de la ulcera hasta la serosa.
3. AMEBOMA O GRANULOMA AMEBIANO que cursa con DIARREA SANGUINOLENTA Y
OBSTRUCCIÓN INTESTINAL.
• Cual es el cuadro clínico de la amebosis cutánea?
1. ULCERAS FUGEDENICAS (de COLOR PURPURA, de crecimiento rápido, MUY DOLOROSAS y
ABUNDANTE MATERIAL NECRÓTICO),
2. LESIONES VEGETANTES (friables, de evolución subaguda, LOCALIZADAS EN LOS PLIEGUES
CUTÁNEOS),
3. ENTAMOEBOSIS (se presenta por una reacción de hipersensibilidad, CON LESIONES
SEMEJANTES A URTICARIA.
• Como se manifiesta el absceso hepático amebiano?
• R = De inicio agudo, CON HEPATOMEGALIA Y DOLOR EN HIPOCONDRIO DERECHO QUE SE
IRRADIA HACIA EL HOMBRO.
AMEBOSIS
• Como diagnosticas amibiasis?
• R = COPROPARISOTOSCOPICO EN FRESCO, hematoxilina férrica o TINCIÓN
DE GOMORI, en busca de TROFOZOITOS, ELISA. En la piel se busca
trofozoito.
• Cual es el tratamiento de amibiasis?
1) METRONIDAZOL, TINIDAZOL.
2) Dehidrohemetina y emetina son amebicidas de la pared intestinal, pero no
sobre la luz intestinal siendo estos YODOQUINOL y paramomicina.
• Que medicamento es eficaz para la amebiasis hepática?
• R = CLOROQUINA
TRIPANOSOMIASIS AFRICANA O
ENFERMEDAD DEL SUENO
• Cual es el agente etiológico de la TRIPANOSOMIASIS AFRICANA O ENFERMEDAD DEL
SUEÑO y cual es su vector?
• R = Tripanosoma BRUCEI, Tripanosoma RHODESIENSE y Tripanosoma GAMBIENSE. Su
vector es la MOSCA TSE TSE.
• Cual es el agente causal de la tripanosomiasis americana o ENFERMEDAD DE CHAGAS
y cual es su vector?
• R = TRIPANOZOMA CRUZI y su vector es la chinche
• Cual es el ciclo biológico de los tripanosomas?
• R = Al alimentarse de un individuo con la infección, los vectores ingieren el amastigote,
que se multiplica dentro del tracto digestivo y las FORMAS INFECCIOSAS
(TRIPOMASTIGOTE) son eliminadas en las heces; la infección se produce al depositar las
heces en una herida de la piel antes de picar en las membranas mucosas o en la
conjuntiva. Posteriormente, se multiplican en el sitio de la entrada, y entran a la sangre
como tripomastigotes circulantes e invaden a las células, CON PREDILECCIÓN POR EL
MIOCARDIO, MUSCULO LISO Y SNC.
TRIPANOSOMIASIS O ENFERMEDAD
DE CHAGAS
• Como se manifiesta la tripanosomiasis o ENFERMEDAD DE CHAGAS POR T. CRUZY?
• R = En la piel, en el sitio de entrada se observa CHAGOMA que es PARECIDO A LA FURUNCULOSIS CON
LINFADENOPATIA REGIONAL. En conjuntiva, edema bipalpebral unilateral, CONJUNTIVITIS Y
LINFADENITIS PRE AURICULAR (SIGNO DE ROMANA) CON FIEBRE Y ESPLENOMEGALIA LEVE.
CRONICAMENTE se manifiesta por INSUFICIENCIA CARDIACA PREDOMINANTE DEL LADO DERECHO, con
arritmias ventriculares y episodios de tromboembolia sistémica o pulmonar. MEGA ESÓFAGO Y
MEGACOLON con síntomas de disfagia, constipación intensa, hipertrofia parotidea.
• Como se diagnostica la tripanosomiasis americana o ENFERMEDAD DE CHAGAS?
• R = En la FASE AGUDA se observa con la TINCIÓN DE GIEMSA de sangre anti coagulada. Diagnostico
serológico se basa en hemaglutinación indirecta o ELISA.
• Como se diagnostica la TRIPANOSOMIASIS AFRICANA?
• R = Para T. GAMBIENSE por medio de PRUEBAS DE AGLUTINACIÓN y para T. RHODESIENSE es mediante
la observación del parasito en la tinción de GIEMSA.
• Cual es el tratamiento de elección para la TRIPANOSOMIASIS AMERICANA O CHAGAS?
• R = NIFURTIMOX por 30-90 DÍAS o BENZIDIMAZOL de 30-90 DIAS.
• Cual es el tratamiento de la TRIPANOSOMIASIS AFRICANA?
• R = Para T. GAMBIENSE ES PENTAMIDINA POR 10 DÍAS. Para T. RHODESIENSE ES LA SURAMINA
POR 7 DÍAS.
BABEIOSIS
• Cual es el agente causal de la babeiosis y cual es su vector?
• R = Enfermedad INTRAERITROCITARIA, causada por PROTOZOARIO BABEZIA MICROTI Y DIVERGENS,
transmitida por el vector que es una GARRAPATA de nombre IXODES SCAPULARIS
• Cual es el ciclo biológico de la babesiosis?
• R = Cuando las GARRAPATAS I. SCAPULARIS se alimentan de un animal infectado, ingieren los parásitos en el
intestino, se produce la esquizogonia y posteriormente los merozoitos que infectan las células epiteliales del
vector, y son transmitidas a glándulas salivales por la vía linfática y alimentarse del nuevo huésped le
transmiten la infección.
• Como se observan los parásitos de la babeiosis en los eritrocitos?
• R = INCLUSIONES EN FORMA DE CRUZ lo que causa ANEMIA HEMOLÍTICA
• Cual es el cuadro clínico de la babeiosis?
• R = Clínicamente se manifiesta por FIEBRE, ANEMIA HEMOLÍTICA, cefalea, diaforesis, MIALGIAS, artralgias,
fatiga, ictericia, ESPLENOMEGALIA.
• Como diagnosticas la babeiosis?
• R = Identificación del PARASITO INTRAERITROCITARIO EN FROTIS CON TINCIÓN DE GIEMSA. ANTICUERPOS
ENTRE 1ERA Y 4TA SEMANA. En bh se observa ANEMIA, TROMBOCITOPENIA y LEUCOCITOSIS leve.
• Cual es el tratamiento de la babeiosis?
• R = Puede resolverse sin tratamiento. Para LA BABEIOSIS SINTOMÁTICA ES CON QUINIDINA MAS
CLINDAMICINA.
• Que tipo de babeiosis tiene peor pronostico?
• R = B. DIVERGENS con 40% de muertes.
TENIASIS
• Cual es el agente causal de la teniasis?
• R = T. saginata y T. solium
• Cuales son las partes estructurales que componen a la tenia?
• R = En su extremo anterior tienen una estructura llamada ESCÓLEX (CABEZA), seguida de un
cuello. En su escólex presentan cuatro ventosas orales. En sus ventosas tiene un róstelo, con
una corona doble de ganchos (escolices). El CUERPO (ESTRÓBILO) se conforma por varios
segmentos llamados PROGLOTIDOS, que en su interior tienen ÓRGANOS SEXUALES.
• Cuanto miden los adultos de T. SAGINATA?
• R = 2-7 METROS de largo
• Cuanto puede llegar a medir la T. SOLIUM?
• R = 15-18 METROS
• Cual es el ciclo biológico de la teniasis?
• R = La teniasis intestinal es causada por la INGESTA DE CISTICERCOS. Al legar a intestino
delgado, se unen a la pared intestinal. De los 2 a los 4 meses de la infección, alcanzan su
madurez, caracterizada por la presencia de proglotidos gravidos que liberan huevos y se
excretan por las heces. La cisticercosis ocurre por la ingestión de los alimentos o agua
contaminados con excremento humano que contiene huevos o proglotidos. Los huevos se
abren en el intestino, liberando a las oncoesferas, que invaden la pared intestinal pasando al
torrente sanguíneo y linfático para migrar hacia musculo estriado, cerebro e hígado.
TENIASIS
• Cual es el cuadro clínico de la teniasis?
• R = Normalmente es asintomática. Cuando cursa con síntomas son:
PLENITUD ABDOMINAL, FATIGA, DOLOR LEVE OCASIONAL E HIPOREXIA.
• Que déficit vitamínico cursa con teniasis?
• R = Déficit de VITAMINA B12
• Cual es el cuadro clínico de CISTICERCOSIS en el SNC?
• R = CRISIS CONVULSIVAS, cefaleas, vomito, HIPERTENSIÓN INTRACRANEAL.
• Como diagnosticas la teniasis?
• R = COPROPARASITOSCOPICO SE OBSERVAN HUEVOS, ELISA.
• Cual es el tratamiento para teniasis?
• R = PRAZICUANTEL a DOSIS ÚNICA DE 10 MG/KG. Ninguna droga mata a
los huevos de tenia solium. NICLOSAMIDA para TENIA SAGINATA
ASCARIASIS
• Cuanto mide el macho y la hembra de áscaris?
• R = La HEMBRA es mas grande mide 25-30 CM y el MACHO 25 CM.
• Cual es el ciclo biológico de la ascariasis?
• R = La infección ocurre por la ingestión de huevos maduros. Los huevos al
ser eliminados por las heces, no tienen envoltura y son infecciosos a las 2-3
semanas en el suelo o el agua. Al ser ingeridos eclosionan en el intestino,
las larvas inmaduras penetran la pared intestinal y SE DIRIGEN HACIA LOS
ALVEOLOS A TRAVÉS DEL TORRENTE SANGUÍNEO, posteriormente,
ASCIENDEN POR LOS BRONQUIS HASTA LA FARINGE, DONDE SON
DEGLUTIDOS COMPLETANDO SU DESARROLLO EN EL INTESTINO DELGADO.
• Cual es el cuadro clínico de la ascariasis?
• Pulmonar: SÍNDROME DE LOFFLER (NEUMONÍA EOSINOFILICA) POR
MIGRACIÓN ALVEOLAR, CON TOS PRODUCTIVA, DISNEA, sibilancias, dolor
retroesternal, FIEBRE.
• Intestino, cursa asintomático pero puede ocasionar APENDICITIS,
OBSTRUCCIÓN DE CONDUCTOS BILIARES.
ASCARIASIS
• Como diagnosticas ascariasis?
• R = Hallazgo de HUEVOS POR COPROPARASITOSCOPIO POR CONCENTRACIÓN DE
FLOTACIÓN (FAUST) O POR SEDIMENTACIÓN (RITCHIE) QUE ES CUALITATIVO.
Método cuantitativo de Stoll y Kato-Katz/ Kato-Miura para determinar numero de
parásitos.
• Cual es el tratamiento de la ascariasis?
• R = ALBENZAZOL Y EL PAMOTATO DE PIRANTEL son los medicamentos de elección.
• En que momento del embarazo puedes tratar ascariasis?
• R = En el 3ER TRIMESTRE
• Como actúa el pamotato de pirantel?
• R = Inhibe la acetilcolinesterasa y bloquea la transmisión de la placa
neuromuscular.
• Cuales son algunas de las complicaciones extraintestinales de la ascariasis?
• R = Pancreatitis, colangitis, colecistitis, absceso y perforación hepática, obstrucción
de vías respiratorias y rotura del conducto onfalomesenterico.
OXIUROSIS
• Cual es el agente causal de la Oxiurosis?
• R = Nematodo ENTEROBIUS VERMICULARIS, tiene la forma de alfiler.
• Cual es el ciclo biológico de la Oxiurosis?
• R = Los parásitos adultos SE ENCUENTRAN EN EL CIEGO, donde copulan y una vez que han
terminado LA HEMBRA MIGRA HACIA EL ANO, depositando DURANTE LAS NOCHES SUS
HUEVOS EN LA REGIÓN PERIANAL, secretando una sustancia que PRODUCE PRURITO
INTENSO, lo que ocasiona el rascado y la contaminación de las manos.
• Cual es el cuadro clínico de la oxiurosis?
• R = PRURITO PERIANAL O VULVAR NOCTURNO.
• Como diagnosticas oxiurosis?
• R = CON CINTA ADHESIVA PERIANAL observando los parásitos CON TINCIÓN GRAM
• Cual es el tratamiento de la oxiurosis?
• R = Se debe tratar a toda la familia con ALBENDAZOL 400 MG/DOSIS ÚNICA
ESTRONGILOIDOSIS
• Cual es el agente causal de la estrongiloidosis?
• R = Helminto STRONGYLOIDES STERCORALIS
• Cual es el ciclo biológico de la estrongiloidosis?
• R = Los adultos copulan en el yeyuno del huésped, y las hembras fecundadas entran a la pared intestinal y
permanecen en la submucosa, los huevos larvados se desprenden de la submucosa y en la luz intestinal liberan
a la larva rabditoide, pudiendo transformarse en larvas filariformes las cuales pueden penetrar el intestino
pasando a la circulación donde digieren a diferentes tejidos.
• De donde se obtiene la estrongiloidosis?
• R = De CAMINAR DESCALZO, atraviesan la piel causando infección
• Cual es el cuadro clínico de la estrongiloidosis?
1) En el sitio DONDE PENETRA LA LARVA filariforme, se PRESENTAN LESIONES PAPULOSAS, ERITEMATOSAS Y
PRURIGINOSAS CON EDEMA LOCAL E INFLAMACIÓN.
2) Si es en LA REGION PERIANAL SE PRESENTA COMO DERMATITIS PERIANAL RADIADA.
3) Intestinales: se manifiestan por ulceras, DIARREA, DOLOR EPIGÁSTRICO URENTE, meteorismo, melena.
4) Pulmonar: con SÍNDROME DE LOFFLER, con tos, irritación traqueal, fiebre y hemoptisis.
• Como diagnosticas la estrongiloisosis?
• R = El COPROPARASITOSCOPICO por concentración- flotación (Faust) se OBSERVAN LAS LARVAS.
• Cual es el tratamiento de la estrongiloidosis?
• R = IVERMECTINA o ALBENDAZOL
TRIQUINOSIS
• Cual es el agente causal de la triquinosis?
• R = TRICHINELLA SPIRALIS la cual se obtiene al consumir CARNE DE CERDO
• Cual es el ciclo biológico de la triquinosis?
• R = Consumiendo carne de cerdo, los jugos gástricos liberan las larvas enquistadas, maduran en el intestino
delgado donde se reproducen. Posteriormente atraviesan la mucosa intestinal y se diseminan por la vía
linfática/sanguínea hacia otros tejidos, especialmente musculo estriado pobre en glucógeno y de mayor
actividad (diafragma, intercostales, bíceps, cuádriceps, lengua y maseteros) provocando una reacción
inflamatoria.
• Cual es el cuadro clínico de la triquinosis?
• R = Se manifiesta en 3 fases.
1) Fase intestinal: con dolor mesogastrico tipo cólico, mal estar general, nausea, vomito, cefalea y diaforesis.
2) Fase de migración: edema, fiebre, diaforesis, fotofobia, conjuntivitis,
3) Fase muscular: DOLOR MUSCULAR QUE EXACERBA CON EL EJERCICIO.
4) A la ex fis se observan hemorragias en forma de astilla en unas retina y sub conjuntivales.
• Como se realiza el diagnostico la triquinosis?
• R = Coproparasitoscopio, biopsia de músculos dolorosos tenidos con hematoxilina y eosina con elevación de la
CPK.
• Cual es el tratamiento de la triquinosis?
1) Fase intestinal se da ALBENDAZOL 400 mg/12 x 15 días.
2) Fase muscular es intrahospitalario con antiparasitarios mas AINES.
FASCIOLOSIS
• Cual es el agente causal de la fasciolosis?
• R = FASCIOLA HEPÁTICA
• Cual es el huésped intermediario de la fasciola hepática?
• R = Los CARACOLES
• Cual es el ciclo biológico de la fasciola hepática?
• R = Al ingerir las metacercarias, llegan al intestino delgado eclosionando, dejan salir las alrcas que atraviesan la
pared intestinal y se dirigen a hígado, hasta llegar a los conductos biliares, donde se establecen para
desarrollarse. Al fecundar en el hígado los huevos pasan al duodeno y se expulsan con la materia fecal,
posteriormente el opérculo se abre para dejar salir al miracidio, el cual es una larva ciliada, que le permite
desplazarse y penetrar a los caracoles pulmonados donde se desarrolla el esporoquiste.
• Cual es el cuadro clínico de la fasciolosis?
• R = DOLOR INTESTINAL, FIEBRE ALTA, HEPATO-ESPLENOMEGALIA, ICTERICIA con presencia de microabscesos y
necrosis.
• Como diagnosticas la fasciolosis?
• R = EOSINOFILIAELEVADA, COPROPARASITOSCOPIO SERIADO
• Cual es el tratamiento de elección para la Fasciolosis?
• R = El BIOTINOL, se puede usar DEHIDROHEMETINA, METRONIDAZOL Y PRAZICUANTEL.
ELEFANTIASIS O FILARIOSIS LINFÁTICA
• Cuales son los agentes etiológicos de la elefantiasis o filariosis linfática?
• R = WUCHERERIA BANCROFTI y B. malayi
• Cual es el agente causal de la Filariosis?
• R = Microfilaria
• Cual es el vector de la filariosis?
• R = MOSQUITOS O MOSCAS.
• Cual es el ciclo biológico de la filariasis?
• R = Las microfilarias son ingeridas por insectos hematófagos y transmitidas a los
huéspedes donde se ALOJAN EN SANGRE Y TEJIDO LINFÁTICO
• Cual es el cuadro clínico de la filariasis?
• R = FILARIASIS LINFÁTICA: FIEBRE CON/SIN INFLAMACIÓN DE GANGLIOS Y VASOS
LINFÁTICOS, PUEDE HABER EPIDIDIMITIS U ORQUITIS. En la infección crónica causa
OBSTRUCCIÓN LINFATICA CAUSANDO EDEMA, HIDROCELE, LINFEDEMA ESCROTAL,
VARICES LINFÁTICAS Y ELEFANTIASIS.
ELEFANTIASIS O FILARIOSIS LINFÁTICA
• Que te provoca la QUILURIA de la filariasis?
• R = La ROTURA DE LOS VASOS LINFÁTICOS EN EL TRACTO URINARIO
• Como se manifiesta la oncocercosis transmitida por la inoculación de larvas en la
picadura de mosca en filariasis?
• R = El parasito se aloja en la piel formando nódulos con edema y eritema local.
• Cuales son los agentes causales en la oncocercosis o ceguera de los ríos en
filariasis?
• R = ONCHOCERCA VOLVULUS
• Como diagnosticas la filariasis?
• R = Se observan las MICROFILARIAS EN SANGRE Y LINFA. Se toma biopsia de piel.
• Como tratas la filariosis?
• R = DIETILCARBAMACEPINA
ESQUISTOSOMIASIS
• Cual es el ciclo biológico de la esquistosomiasis?
• R = Los huevos son eliminados por las heces, al tener contacto con el agua se liberan las larvas sin
la espina (miracidios), que nadan hasta encontrar un caracolinvadiendo los músculos donde se
transforman en esporocitos, estas salen del caracol en forma de cercarías mismas que penetran
en la piel del hombre cuando nada y se alimentan de eritrocitos.
• Cual es el cuadro clínico de la esquistosomiasis?
1) DERMATITIS CERCARÍA: DONDE HAY PAPULAS ERITEMATOSAS Y PRURIGINOSAS EN EL SITIO DE
LA INOCULACIÓN. En algunos casos se presentan manifestaciones alérgicas caracterizadas por
fiebre, urticaria y eosinofilia.
2) FIEBRE DE KATAYAMA: FIEBRE, MIALGIAS, URTICARIA, tos no productiva, dolor abd, diarrea,
eosinofilia y HEPATOESPLENOMEGALIA.
3) CRONICIDAD POR SCHISOTOMA MANSORI O JAPONICUM: DIARREA SANGUINOLENTA y
hepatoesplenomegalia.
4) CRONICIDAD POR SCHISOTOMA HEMATOBIUM HAY DISURIA, HEMATURIA Y PROTEINURIA,
POSTERIOREMENTE HAY PÓLIPOS EN VEJIGA, GLOMERULONEFRITIS, CISTITIS Y URETRITIS.
• Como diagnosticas esquistosomiasis?
• R = COPROPARASITOSCOPICO identificando los huevos, o en la ORINA, se puede realizar biopsia o
tinción de ELISA.
• Cual es el tratamiento de elección de la esquistosomiasis?
• R = PRAZICUANTEL 20 MG/KG C/12 HRS
TRICOCEFALOSIS/ TRICHURIS
TRICHIRIA
• Cual es el agente causal de la tricocefalosis y a quienes afecta principalmente?
• R = TRICHURIS TRICHURIA y afecta principalmente a niños.
• Cual es el ciclo biológico de la tricocefalosis?
• R = Ingestión de alimentos con huevos larvados. Los huevos eclosionan en el intestino delgado y migran
hacia el ciego, se introduce en la mucosa y deposita sus huevos en el lumen siendo excretados con las
heces.
• Cual es el cuadro clínico de la trichuris trichiria?
• R= DOLOR ABDOMINAL TIPO CÓLICO, pujo, tenesmo, diarrea, distensión abdominal y PROLAPSO
RECTAL.
• Cual es el cuadro clínico de trichuris en niños?
• R = Síndrome disentérico, ANEMIA FERROPENICA, palidez, PROLAPSO RECTAL Y RETARDO EN EL
CRECIMIENTO.
• Como se diagnostica la trichuriasis o tricocefalosis?
• R = COPROPARASITOSCOPIO por concentración-flotacion (Faust). También se pueden utilizar los
CRISTALES DE CHARCOT LEIDEN que son los productos de degradación de los eosinofilos.
• Cual es el tratamiento de elección de tricocefalosis?
• R = ALBENDAZOL Y HIERRO
TOXOCARIASIS
• Cual es el agente causal de la toxocara?
• R = TOXOCARA CANIS, TOXOCARA CATI.
• Cuanto mide el parasito de toxacara?
• R = Menor al de Ascaris lumbricoides 5-10 CM
• Cual es el ciclo biológico de la toxocarosis?
• R = Huevos eliminados en la materia fecal, son ingeridos por el hombre, intestino delgado, penetran a pared
intestinal y migran por torrente sanguíneo a hígado, pulmones, SNC, corazón y los ojos provocando la
FORMACIÓN DE GRANULOMAS EOSINOFILOS.
• Cual es el cuadro clínico de la toxocariasis?
• R = El órgano MAS AFECTADO ES EL HÍGADO, donde se producen GRANULOMAS y los PULMONES
MANIFESTADA COMO SÍNDROME DE LOFFLER, cursando con tos, fatiga, disnea, prurito, dolor abdominal,
hepatomegalia, esosinofilia, hiperglobulinemia (IgE). La toxocariasis ocular donde hay estrabismo y deterioro
visual unilateral donde se observa un granuloma eosinofilico.
• Como realizas el diagnostico de toxocariosis?
• R = ELISA. Hipergamaglobulinemia IgE. En LESIONES HEPÁTICAS se observan lesiones HIPOECOICAS.
• Cual es el tratamiento de elección la toxocariasis?
• R = ALBENDAZOL 400 MG CA/12 X 21 DIAS.
CASO CLINICO
• Cual es el cuadro clínico en RN o lactantes de toxoplasmosis cerebral y que
resultados en TAC se observan?
• R = The infection has a predilection for the CNS and the eye, and produces
encephalitis in utero. Symptoms can be evident in the first few days of life. Infants
born with active disease may have fever, rash, seizures, and hepatosplenomegaly
at birth. En la TAC se observan calcificaciones cerebrales
• - A 6-month-old child presents with recurrent seizures and poor development. The
evaluation reveals a baby with hydrocephalus, impaired movement of the
extremities, hypotonia, and retinal abnormalities. A computerized tomography
(CT) scan demonstrates large ventricles and calcified lesions. Which of the
following is the most likely diagnosis?
• (A) Tay-Sachs disease
• (B) congenital hydrocephalus
• (C) kernicterus
• (D) toxoplasmosis
• (E) congenital neurosyphilis
HERPES VIRUS SIMPLE 2
Cual es la manifestación clínica de herpes virus simple 2?
R = VESÍCULAS CON BASE ERITEMATOSA con hipertrofia ganglionar
local
-A 23-year-old woman develops vesicular lesions on an erythematous
base on her vulvar area. She has tender lymphadenopathy and
dysuria as well. Which of the following is the most likely causative
organism?
• (A) cytomegalovirus (CMV)
• (B) gonococcus
• (C) herpes simplex virus type 2 (HSV-2)
• (D) Treponema pallidum
• (E) varicella zoster
-HSV-2 genital infections may be associated with
fever, malaise, and anorexia. Vesicular lesions
usually ulcerate rapidly and become covered
with exudate. There is a 90% chance of
recurrent symptoms in the first year following
a primary infection. HSV-1 genital infections
are similar, but the chance of recurrence is
less.
OOFORITIS
• Cual es la complicación mas común de parotiditis en las mujeres?
• R = Ooforitis
-An 18-year-old woman presents with headache, anorexia, chilly sensations,
and discomfort on both sides of her jaw. She has also noticed discomfort
in both lower abdominal quadrants. Physical examination reveals bilateral
enlarged parotid glands that are doughy, elastic, and slightly tender; with a
reddened orifice of Stensen’s duct. Her abdomen is soft with bilateral
lower quadrant abdominal tenderness; a temperature of 38.5°C; and a
pulse rate of 92/min. Laboratory data show hemoglobin 13 g/dL;
hematocrit 40%; white blood cells (WBC) 9000/mL, with 35% segmented
neutrophils, 7% monocytes, and 58% lymphocytes. Which of the following
is the most likely cause for her abdominal pain and tenderness?
• (A) mesenteric lymphadenitis
• (B) oophoritis
• (C) gonorrhea
• (D) peritoneal metastases
• (E) intestinal hyperperistalsis
-Pain referring to either or both lower quadrants
is common when oophoritis is present. Fever
usually accompanies oophoritis. Sterility is not
a consequence of mumps oophoritis.
CASO CLINICO
- A 23-year-old woman visits your office because of headache, malaise,
anorexia, pain in both sides of her jaw, and discomfort in both lower
abdominal quadrants. Physical examination reveals enlarged
parotid glands; bilateral lower quadrant abdominal tenderness; a
temperatura of 38.7°C; and a pulse rate of 92/min. Serologic testing
(IgM) confirms the diagnosis of mumps. Which of the following is
the most appropriate treatment for this condition?
• (A) symptomatic
• (B) immunization
• (C) broad-spectrum antibiotics
• (D) sulfonamides
• (E) steroids
- Antibiotics, sulfas, steroids, and mumps
convalescent sera are of no value. Mouth care,
analgesics, and a bland diet are usually
recommended. Glucocorticoids are usually
prescribed for orchitis, although definite
evidence of their effectiveness is lacking.
Prevention via vaccination is the preferred
strategy for mumps.
ETS
ENFERMEDAD POR RASGUNO DE
GATO
• Cual es el agente causal de la enfermedad por rasguño de gato?
• R = BARTONELLA HENSELEAE
• -A 30-year-old man develops a pustular lesion at the site of a cat
scratch on his forearm. This is followed 1 week later by malaise,
fever, and lymphadenopathy. On examination, he has tender axillary
lymph nodes. Which of the following is the most likely causative
organism?
• (A) Bartonella henselae
• (B) Bartonella bacilliformis
• (C) Bartonella quintana
• (D) Coxiella burnetii
• (E) Borrelia burgdorferi
SARAMPION
• Cuales son los signos y síntomas de Sarampión?
• R = TOS, CORIZA Y CONJUNTIVITIS CON FOTOFOBIA. MANCHAS DE
KOPLIC (Lesión blanquecina sobre base eritematosa en mucosa
subyugal). Exantema que inicia cabeza, progresión céfalo-caudal
RESPETA PALMAS Y PLANTAS.
CASO CLINICO
-A 7-year-old child, unvaccinated because of his parents’ religious
beliefs, develops malaise, cough, coryza, and conjunctivitis with a
high fever. Examination of his mouth reveals blue white spots on a
red base beside his second molars. The next day he develops an
erythematous, nonpruritic, maculopapular rash at his hairline and
behind his ears, which spreads over his body. Which of the
following is the most likely diagnosis?
• (A) hand-foot-and-mouth disease (coxsackievirus)
• (B) measles (rubeola)
• (C) rubella (German measles)
• (D) mumps
• (E) pertussis
- This is a typical case of measles. The Koplik’s spots in the
mouth are easily missed with poor illumination. They
are white blue spots of 1 mm on a red background and
are not seen in any other infectious disease. The rash
of measles becomes confluent while that of rubella
does not. Pneumonia is an infrequent complication but
accounts for many measles deaths. Giant cell
pneumonia is also seen, most commonly in children
suffering with a severe disease such as leukemia or
immunodeficiency. Aerosolized ribavirin has been used
to treat severe pneumonia secondary to measles, but
its efficacy is still unclear. The other potentially lethal
complication of measles is encephalitis
CASO CLINICO
- An 8-year-old boy from an impoverished innercity area has
never been vaccinated appropriately. He develops fever,
cough, and coryza. The next day, blue white spots develop
on the buccal mucosa. On the third day, an erythematous,
nonpruritic maculopapular rash develops on the face and
spreads over the entire body. Which of the following is the
most likely complication?
• (A) pneumonia
• (B) encephalitis
• (C) otitis media
• (D) bronchitis
• (E) mastoiditis
- The most common complication of measles is
otitis media, other complications include
mastoiditis, pneumonia, bronchitis,
encephalitis, and lymphadenitis. Otitis media
is usually a bacterial superinfection, and
should be treated with antibiotics.
CASO CLINICO
- A6-year-old boy develops symptoms of cough, fever, and malaise
followed by a generalized maculopapular rash that has spread from
the head downwards. A clinical diagnosis of measles is made. A few
days after the onset of the rash he is drowsy, lethargic, and
complaining of headache. A lumbar puncture,
electroencephalogram (EEG), and computerized tomography (CT) of
the brain exclude other etiologies and confirm the diagnosis of
encephalitis. Which of the following is the most likely delayed
neurologic complication of measles virus encephalitis?
• (A) meningitis
• (B) pure motor paralysis
• (C) autonomic neuropathy
• (D) mental retardation or epilepsy
• (E) “stocking-glove” peripheral neuropathy
- Subacute sclerosing panencephalitis causes
involuntary spasmodic movements and
progressive mental deterioration, frequently
ending in death within a year. It usually occurs in
children whose measles occurred at an early age
(= 2 years). It occurs 6–8 years after the primary
infection. It presents with nonspecific symptoms
such as poor school performance or mood and
personality changes. It then progresses to
intellectual decline, seizures, myoclonus, ataxia,
and visual disturbances. Continued deterioration
results in inevitable death.
MONONUCLEOSIS
• Cual es el cuadro clínico de un paciente con CMV/Mononucleosis?
• R = Faringo- amigdalitis, ADENOPATÍAS CERVICALES- AXILARES ESPLENOMEGALIA.
• - A 21-year-old woman visits her physician because of 3 weeks of a “flu-like” illness.
She reports symptoms of malaise, fever, fatigue, and a sore throat. There is no
weight loss or night sweats, and she has not traveled out of country. Her past
medical history is not significant and she is not taking any medications. Physical
examination is normal except for enlarged cervicallymph nodes. Laboratory data
show hemoglobin 13.2 g/dL; hematocrit 42%; platelets 380,000/mL; WBC 8500/mL,
with 35% segmented neutrophils, 1% eosinophils, and 64% lymphocytes, of which
36% were atypical. A heterophil antibody (sheep cell agglutination) test is negative.
Which of the following is the most likely causative organism?
• (A) herpes simplex
• (B) echovirus
• (C) CMV
• (D) coxsackievirus
• (E) reovirus
-The most common cause of non-EBV
mononucleosis-type syndrome is CMV. It is the
most common presentation of CMV in
nonneonates with normal immune function.
MONONUCLEOSIS INFECCIOSA O
ENFERMEDAD DEL BESO
• Cual es el agente causal de la mononucleosis infecciosa o enfermedad del
beso?
• R = EPSTEIN BARR y es un herpes virus
• A que patologías se asocia el virus del Epstein Bar?
• R = LINFOMA DE BURKITT, linfoma de células T, LEUCOPLASIA ORAL
VELLOSA, carcinoma nasofaríngeo.
• Como dx al virus de Epstein bar?
• R = AGLUTININAS HETEROFILICAS, MONO test.
• Cual es el tx de Epstein Bar?
• R = ACICLOVIR
CASO CLINICO
• -A 17-year-old man presents with new symptoms of fatigue, malaise,
fever, and a sore throat. He has no significant past medical history and is
not on any medications. Physical examination is entirely normal except for
enlarged, palpable cervical, lymph nodes. He reports no weight loss or
night sweats. Laboratory investigations include a normal chest x-ray,
negative throat swab, but abnormal blood film with atypical lymphocytes.
The hemoglobin is 15.5 g/dL; hematocrit 42%; platelets 290,000/mL; WBC
10500/mL, with 45% segmented neutrophils, 1% eosinophils, and 54%
lymphocytes, of which 36% were atypical. Which of the following is the
most appropriate initial diagnostic test?
• (A) lymph node biopsy
• (B) bone marrow
• (C) erythrocyte sedimentation rate (ESR)
• (D) heterophil antibody (sheep cell agglutination) test
• (E) hepatic biopsy
-The presence of IgG antibodies by the indirect
immunofluorescence test indicates recent or
prior EBV infection. IgM antibodies indicate
recent infection only. Heterophil antibodies are
present in 50% of children and 90–95% of
adolescents and adults with infectious
mononucleosis. Monospot tests are the best
diagnostic tools but may not turn positive until
the second or third week of the illness. Specific
EBV antibodies and cultures are rarely used.
HERPEANGINA
• Que enfermedades causa el VIRUS COXSACKIE?
• Grupo A : HERPEANGINA, ENFERMEDAD MANO-PIE-BOCA y
conjuntivitis hemorrágica aguda, el
• Grupo B:pleurodinia, miocarditis, pericarditis y
meningoencefalitis.
• Cuales son las características de la herpeangina?
• R = LESIONES VESICULOSAS Y ULCEROSAS EN FARINGE,
PARTICULARMENTE EN LOS PILARES ANTERIORES, las
amígdalas y el paladar, vomito, anorexia, fatiga y ataque al
estado general.
CASO CLINICO
- A previously healthy 19-year-old female university student
develops myalgia, headache, fever, and malaise. Blood tests
reveal lymphocytosis, with 20% of the lymphocytes being
atypical. She remains tired and unwell for 6 weeks, but
repeated tests for heterophil antibody are negative. Which
of the following is the most likely diagnosis?
• (A) Epstein-Barr virus (EBV) infection
• (B) primary HIV infection
• (C) human herpes virus type 7 (HHV-7)
• (D) CMV infection
• (E) toxoplasmosis
VALVULAS NATIVAS
• Cual es el agente causal de endocarditis en caso de reemplazo valvular con
válvulas nativas?
• R = STAPHYLOCOCCUS EPIDERMIDIS
- 60-year-old man presents with fever and malaise 6 weeks after mitral valve
replacement. On examination, his temperature is 38 C, blood pressure 130/80 mm
Hg, pulse 80/min, and a loud pansystolic murmur at the apex, which radiates to the
axilla. He has no skin or neurologic findings. Which of the following is the most
likely causative organism?
• (A) Staphylococcus aureus
• (B) a fungus
• (C) Staphylococcus saprophyticus
• (D) pneumococcus
• (E) Staphylococcus epidermidis
- About half of all early-onset (<60 days after
surgery) prosthetic endocarditis is caused by
staphylococcal infection, with S. epidermidis
predominating. Early-onset prosthetic
endocarditis is generally the result of
intraoperative contamination of the prosthesis
or a bacteremic postoperative complication.
CASO CLINICO
- A 56-year-old man is having intermittent fevers and malaise for the past 2
weeks. He has no other localizing symptoms. Two months ago, he had
valve replacement surgery for a bicuspid aortic valve. Amechanical valve
was inserted and his postoperative course was uncomplicated. On
examination, his temperature is 38 C, blood pressure 124/80 mm Hg,
pulse 72/min, and head and neck are normal. There is a 3/6 systolic
ejection murmur, the second heart sound is mechanical, and a 2/6 early
diastolic murmur is heard. The lungs are clear and the skin examination is
normal. Three sets of blood cultures are drawn and an urgent
echocardiogram is ordered. Which of the following is the most likely
causative organism?
• (A) Staphylococcus aureus
• (B) S. epidermidis
• (C) S. viridans
• (D) enterococci
• (E) Candida
- S. epidermidis is still the leading cause of
prosthetic valve endocarditis in the early
postoperative period (usually defined <2
months). It is usually the consequence of
intraoperative contamination or postoperative
bacteremia. Endocarditis occurring 12 months
after surgery is usually due to the same
organisms that cause native valve
endocarditis.
ENDOCARDITIS
• Cuales son los agentes infecciosos involucrados en la endocarditis sub aguda?
• R = Estreptococos del grupo viridans
• Cual es el tratamiento para los estreptococos del grupo viridans?
• R = PENICILINA + AMINOGLUCOSIDO POR 30 DÍAS
• Cual es el agente causal de endocarditis de Litman Sax?
• R = AUTOINMUNE, común en LUPUS.
• Cual es el procedimiento terapéutico profiláctico para intervención cardiaca o
dental?
• R = AMOXICILINA 2 GR 1 HR ANTES DEL PROCEDIMIENTO, si es ALÉRGICO
CLINDAMICINA 600 MG, AZITROMICINA 500 MG 1 HR ANTES DEL EVENTO.
• En endocarditis meticilino resistente que medicamento das?
• R = TEICOPLANINA, en gram (-) cefalosporinas de 3era
NEUMONIA
• Cual es el principal agente patógeno en neumonía adquirida en la comunidad?
• R = Streptococcus pneumoniae
• -A 73-year-old man from a nursing home develops headache, fever, cough, sore
throat, malaise, and severe myalgia during a community outbreak affecting
numerous other residents at the home. The symptoms gradually resolve after 3
days, and he starts feeling better but then there is a reappearance of his fever,with
cough and yellow sputum production. On examination, his temperature is 38.5 C,
pulse 100/min, respiration 24/min, oxygen saturation 88% and crackles in the right
lower lung base, bronchial breath sounds and dullness on percussion. CXR reveals a
new infiltrate in the right lower lobe. Which of the following is the most likely
causative organism?
• (A) primary viral pneumonia
• (B) an autoimmune reaction
• (C) Mycoplasma pneumoniae
• (D) Streptococcus pneumoniae
• (E) Neisseria catarrhalis
- This man has a complication of viral influenza; a
secondary bacterial pneumonia has developed. This
usually occurs 2–3 days after the initial viral symptoms
resolve. Primary viral pneumonia with influenza is not
common. S. pneumoniae, Staphylococcus, and H.
influenzae are the most common bacterial invaders in
pulmonary complications of influenza. Pneumonia is
the leading cause of death and may also be due to S.
pneumonia and H. influenzae. Mixed viral and bacterial
pneumonia is common; pure viral pneumonia in
influenza is uncommon (but can be very severe).
NEUMONIA
• Cual es el patógeno mas frecuente en la comunidad?
• 1. Streptococo pneumonie 2.Micoplasma pneumonie 3.Clamidia pneumoniae
• Agente etiológico de neumonía hospitalaria?
• 1.Pseudomona 2. Kliebsella 3.E.Coli
• Agentes etiológicos de neumonía adquirida en la comunidad con bronco aspiración?
• Px ALCOHÓLICOS y/ o PX QUE CONVULSIONAN y los agentes BACTEROIDES ORALIS, streptococo
• Agentes etiológicos para neumonía en px atípica?
• INFLUENZA H1N1 MICOPLASMA PNEUMONIE
• Caso clínico: paciente que estuvo en contacto con aire acondicionados o torre de enfriamento o nebulazadores que
durante un tiempo no funciono… desarrolla neumonía
• cual seria el agente causal?
• Legionella pneumonie
• Tx:
• ERITROMICINA o CLARITROMICINA si viene
NEUMONIA
• Enfermedad ocupacional que desarrollan personas que cuidan pájaros?
• PSITACOSIS agente causal: CHLAMYDIA PSITTACI
• Tx :
• TETRACICLINA (doxiciclina)
• Caso clínico de MICOPLASMA PNEUMONIE caract y tx?
• Es una neumonía atípica FENOMENO DE REYNAUD, ERITEMA MULTIFORME. Hemolisis, MIRINGITIS BULOSA
Rx: Pneumoniae en Parches
• Tx:
• ERITROMICINA (si vienen azitromicina o claritromicina contestar esas)
• Forma en que se realiza el dx:
• Prueba de fijación de complementos (la de mas especificidad y sensibilidad )ANTICUERPOS FRÍOS
• En que consiste el sx de austria?
• ENDOCARDITIS, NEUMONÍA y MENINGITIS…. Se presenta en px ALCOHÓLICOS
• Agente causal:
• STRPTOCOCO PNEUMONIE
• Cual es la imagen radiográfica de la neumonía adquirida en la comunidad: Broncograma aéreos dentro de la
consolidación pulmonar.
NEUMONIA
• Que neumonía se relaciona con neumatocele?
• Neumonía estafilococica (burbujas de aire en el parénquima pulmonar)?
• La neumonía adquirida en el hospital cual es su imagen radiológica?
• Patrón necrotizante, de focos múltiples
• Neumonía adquirida en comunidad por bronco aspiración con pérdida del
conocimiento que partes está afectada?
• Esta afectado el PULMÓN DERECHO, el LÓBULO MEDIO y el SEGMENTO
POSTERIOR
• Neumonia en un px con vih cual es su agente causal?
• Neumonía por PNEUMOCYSTIS JIROVECI
NEUMONIA
• Px con VIH con sospecha de neumonía por pneumocystis jiroveci que tinción se realiza para corraborar dx?
• R: metenamina argéntica por NITRATO DE PLATA
• Px con micoplasma el mejor estudio es?
• 1era pruebas de FIJACIÓN DE COMPLETOS
• 2do opción ANTICUERPOS FRÍOS
• Tratamiento de neumonía adquirida en la comunidad?
1) Si no tiene enfermedad concomitante:
2) MACROLIDO (azitromicina o claritromicina) tx de 1era elección
3) Con enfermedad concomitante:
4) FLUOROQUINOLONAS (levo, moxi, gatifloxacina)
• Tx de px con neumonía adquirida en la comunidad pero que por sus caract. Es internado?
• CEFALOSPORINA DE 3RA GENERACIÓN MAS MACROLIDO: ej cefotaxima + claritromicina
• Tx de neumonía por neumococo?
• FLUOROQUINOLONA LEVOFLOXACINO (NOponer PENICILINA… ojo)
NEUMONIA
• Px con neumonía adquirida en comunidad pero VIENE GRAVE que
debe ser INTERNADO EN TERAPIA intensiva?
• Fluoroquinolona + ceftriaxona o fluoroquinolona + cefotaxima
• Tx paciente con neumonía ADQUIRIDA INTRAHOSPITALARIA,
internado en terapia intensiva?
• Pseudomona, ecoli, kliebsella FLUOROQUINOLONA + CEFALOSPORINA
antipseudomonica (CEFTAZIDIMA, cefoperazona) o (carbepenem o iminpenem )
solo
• Neumonía hospitalaria por staphilococos aureus cual es el tx?
• VANCOMICINA (útil incluso para meticilino resistentes)
CASO CLINICO
- A 4-year-old boy is sent to the emergency room because of
clinical suspicion of meningitis. He has been ill for 2 days
with fever and lethargy. On examination, he is febrile, the
neck is stiff, and papilledema is present. There is no rash,
the lungs are clear, and heart sounds normal. Which of the
following is the most likely causative organism?
• (A) Neisseria meningitidis
• (B) Streptococcus pneumoniae
• (C) Haemophilus influenzae
• (D) Staphylococcus
• (E) Listeria species
- Since the introduction of H. influenzae type B
vaccine, S. pneumoniae has become the most
common type of meningitis in infants and
toddlers
TUBERCULOSIS
• Cuales son las manifestaciones extrapulmonares de la Tb?
• R = Meningitis, pericarditis, invasión ósea, gastrointestinal y urinaria
• Cual es la tinción especifica para Tb y reacción PPD positiva?
• R = Tinción de ZIEHL NEELSEN. En paciente inmunocomprometido ES + SI ES > 5 MM
• Como se muestra el LCR con Tb?
• R = CON PROTEÍNAS Y CELULARIDAD AUMENTADAS E HIPOGLUCORRAQUIA.
• Como se da el tratamiento para Tb?
• FASE INTENSIVA: Se dan 3 FÁRMACOS MAS ETAMBUTOL Y ESTREPTOMICINA hasta
completar 60 DOSIS
• FASE DE SOSTEN: ISONIAZIDA Y PIRAZINAMIDA 3 VECES A LA SEMANA DANDO 45 DOSIS.
• Como das quimioprofilaxis a contactos de pacientes con Tb?
• R = 6M EN CONTACTOS <5ª Y EN LOS CONTACTOS DE 5 – 14ª NO VACUNADOS CON
ISONIAZIDA SIN EXCEDER 300 MG.
• Como se le llama a la disminucion de la respuesta tuberculinica con el paso de la edad?
• R = Efecto Spink
TUBERCULOSIS
• Cual es el mecanismo de acción de la RIFAMPICINA?
• R = Es un BACTERICIDA que inhibe la subunidad beta de la RNA POLIMERASA,
IMPIDIENDO LA SÍNTESIS PROTEÍNICA.
• Cual es el mecanismo de acción de la ISONIACIDA?
• R = Es un ANÁLOGO DE LA NICOTINAMIDA que inhibe la SÍNTESIS DE ACIDO
MICOLICO y el metabolismo de los lípidos que CAUSA LA ROTURA DE LA PARED, el
cual DEBE ADMINISTRARSE CON PIRIDOXINA YA QUE PUEDE CAUSAR
NEUROPATÍA PERIFÉRICA. Se usa la Estreptomicina en caso de resistencia a
Isoniacida.
• Cual es el mecanismo de acción de la PIRAZINAMIDA?
• R = ANALOGO DE LA NICOTINAMIDA que actúa como BACTERICIDA en un medio
acido, lo que le permite actuar contra los bacilos intracelulares.
• Cual es el mecanismo de acción del ETAMBUTOL?
• R = Tiene actividad BACTERIOSTÁTICA. En menores de 12ª puede causar NEURITIS
RETROBULBAR.
TUBERCULOSIS
• Grupo de edad en que es mas frecuente la micobacteriosis ganglionar?
• Es mas frecuente en los niños (en segundo lugar la tb pulmonar) se divide en escrofulacia y la
tuberculosa
• Grupo de edad en que es mas frecuente la micobacteriosis pulmonar?
• En adultos: la primera causa es la tb pulmonar y la #2 la tb ganglionar
• Causa mas frecuente de Síndrome de piura estéril?
• 1era. Tuberculosis Renal
• 2da. Micoplasma Ominis
• Características de TB Renal?
1) Dolor lumbar, disuria, Febricula,
2) Urografía excretor: URETEROS ARROSARIADOS
• Cuando afecta ala columna vertebral se llama?
• Mal de Pott
• Cuanto tiempo se debe mantener en aislamiento a alguien que padece tuberculosis activa?
• R = 2-3 meses
TUBERCULOSIS
• Diagnostico diferencia de tb intestinal?
• Enfermedad de Cronh y cáncer intestinal
• Cual es la utilidad de la BCG?
• Proteger contra MENINGITIS TUBERCULOSA en los niños (su protección no dura
mas de 10 años)
• Características de una meningitis tuberculosa?
• Evolución subaguda, tiende a obstruir los agujeros de lushka y maggendi,
CARACTERÍSTICAMENTE EL 3-4-6 PAR CRANEAL
AFECTADO
• Cuales son las dos micobacteriosis, en los cuales en el esquema de tratamiento
se agrega corticoesteroides?
• Meningoencefalitis tuberculosa y pericarditis tuberculosa
• Causa mas frecuente de enfermedad de adison?
• 1era.AUTOINMUNIDAD 2da:TUBERCULOSIS (mas frecuente en países
subdesarrollados)
TUBERCULOSIS
• Cual es la causa mas frecuente de origen infeccioso de calcificación de suprarrenales?
• Enfermedad de Adison (astenia, adinamia, HIPERPIGMENTACION CUTANEA elevada,
NA BAJO, K ELEVADO, acidosis elevada, hipotensial arterial)
• Diagnostico definitivo de TB?
• Cultivo LOWENSTEIN-JENSTEIN-HOLTZ
• Que es el complejo de Ranke?
• Nódulos linfático hiliares o parahiliares afectados por tb + complejo primario de Gonh
• Cual es el sitio mas frecunte del complejo primario de Gonh?
• Lóbulo inferior del lóbulo derecho
• Como se interpreta Intradermoreaccion de Monteaux?
• Se lee a las 72 HRS DE HABERLO APLICADO.. SI TIENE MAS DE (10 MM) 1 CM DE
INDURACIÓN SE CONSIDERA POSITIVO
TUBERCULOSIS
• Si es un px con VIH, a cuanto se considera positiva?
• .5 cm (5MM)
• Que medicamentos constituye los 4 meses restantes de tx de px con tb?
• Isoniacida + rifampicina
• Que antifimico queda contraindicado en la mujer embarazada?
• ESTREPTOMICINA…. El tx es con ISO RIFA ETAMBUTOL + PIRIDOXINA POR 9 MESES (para
disminuir el efecto indeseable de la isoniacida –polineuropatia y hepatitis-)
• Cuales son las tuberculosis que se da tx por 9 meses?
• En la tb MILIAR, tb SNC, tb ARTICULAR u OSEA
• Los px con resistencia solo a isoniacida que tx se deberá dar? (Se llama RESISTENTE si lo es
a isoniacida)
1) RIFAMPICINA, PIRASINAMIDA Y ETAMBUTOL O RIFAMPICINA, PIRASINAMIDA
ESTREPTOMICINA POR 6 MESES
2) Rifampicina + etambutol por un año (se llama MULTIRESISTENTE si es a iso y
rifampicina)
INFECTOLOGÍA.pdf
INFECTOLOGÍA.pdf
INFECTOLOGÍA.pdf
INFECTOLOGÍA.pdf
INFECTOLOGÍA.pdf
INFECTOLOGÍA.pdf
INFECTOLOGÍA.pdf
INFECTOLOGÍA.pdf
INFECTOLOGÍA.pdf
INFECTOLOGÍA.pdf
INFECTOLOGÍA.pdf
INFECTOLOGÍA.pdf
INFECTOLOGÍA.pdf
INFECTOLOGÍA.pdf
INFECTOLOGÍA.pdf
INFECTOLOGÍA.pdf
INFECTOLOGÍA.pdf
INFECTOLOGÍA.pdf
INFECTOLOGÍA.pdf
INFECTOLOGÍA.pdf

Más contenido relacionado

Similar a INFECTOLOGÍA.pdf

Meningitis aguda, agente causales bacterianos y vorales
Meningitis aguda, agente causales bacterianos y voralesMeningitis aguda, agente causales bacterianos y vorales
Meningitis aguda, agente causales bacterianos y voralesErwinRiberaAez
 
22sepsisneonatal20091 090418232609-phpapp02
22sepsisneonatal20091 090418232609-phpapp0222sepsisneonatal20091 090418232609-phpapp02
22sepsisneonatal20091 090418232609-phpapp02Mi rincón de Medicina
 
Caso clinico neisseria meningitidis
Caso clinico neisseria meningitidisCaso clinico neisseria meningitidis
Caso clinico neisseria meningitidisAnaa Alvarezz
 
Caso clinico de Neisseria Meningitidis
Caso clinico de Neisseria Meningitidis Caso clinico de Neisseria Meningitidis
Caso clinico de Neisseria Meningitidis Anaa Alvarezz
 
Manejo neumonia adquirida en la comunidad
Manejo neumonia adquirida en la comunidadManejo neumonia adquirida en la comunidad
Manejo neumonia adquirida en la comunidadCentro de Salud El Coto
 
PRESENTACIÓN DE CASO CLINICO N.º 4.pptx
PRESENTACIÓN DE CASO CLINICO N.º 4.pptxPRESENTACIÓN DE CASO CLINICO N.º 4.pptx
PRESENTACIÓN DE CASO CLINICO N.º 4.pptxADELMORODRIGOSILVASA
 
Neumonia Adquirida en La Comunidad
Neumonia Adquirida en La ComunidadNeumonia Adquirida en La Comunidad
Neumonia Adquirida en La Comunidadsohorin
 
Neumonía adquirida en la comunidad 1
Neumonía adquirida en la comunidad 1Neumonía adquirida en la comunidad 1
Neumonía adquirida en la comunidad 1Diomedes Cerrud
 
meningitis encefaltis.pptx
meningitis encefaltis.pptxmeningitis encefaltis.pptx
meningitis encefaltis.pptxSamara Tenorio
 
22sepsisneonatal20091 090418232609-phpapp02
22sepsisneonatal20091 090418232609-phpapp0222sepsisneonatal20091 090418232609-phpapp02
22sepsisneonatal20091 090418232609-phpapp02Mi rincón de Medicina
 
Fiebre tifoidea medicina interna 2(seminario ppt)
Fiebre tifoidea medicina interna 2(seminario ppt)Fiebre tifoidea medicina interna 2(seminario ppt)
Fiebre tifoidea medicina interna 2(seminario ppt)wilmanBoluarteAyquip
 
Infecciones agudas del sistema nervioso centra lpptx
Infecciones agudas del sistema nervioso centra lpptxInfecciones agudas del sistema nervioso centra lpptx
Infecciones agudas del sistema nervioso centra lpptxmurgenciasudea
 
MENINGITIS -Dr Carlos Tousaint.pptx
MENINGITIS -Dr Carlos Tousaint.pptxMENINGITIS -Dr Carlos Tousaint.pptx
MENINGITIS -Dr Carlos Tousaint.pptxCarlosTousaint1
 

Similar a INFECTOLOGÍA.pdf (20)

Meningitis aguda, agente causales bacterianos y vorales
Meningitis aguda, agente causales bacterianos y voralesMeningitis aguda, agente causales bacterianos y vorales
Meningitis aguda, agente causales bacterianos y vorales
 
Enfermedades oportunistas
Enfermedades oportunistasEnfermedades oportunistas
Enfermedades oportunistas
 
22sepsisneonatal20091 090418232609-phpapp02
22sepsisneonatal20091 090418232609-phpapp0222sepsisneonatal20091 090418232609-phpapp02
22sepsisneonatal20091 090418232609-phpapp02
 
Caso clinico neisseria meningitidis
Caso clinico neisseria meningitidisCaso clinico neisseria meningitidis
Caso clinico neisseria meningitidis
 
Caso clinico de Neisseria Meningitidis
Caso clinico de Neisseria Meningitidis Caso clinico de Neisseria Meningitidis
Caso clinico de Neisseria Meningitidis
 
Manejo neumonia adquirida en la comunidad
Manejo neumonia adquirida en la comunidadManejo neumonia adquirida en la comunidad
Manejo neumonia adquirida en la comunidad
 
PRESENTACIÓN DE CASO CLINICO N.º 4.pptx
PRESENTACIÓN DE CASO CLINICO N.º 4.pptxPRESENTACIÓN DE CASO CLINICO N.º 4.pptx
PRESENTACIÓN DE CASO CLINICO N.º 4.pptx
 
Neumonia Adquirida en La Comunidad
Neumonia Adquirida en La ComunidadNeumonia Adquirida en La Comunidad
Neumonia Adquirida en La Comunidad
 
Meningoencefalitis
MeningoencefalitisMeningoencefalitis
Meningoencefalitis
 
MENINGITIS
MENINGITISMENINGITIS
MENINGITIS
 
Generalidades snc
Generalidades sncGeneralidades snc
Generalidades snc
 
Infección por vih
Infección por vihInfección por vih
Infección por vih
 
MENINGITIS_NEONATAL.pptx
MENINGITIS_NEONATAL.pptxMENINGITIS_NEONATAL.pptx
MENINGITIS_NEONATAL.pptx
 
Neumonía adquirida en la comunidad 1
Neumonía adquirida en la comunidad 1Neumonía adquirida en la comunidad 1
Neumonía adquirida en la comunidad 1
 
meningitis hsb.pptx
meningitis hsb.pptxmeningitis hsb.pptx
meningitis hsb.pptx
 
meningitis encefaltis.pptx
meningitis encefaltis.pptxmeningitis encefaltis.pptx
meningitis encefaltis.pptx
 
22sepsisneonatal20091 090418232609-phpapp02
22sepsisneonatal20091 090418232609-phpapp0222sepsisneonatal20091 090418232609-phpapp02
22sepsisneonatal20091 090418232609-phpapp02
 
Fiebre tifoidea medicina interna 2(seminario ppt)
Fiebre tifoidea medicina interna 2(seminario ppt)Fiebre tifoidea medicina interna 2(seminario ppt)
Fiebre tifoidea medicina interna 2(seminario ppt)
 
Infecciones agudas del sistema nervioso centra lpptx
Infecciones agudas del sistema nervioso centra lpptxInfecciones agudas del sistema nervioso centra lpptx
Infecciones agudas del sistema nervioso centra lpptx
 
MENINGITIS -Dr Carlos Tousaint.pptx
MENINGITIS -Dr Carlos Tousaint.pptxMENINGITIS -Dr Carlos Tousaint.pptx
MENINGITIS -Dr Carlos Tousaint.pptx
 

Más de Patricia Abdulazis Elneser (9)

valentinaboteroparra.2002_Parte7(1).pdf
valentinaboteroparra.2002_Parte7(1).pdfvalentinaboteroparra.2002_Parte7(1).pdf
valentinaboteroparra.2002_Parte7(1).pdf
 
Ecologia_de_poblaciones.pdf
Ecologia_de_poblaciones.pdfEcologia_de_poblaciones.pdf
Ecologia_de_poblaciones.pdf
 
Indicador...ppt
Indicador...pptIndicador...ppt
Indicador...ppt
 
Microorganism+Maintenance[1].pdf
Microorganism+Maintenance[1].pdfMicroorganism+Maintenance[1].pdf
Microorganism+Maintenance[1].pdf
 
cultivos de referencia.pdf
cultivos de referencia.pdfcultivos de referencia.pdf
cultivos de referencia.pdf
 
Unidad corta. cambio_climatico__1_
Unidad corta. cambio_climatico__1_Unidad corta. cambio_climatico__1_
Unidad corta. cambio_climatico__1_
 
Ahorro agua
Ahorro aguaAhorro agua
Ahorro agua
 
Evaporaci n de_nube_de_conflicto
Evaporaci n de_nube_de_conflictoEvaporaci n de_nube_de_conflicto
Evaporaci n de_nube_de_conflicto
 
Clasificacion w. de breack
Clasificacion w. de breackClasificacion w. de breack
Clasificacion w. de breack
 

Último

NERVIO OLFATORIO. PARES CRANEALES. SISTEMA NERVIOSO
NERVIO OLFATORIO. PARES CRANEALES. SISTEMA NERVIOSONERVIO OLFATORIO. PARES CRANEALES. SISTEMA NERVIOSO
NERVIO OLFATORIO. PARES CRANEALES. SISTEMA NERVIOSOEPICRISISHQN1
 
Clase 13 Artrologia Cintura Escapular 2024.pdf
Clase 13 Artrologia Cintura Escapular 2024.pdfClase 13 Artrologia Cintura Escapular 2024.pdf
Clase 13 Artrologia Cintura Escapular 2024.pdfgarrotamara01
 
(2024-04-17) PATOLOGIAVASCULARENEXTREMIDADINFERIOR (ppt).pdf
(2024-04-17) PATOLOGIAVASCULARENEXTREMIDADINFERIOR (ppt).pdf(2024-04-17) PATOLOGIAVASCULARENEXTREMIDADINFERIOR (ppt).pdf
(2024-04-17) PATOLOGIAVASCULARENEXTREMIDADINFERIOR (ppt).pdfUDMAFyC SECTOR ZARAGOZA II
 
biomagnetismo.ppt para medicina complementaria
biomagnetismo.ppt para medicina complementariabiomagnetismo.ppt para medicina complementaria
biomagnetismo.ppt para medicina complementariairina11171
 
atencion del recien nacido CUIDADOS INMEDIATOS.ppt
atencion del recien nacido CUIDADOS INMEDIATOS.pptatencion del recien nacido CUIDADOS INMEDIATOS.ppt
atencion del recien nacido CUIDADOS INMEDIATOS.pptrosi339302
 
SISTEMA OBLIGATORIO GARANTIA DE LA CALIDAD EN SALUD SOGCS.pdf
SISTEMA OBLIGATORIO GARANTIA DE LA CALIDAD EN SALUD SOGCS.pdfSISTEMA OBLIGATORIO GARANTIA DE LA CALIDAD EN SALUD SOGCS.pdf
SISTEMA OBLIGATORIO GARANTIA DE LA CALIDAD EN SALUD SOGCS.pdfTruGaCshirley
 
Torax normal-Oscar 2024- principios físicos del rx de torax
Torax normal-Oscar 2024- principios físicos del rx de toraxTorax normal-Oscar 2024- principios físicos del rx de torax
Torax normal-Oscar 2024- principios físicos del rx de toraxWillianEduardoMascar
 
Aparato digestivo (irrigación, internación, anatomía)
Aparato digestivo (irrigación, internación, anatomía)Aparato digestivo (irrigación, internación, anatomía)
Aparato digestivo (irrigación, internación, anatomía)Majo472137
 
EMBARAZO MULTIPLE, su definicioón y cuidados de enfermería
EMBARAZO MULTIPLE, su definicioón y cuidados de enfermeríaEMBARAZO MULTIPLE, su definicioón y cuidados de enfermería
EMBARAZO MULTIPLE, su definicioón y cuidados de enfermeríaCaRlosSerrAno799168
 
mapa-conceptual-del-sistema-endocrino-4-2.pptx
mapa-conceptual-del-sistema-endocrino-4-2.pptxmapa-conceptual-del-sistema-endocrino-4-2.pptx
mapa-conceptual-del-sistema-endocrino-4-2.pptxDanielPedrozaHernand
 
Psicología: Revista sobre las bases de la conducta humana.pdf
Psicología: Revista sobre las bases de la conducta humana.pdfPsicología: Revista sobre las bases de la conducta humana.pdf
Psicología: Revista sobre las bases de la conducta humana.pdfdelvallepadrob
 
Hiperleucocitosis y leucostasis medicina interna
Hiperleucocitosis y leucostasis medicina internaHiperleucocitosis y leucostasis medicina interna
Hiperleucocitosis y leucostasis medicina internafercont
 
PROCESO DE EXTRACCION: MACERACION DE PLANTAS.pptx
PROCESO DE EXTRACCION: MACERACION DE PLANTAS.pptxPROCESO DE EXTRACCION: MACERACION DE PLANTAS.pptx
PROCESO DE EXTRACCION: MACERACION DE PLANTAS.pptxJOSEANGELVILLALONGAG
 
urgencia y emergencia. Diferencias y ejemplos
urgencia y emergencia. Diferencias y ejemplosurgencia y emergencia. Diferencias y ejemplos
urgencia y emergencia. Diferencias y ejemploscosentinojorgea
 
Cartilla de Prestadores de Prevencion Salud
Cartilla de Prestadores de Prevencion SaludCartilla de Prestadores de Prevencion Salud
Cartilla de Prestadores de Prevencion Saludfedesebastianibk1
 
redox y pilas temario 2 bachillerato ebau
redox y pilas temario 2 bachillerato ebauredox y pilas temario 2 bachillerato ebau
redox y pilas temario 2 bachillerato ebauAnaDomnguezMorales
 
anatomia de la PELVIS EN GENERAL anatomia.pptx
anatomia de la PELVIS EN GENERAL anatomia.pptxanatomia de la PELVIS EN GENERAL anatomia.pptx
anatomia de la PELVIS EN GENERAL anatomia.pptxJuanGabrielSanchezSa1
 
la CELULA. caracteristicas, funciones, i
la CELULA. caracteristicas, funciones, ila CELULA. caracteristicas, funciones, i
la CELULA. caracteristicas, funciones, iBACAURBINAErwinarnol
 

Último (20)

NERVIO OLFATORIO. PARES CRANEALES. SISTEMA NERVIOSO
NERVIO OLFATORIO. PARES CRANEALES. SISTEMA NERVIOSONERVIO OLFATORIO. PARES CRANEALES. SISTEMA NERVIOSO
NERVIO OLFATORIO. PARES CRANEALES. SISTEMA NERVIOSO
 
Clase 13 Artrologia Cintura Escapular 2024.pdf
Clase 13 Artrologia Cintura Escapular 2024.pdfClase 13 Artrologia Cintura Escapular 2024.pdf
Clase 13 Artrologia Cintura Escapular 2024.pdf
 
(2024-04-17) PATOLOGIAVASCULARENEXTREMIDADINFERIOR (ppt).pdf
(2024-04-17) PATOLOGIAVASCULARENEXTREMIDADINFERIOR (ppt).pdf(2024-04-17) PATOLOGIAVASCULARENEXTREMIDADINFERIOR (ppt).pdf
(2024-04-17) PATOLOGIAVASCULARENEXTREMIDADINFERIOR (ppt).pdf
 
biomagnetismo.ppt para medicina complementaria
biomagnetismo.ppt para medicina complementariabiomagnetismo.ppt para medicina complementaria
biomagnetismo.ppt para medicina complementaria
 
atencion del recien nacido CUIDADOS INMEDIATOS.ppt
atencion del recien nacido CUIDADOS INMEDIATOS.pptatencion del recien nacido CUIDADOS INMEDIATOS.ppt
atencion del recien nacido CUIDADOS INMEDIATOS.ppt
 
SISTEMA OBLIGATORIO GARANTIA DE LA CALIDAD EN SALUD SOGCS.pdf
SISTEMA OBLIGATORIO GARANTIA DE LA CALIDAD EN SALUD SOGCS.pdfSISTEMA OBLIGATORIO GARANTIA DE LA CALIDAD EN SALUD SOGCS.pdf
SISTEMA OBLIGATORIO GARANTIA DE LA CALIDAD EN SALUD SOGCS.pdf
 
Torax normal-Oscar 2024- principios físicos del rx de torax
Torax normal-Oscar 2024- principios físicos del rx de toraxTorax normal-Oscar 2024- principios físicos del rx de torax
Torax normal-Oscar 2024- principios físicos del rx de torax
 
Situaciones difíciles. La familia reconstituida
Situaciones difíciles. La familia reconstituidaSituaciones difíciles. La familia reconstituida
Situaciones difíciles. La familia reconstituida
 
Aparato digestivo (irrigación, internación, anatomía)
Aparato digestivo (irrigación, internación, anatomía)Aparato digestivo (irrigación, internación, anatomía)
Aparato digestivo (irrigación, internación, anatomía)
 
EMBARAZO MULTIPLE, su definicioón y cuidados de enfermería
EMBARAZO MULTIPLE, su definicioón y cuidados de enfermeríaEMBARAZO MULTIPLE, su definicioón y cuidados de enfermería
EMBARAZO MULTIPLE, su definicioón y cuidados de enfermería
 
(2024-04-17) ULCERADEMARTORELL (ppt).pdf
(2024-04-17) ULCERADEMARTORELL (ppt).pdf(2024-04-17) ULCERADEMARTORELL (ppt).pdf
(2024-04-17) ULCERADEMARTORELL (ppt).pdf
 
mapa-conceptual-del-sistema-endocrino-4-2.pptx
mapa-conceptual-del-sistema-endocrino-4-2.pptxmapa-conceptual-del-sistema-endocrino-4-2.pptx
mapa-conceptual-del-sistema-endocrino-4-2.pptx
 
Psicología: Revista sobre las bases de la conducta humana.pdf
Psicología: Revista sobre las bases de la conducta humana.pdfPsicología: Revista sobre las bases de la conducta humana.pdf
Psicología: Revista sobre las bases de la conducta humana.pdf
 
Hiperleucocitosis y leucostasis medicina interna
Hiperleucocitosis y leucostasis medicina internaHiperleucocitosis y leucostasis medicina interna
Hiperleucocitosis y leucostasis medicina interna
 
PROCESO DE EXTRACCION: MACERACION DE PLANTAS.pptx
PROCESO DE EXTRACCION: MACERACION DE PLANTAS.pptxPROCESO DE EXTRACCION: MACERACION DE PLANTAS.pptx
PROCESO DE EXTRACCION: MACERACION DE PLANTAS.pptx
 
urgencia y emergencia. Diferencias y ejemplos
urgencia y emergencia. Diferencias y ejemplosurgencia y emergencia. Diferencias y ejemplos
urgencia y emergencia. Diferencias y ejemplos
 
Cartilla de Prestadores de Prevencion Salud
Cartilla de Prestadores de Prevencion SaludCartilla de Prestadores de Prevencion Salud
Cartilla de Prestadores de Prevencion Salud
 
redox y pilas temario 2 bachillerato ebau
redox y pilas temario 2 bachillerato ebauredox y pilas temario 2 bachillerato ebau
redox y pilas temario 2 bachillerato ebau
 
anatomia de la PELVIS EN GENERAL anatomia.pptx
anatomia de la PELVIS EN GENERAL anatomia.pptxanatomia de la PELVIS EN GENERAL anatomia.pptx
anatomia de la PELVIS EN GENERAL anatomia.pptx
 
la CELULA. caracteristicas, funciones, i
la CELULA. caracteristicas, funciones, ila CELULA. caracteristicas, funciones, i
la CELULA. caracteristicas, funciones, i
 

INFECTOLOGÍA.pdf

  • 2.
  • 3. MENINGITIS • Cual es la presión de apertura a la toma de LCR en meningitis bacteriana? • R = Normal <180 o elevada > 180, siendo común 200-500 • Cuales son los datos del LCR de la meningitis bacteriana aguda? I. Presión de abertura: Normal o > 180 siendo común observar valores entre 2 – 500 II. Apariencia: Leucocitos y bacterias turban el LCR III. Glucorraquia: Concentración < 40 (NORMAL > 45) IV. Proteinorraquia: Valores arriba de 50 (NORMAL 40) • Cual es el tratamiento de elección en los contactos de pacientes con enfermedad meningocococica? • R = RIFAMPICINA • Que constituye el síndrome de austrian? • R = MENINGITIS, ENDOCARDITIS y NEUMONÍA por estreptococo pneumoniae en PACIENTE ALCOHÓLICOS
  • 4. MENINGITIS • Tratamiento de elección para meningitis por NEUMOCOCO? • R = CEFTRIAXONA • Tratamiento de elección para meningitis por S. AUREUS METICILINO SENSIBLE? • R = Nafcilina, DICLOXACILINA • Tratamiento de elección para meningitis por S. AUREUS METICILINO RESISTENTE? • R = VANCOMICINA o TEICOPLANINA • Tratamiento de elección para meningitis por LISTERIA? • R = AMPICILINA • Tratamiento de elección para meningitis por H. INFLUENZA? • R = CEFTRIAXONA o cefotaxima • Tratamiento de elección para meningitis por ANAEROBIOS? • R = METRONIDAZOL • Cual es el tratamiento de elección en la meningitis CRIPTOCOCOCICA? • R = ANFOTERICINA B + 5-fluocitosina
  • 5. MENINGITIS • Que medida se debe tomar en los contactos de un paciente con meningitis por Neisseria Meningitidis? • R = RIFAMPICINA oral a contactos cercanos o CIPROFLOXACINO. • - Two students from a university dormitory building have contracted meningitis due to Neisseria meningitides. Which of the following students in the dormitory are most likely to benefit from chemoprophylaxis? • (A) everybody in the dormitory, with oral amoxicillin • (B) close contacts only, with oral amoxicillin • (C) everybody in the dormitory, with oral rifampin • (D) close contacts only, with oral rifampin • (E) everybody in the dormitory, with meningococcal vaccine
  • 6. - Although only close contacts need chemoprophylaxis, it is sometimes given more widely than recommended because of community concern. Meningococcal vaccine is effective against serotype A and C, and will prevent late secondary infection in close contacts. Ciprofloxacin or ofloxacin are alternatives to rifampin
  • 7. MENINGITIS • Característica de meningoencefalitis subaguda? • Meningoencefelatis por tuberculosis • Agente etiológicos en meningitis en recién nacidos? • E.COLI (en el trabajo de parto) y estreptococos del grupo B (AGALACTIAE) • Agente etiológico de meningitis en adultos? • 1.Streptococo pneumoniae 2.Neisseria Meningitidis 3.Streptococo del grupo B • CASO CLÍNICO: Recién nacido, con fontanela abombadas, petequias en la cara anterior del tórax, esta en cunero y alado entro por deshidratación. Actualmente con fiebre, rigidez de nuca, Kerney y Brusinski, punción lumbar con diplococos GRAM NEGATIVOS en LCR • Agente etiológico?: • Meningitis por Neisseria Meningitidis (MENINGOCOCO) • Tx: • PGSC a dosis altas, al personal involucrado: cipro o rifampicina • Medida implantada por la OMS para detección rápida de MENINGITIDIS? • DETERMINACIÓN DE ANTÍGENOS CAPSULARES: Streptococo pneumone, Neisseria Meningitidis, Hemophilus Influenza se detecta en 3 horas –aunque no los solicites-
  • 8. MENINGITIS • Meningitis POR LISTERIA (BACILO GRAM +) esta relacionado con: • LECHE O DERIVADOS LACTEOS, CARNES MAL COCIDAS, INMUNODEPRIMIDOS y px de la tercera edad…. Tx :AMPICILINA como primera opción • Con respecto del Streptococo Pneumoniae caract de infección?: • Se da principalmente en pacientes de los 2 a los 20 años, px ALCOHÓLICOS, con OTITIS PREVIA. • Meningitis por Bacilos GRAM NEGATIVOS caract de infección? • Se presenta en diabéticos, cirróticos, alcohólicos px con IVU, px con cáncer y/o inmunodeprimidos. El bacilo GRAM POSITIVO y entérico mas importante la listeria monocitogena, otro bacilo que no es entérico es la hemophilus Influenza (disminución en su incidencia actualmente) • Agente etiológico de la Meningitis en un px que previamente se le realizo procedimiento neuroquirurgico? • STAPHILOCOCOS AUREUS • Citocinas proinflamatorias que producen daño a nivel SNC, en la barrera hematoencefalica, plexos coroides? • IL 1 Y FNT que aumentan en las primeras 2 horas
  • 9. MENINGITIS • Cantidad normal de proteínas en liquido cefalorraquídeos? • Normal: 40 nos pueden poner 45 • La meningitis es el prototipo de los tres tipos de edema cerebral, cuales son?: 1) EDEMA VASOGENICO (FNT: aumenta la permeabilidad dela Barrera hematopencefalica), 2) EDEMA INTERSTICIAL (a nivel del espacio subaracnoideo hay Exudado, leucocitos producen obstrucción del flujo LCR produciendo este tipo de edema), 3) EDEMA CITOTOXICO (degranulación de los neutrófilos libera metabolitos tóxicos) • Como obtenemos la presión perfusión intracerebral? • Presión arterial media menos la presión intracerebral… una disminución de una presión arterial media puede provocar isquemia e infarto cerebral (p/ej sepsis) • Cuadro clínico de px con meningitis bacteriana? 1) FIEBRE , CEFALEA Y RIGIDEZ DE NUCA (hasta en el 90% de los casos) leucocitosis, alteración del edo. De la conciencia. 2) Nausea , vomito, si tiene RASH EN LA CARA ANTERIOR DEL TORAX (casi seguro MENINGOCOCCEMIA). Datos de hipertensión intracraneal (papiledema, pupilas hiporreactivas, postura de descerebración, tiene reflejo de cushing)
  • 10. MENINGITIS • Laboratorio de LCR? • PUNCIÓN LUMBAR: 1.-PRESION NORMAL DEL LCR 180mm de Agua, 2.-POLIMORFONUCLEARES: menos de 5 polimorfos 3.-GLUCOSA: es el 50% dela glucosa central del px entre 45 y 55 4.- PROTEINAS: 40mg/dl 5.-AGUA DE ROCA 6.-CLORUROS: 90 a 110 en LCR • En quienes se realiza el cultivo en TINTA CHINA? • CRIPTOCOCO NEOFORMANS EN PX CON VIH • Caso clinico • Px con rigidez de nuca, brusinski +, irritado Cefalea, fiebre, sin traumatismo craneoencefálico, ANTECEDENTE DE NADAR EN UN MANANTIAL LANZÁNDOSE CLAVADOS, a la punción lumbar: aspecto turbio, presión:200mmhg glucosa:20mg/dl proteínas:85 mg/dl Tinción de GRAM fue NEGATIVA, el cultivo se realizo, que tipo de meningitis tiene este px: • a.-M aseptica b.-M purulenta c.-M Tuberculosa (Agente etiológico: NAEGLERIA o gardenelas son amibas de vida silvestre) muy agresivas, con pronostico mortal, • Tx?: • ANFOTERICINA B • La meningitis Viral caract del LCR: • CELULARIDAD: NORMAL PROTEÍNAS: ligeramente AUMENTADAS ASPECTO: AGUA DE ROCA GLUCOSA: NORMAL PRESIÓN: nl o ligeramente NORMAL CLORUROS: NORMALES
  • 11. MENINGITIS • CASO CLÍNICO: Px con fiebre intensa (40), ingresa al servicio de urg, con trastorno del edo. De conciencia, con 2 crisis convulsivas.. de ocupación CUIDADOR DE CABALLOS (antecedente de haber enterrado a su caballo y después fue atacado por aves de rapiña) • LCR: Glucosa: 100 liquido transparente, presión: 185mm de agua, celularidad: linfocitos cloruros normales • Cual es la impresión dx: • MENINGOENCEFALITIS VIRAL EQUINA VENEZOLANA • CASO CLÍNICO: Niña Indígena procedente de AREA RURAL, se presenta al servicio de urg. Con 3 semanas de evolución con fiebre persistente, vomito, trastorno del estado de conciencia, ala exploración física presenta papiledema ligero, con afectación del 3er, 4to, 6to par craneal, con rigidez de nuca. LCR: ASPECTO XANTOCROMICO, Presión: 220mm de agua Glucosa: 30 Proteínas: de 75mg/dl , Celularidad por linfocitos CLORUROS: 70 Meningitis tuberculosa • Tx de meningitis en Recién Nacidos? • AMPICILINA (e.coli)- CEFOTAXIMA (streptococo)
  • 12. MENINGITIS • Tx de meningitis en niños de 1 a 3 meses? • AMPI + CEFO o CEFTRIAXONA + (valorar administración de DEXAMETASONA) para evitar VENTRICULITIS • Tx de meningits en px de 3 meses y menos de 50 años? • CEFOTAXIMA + VANCOMICINA o CEFTRIAXONA + VANCOMICINA (cubrir cocos gram +, Y bacilos gram -) • Tx de meningitis en un px mas de 50 años con antecedente de alcoholismo o tiene DM • AMPICILINA + VANCOMICINA + CEFALOSPORINA (tx p cocos gram + bacilos gram -) • Px neutropenico, o px con meningitis con TRAUMATISMO CRANEOENCEFALICO, o que le hicieron un procedimiento neuroquirurgico que tx se le dara? • Cubrir Gram - (ceftazidil), cubrir Gram + meticilino resistente (Vancomicina) • px que en el cultivo presenta meningococo que tx se da? • PENICILINA G si es resistente: CEFTRIAXONA O CEFOTAXIMA y si es alérgico VANCOMICINA (2 gr x dia) • Px que le hicieron punción lumbar y obtuvieron en el LCR un cultivo Gram – que tx: • Pseudomona aeroginosa: Ceftazidine(6gr x dia) Kliebsella, e coli: CEFOTAXIMA O CEFTRIAZONA (4gr x dia) • Tx para: Listeria: AMPICILINA(12 gr al dia) H. Influenza: CEFTRIAXONA o cefotaxima Anaerobios: METRONIDAZOL (2 grx dia)
  • 13.
  • 14.
  • 15.
  • 16.
  • 17. FOD • Cual es la definición de FOD? • R = Elevación de la temperatura por arriba de 38 GRADOS en varias determinaciones durante mas de 3 SEMANAS sin llegar al diagnostico DESPUÉS DE 1 SEMANA de estudio hospitalario • Cual es la definición de FOD en paciente hospitalizados? • R = Paciente hospitalizado 24 HRS CON FIEBRE y 2 CULTIVOS NEGATIVOS después DE 2 DÍAS. • Define FOD en paciente inmunodeficiente? • R = FIEBRE >38.3 en paciente con NEUTROPENIA < 500 durante > 3 DÍAS y cultivos negativos después DE 2 DÍAS. • Define FOD asociado a VIH? • R = Fiebre por encima de 38.3 GRADOS y que permanece sin diagnostico por MAS DE 3 SEMANAS • Cuales son las principales causas de FOD? 1. Infección por TB y CMV 2. Neoplasia por linfoma o leucemia 3. Idiopática si después de 6m de estudio no hay causa
  • 18. FOD • Que posibilidades diagnosticas te sugieren los estudios de BH en FOD? 1. Neutrofilia: Infecciones bacterianas 2. Linfocitosis: Virus, micobacterias, Brucella, Salmonella, Rickettsia y Leishmania 3. Eosinofilia: Neoplasias, parasitosis y vasculitis. 4. Leucopenia: Neoplasia hematológica o inmunosupresión. 5. > VSG: Sepsis, polimialgia reumática y AR. • Cual es el pronostico del FOD? • R = Bueno aunque no se identifique la causa
  • 19. FOD • Cual es el mecanismo de acción por el cual las citocinas generan fiebre? • Se liberan por los monocitos llegan por la sangre al SNC a nivel del hipotálamo y estimulan la síntesis de PROSTAGLANDINAS E2 Y LA PROTAGLANDINA S2 ALFA que a su vez estimula el centro termorregulador para producir fiebre… • Mecanismo por el cual los corticoides disminuyen la fiebre? • Son inmunosupresores inhiben la fagocitosis entonces no hay liberación de citocinas y x lo tanto no hay fiebre • Que diferencia hay entre hipertermia y fiebre? • En la hipertermia hay un daño a nivel hipotalámico (gralmente fármacos: haloperidol, anestésicos inhalados p ej) y provocar una reacción idiosincrática… en la hipertermia hay temperaturas de 40-41 grados y no esta mediada por citocinas • Efecto perjudicial de la fiebre en el embarazo? • En los primeros meses puede provocar anencefalia..
  • 20. FOD • Mecanismo por el cual el paciente de larga evolución padece de anemia? • Las CITOCINAS PROINFLAMATORIAS compite por la TRANSFERRINA • Características del síndrome febril agudo? • Paciente con fiebre menos de dos semanas de evolución, o fiebre cuantificada con sintomatología inespecífica, por lo general debida principalmente a enf. Biliares son auto limitadas • Caract de la fiebre de origen a determinar? • Fiebre mayor de 3 semanas que no cede, con fiebre mayor de 38 grados • Etiología de la fiebre de origen indeterminado en un px con vih? • Pneumocistis jirovecci, Micobacterium avium Intracelular, Linfomas • Cáncer mas frecuente en los niños? • Leucemia • Y en los adultos jóvenes? • Linfoma • Características del síndrome de horner? • Paciente con cáncer de pulmón, con ptosis y miosis, facies hipocráticas, EL TUMOR SE ENCUENTRA DEL LADO DE LA CARA DONDE NO ESTA SUDANDO.
  • 21. SEPSIS • Como se diagnostica el síndrome de respuesta inflamatoria sistémica? • R = Cuando se presentan 2 o mas de los siguientes parámetros: 1. FIEBRE corporal > 38 o hipotermia < 36 2. TAQUIPNEA con FR > 24 x minuto 3. TAQUICARDIA > 90 x minuto 4. LEUCOCITOSIS > 12,000 o leucopenia < 4,000 o 10% de bandas • Como diagnosticas sepsis grave? • R = Sepsis asociada a disfunción de un órgano con los siguientes datos: 1. Cardiovascular: TAS < 90 mm Hg que responde a fluidoterapia 2. Respiratoria: Relación FiO2/PaO2 < 200 3. Renal: Diuresis < 0.5 ML/KG/HR durante 1 hr a pesar de fluidoterapia 4. Trombocitopenia: < 80,000 o su reducción a la mitad con respecto a la obtenida 72 hrs antes 5. Acidosis metabólica: Con PH < 7.30 6. Choque séptico: Hipotensión con TAS < 90 durante 1 hr y que no responde a fluidoterapia necesitando vasopresores
  • 22. SEPSIS • En caso de bacteriemia o sepsis cuales son algunas características distintivas de los agentes patológicos? A. Neisseria meningitidis: Se acompaña de PURPURA O PETEQUIAS B. Rikettsia: Cuando aparecen LESIONES PETEQUIALES por mordida de garrapata en zona endémica C. Pseudomona aureoginosa: ECTIMA GANGRENOSO D. S. Aureus o S. Pyogenes: ERITRODERMIA GENERALIZADA. • Cual es la complicación mas frecuente de la bacteriemia? • R = SIRPA caracterizado por HIPOXEMIA E INFILTRADOS PULMONARES DIFUSOS.
  • 23.
  • 24. FASCITIS NECROSANTE • Cual es la secuencia de eventos para que sea manifiesta la fascitis necrosante? • R = Dolor – Fiebre – aumento de volumen local con eritema e hiperestesia – epidermis se indura adquiriendo un color rojo/café donde se forman vesículas - piel se necrosa y desprende • Cuales son las variables de presentación de la fascitis necrosante? A. GANGRENA DE FOURNIER: Afecta escroto, pene, perineo con la probable extensión a muslos y abdomen B. GANGRENA SINERGICA DE MELENEY: Da como resultado por la combinación de S. Aureus + anaerobios. • Cual es el agente causal de la fascitis necrosante? • R = ESTREPTOCOCO GRUPO A
  • 25.
  • 26.
  • 27. ABSCESO CEREBRAL • Cual es la causa mas común de absceso cerebral? • R = INFECCIONES CRÓNICAS de los oídos y senos paranasales, en pacientes trasplantados es debido a hongos- Aspergillus • Cual es el cuadro clínico del absceso cerebral? • R = Similar al de un tumor endocraneal mas síndrome febril • Localización mas frecuente del absceso cerebral? • R = Frontal • Cual es el manejo del absceso cerebral? • R = CRANEOTOMÍA, PUNCIÓN O ASPIRACIÓN + PENICILINA G + METRONIDAZOL considerando que el agente puede ser estreptococo, neumococo o anaerobios mixtos.
  • 28. ENCEFALITIS • Cual es el agente etiológico de la encefalitis de San Luis? • R = Mosquitos en EUA • Cual es el agente etiológico de la encefalitis equina oriental? • R = Afecta a caballos y a veces a humanos, se desconoce agente • Cual es el agente etiológico de la encefalitis de La Crosse? • R = ARDILLAS en el bosque. • Cual es el agente etiológico de la encefalitis del Nilo Occidental? • R = Esta se manifiesta en dichas regiones epidemiológicas. • Cual es el manejo de las encefalitis virales? • R = SOPORTE, no dar esteroides.
  • 29. NEUROCISTICERCOSIS • Cual es el agente infeccioso en neurocisticercosis? • R = Tenia SOLIUM • Cual es el cc de la neurocisticercosis? 1) Forma activa: Sin evidencia del parasito con hallazgos en la TAC de calcificaciones o hidrocefalia 2) Parenquimatosa: Larvas en corteza y ganglios basales 3) Forma subaracnoidea: Con fibrosis leptomeningea provocando neuropatías por atrapamiento o hidrocefalia • Cual es el método diagnostico de elección para le neurocisticercosis? • R = TAC de elección e IRM en casos dudosos. • Cual es el manejo de la neurocisticercosis? • R = ALBENDAZOL 400 mg c/12 1 SEMANA, PRAZICUANTEL X 15 DÍAS. SE RECOMIENDA EL USO DE PREDNISONA 1 DÍA ANTES DEL FÁRMACO Y CONTINUAR CON DISMINUCIÓN DE LA DOSIS 14 DÍAS DESPUÉS.
  • 30. TETANOS • Cual es el cuadro clínico del tétanos? • R = Comienza con ESPASMOS LEVES EN LA MANDÍBULA (trismo), el cuello y la cara. La RIGIDEZ se desarrolla rápidamente en el TÓRAX, ESPALDA, MÚSCULOS ABDOMINALES y en ocasiones la laringe interfiriendo con la respiración. Los espasmos musculares son contracciones súbitas, fuertes y dolorosas. • Cual es el tratamiento de elección para tétanos? • R = PENICILINA SÓDICA CRISTALINICA 20, 000 000. Se debe administrar globulina inmunitaria antitetánica 5000 U/IM y una vez recuperado el paciente se da esquema completo.
  • 31. CASO CLINICO • -A 22-year-old recent immigrant to the United States has never been vaccinated for tetanus. He sustains a minor, but soil- contaminated, injury. Which of the following statements is correct? • (A) tetanus usually develops within 2 weeks following exposure • (B) tetanus always develops within 4 hours following exposure in patients who have not been previously immunized • (C) tetanus may develop many months or years following exposure in susceptible individuals • (D) the usual incubation period for tetanus is 48 hours • (E) tetanus may be prevented with penicillin
  • 32. -In tetanus, an acute onset is usual. The median onset is 7 days, and 90% present within 14 days of injury. The organism is an anaerobic, motile Gram-positive rod. It has the ability to survive for years in the form of spores, which are resistant to disinfectants and heat. Tetanus can occur in nonimmunized individuals, or those who have neglected their booster shots. Penicillin, or metronidazole, is used in treatment, but their efficacy is not clear
  • 33. CARBUNO O ANTRAX • Cual es el agente etiológico del Carbunco o ANTRAX? • R = BACILLUS ANTHRACIS, bacilo GRAM + • La forma cutánea de carbunco o ántrax que es la mas común como se manifiesta? • R = Desarrollo de PÁPULA EN ZONA DE INOCULACIÓN, RODEADA DE VESÍCULAS, LA ZONA CENTRAL SE ULCERA Y SECA DESARROLLÁNDOSE ZONA DEPRIMIDA DE COLOR NEGRO, NO DOLOROSO. Se resuelve espontáneamente pero 20% puede ser fatal. • Como se manifiesta la forma intestinal de carbunco o ántrax? • R = FIEBRE, DISNEA, CIANOSIS, desorientación y signos de septicemia. Evoluciona rápidamente a choque, coma y muerte. • Como se manifiesta el carbunco o ántrax respiratorio y quienes lo PADECEN FRECEUNTEMENTE? • R = Se manifiesta en personas que MANIPULAN PIEL Y LANAS. Se produce FIEBRE CON TOS NO PRODUCTIVA y mal estar general, con buena evolución a 2-3 días y a continuación SÚBITAMENTE PRESENTA DIFICULTAD RESPIRATORIA GRAVE, CIANOSIS Y SEPTICEMIA FATAL CON MUERTE EN <24 HRS. • Como realizas el diagnostico de carbunco o ántrax? • R = Identificación del bacilo con TINCIÓN GRAM, PCR. • Cual es el manejo de carbunco o ántrax? • R = CIPROFLOXACINO, levofloxacino o PENICILINA G. • Como previenes el carbunco o ántrax? • R = PROTECCIÓN DE PIEL, MUCOSAS Y LA VACUNACIÓN
  • 34.
  • 35. SALMONELOSIS • Cuantos antígenos tiene la salmonella typhi? • R = 3: H FLAGELAR, K CAPSULAR y O SOMÁTICO • Cuales son los sitios frecuentes de infección secundaria por salmonella typhi? • R = Hígado, bazo, la medula ósea, PLACAS DE PEYER DEL ÍLEON TERMINAL y la vesícula biliar. • Cual es el cuadro clínico característico de salmonella typhi? • R = ROSÉOLA TIFOIDICA que se caracteriza por MACULAS ERITEMATOSAS DE 2-4 MM QUE BLANQUEAN A LA PRESIÓN, localizadas en la parte superior del abdomen y tórax anterior y que por lo general dura de 2-3 días. • Cual es la complicación mas frecuente de la salmonella typhi? • R = La mas común es la HEMORRAGIA GI que resulta de la erosión de un vaso de la pared intestinal secundaria a la necrosis de las placas de peyer en el íleon terminal • Cual es el medio diagnostico mas sensible en la primer semana? • R = HEMOCULTIVO, después el mielocultivo. • Semana en la que se presentan las complicaciones de fiebre tifoidea? • R = Fines de SEGUNDA Y TERCERA SEMANA • Con que inoculo de S. Tiphy se desarrolla la enfermedad? • R = 100,000
  • 36. SALMONELOSIS • A que semana aparece la roséola tifoidica, la cual aparece a la digito presión? • R = 2da semana • Como dx a un portador crónico de salmonella? • R = COPROCULTIVOS + DURANTE 1ª • En que semana los px se perforan o tienen hemorragias con choque hipovolemico? • R = 3ERA semana • En que semanas en la fiebre tifoidea se realizan los cultivos específicos? • R = 1era HEMOCULTIVO, 2da MIELOCULTIVO y 3era COPROCULTIVO/urocultivo • Cual es el cuadro clínico de la salmonelosis? • R = Fiebre elevada 2-3 semanas, dolor abdominal, DIARREA EN SOPA DE CHICHARO, ROSÉOLA, DELIRIO, ESPLENOMEGALIA. • Cual es la prueba de laboratorio confirmatoria de fiebre tifoidea? • R = 1:640, LEUCOPENIA y cuadro clínico característico
  • 37. SALMONELOSIS • Cual es el sitio donde suele albergarse la salmonella? • R = Vesícula biliar • Que se presenta en la primera semana de la salmonelosis? • R = FIEBRE, HIPERSENSIBILIDAD ABDOMINAL, TIFLITIS (CUANDO HAY DOLOR EN FOSA ILIACA DERECHA POR INVASIÓN DE LA PLACA DE PEYER Y CREPITA AL TACTO) • Que se presenta en la segunda semana en la fiebre tifoidea? • R = DIARREA, estreñimiento, delirio, EXANTEMA EN CARA ANTERIOR DE TÓRAX QUE SEDE A LA DIGITO PRESIÓN. El sistema retículo-endotelial se hipertrofia con HEPATO/ESPLENOMEGALIA, hiperplasia de las PLACAS DE PEYER. Bazo e hígado desarrollan nódulos tifoideos. • Cuales son las complicaciones habituales de la fiebre tifoidea en la 3era semana? • R = HEMORRAGIA Y PERFORACIÓN INTESTINAL, peritonitis con placa simple de abd con aire libre. Orquitis, meningitis, nefritis. • Cual es el tratamiento de la salmonella multiresistente o en lugares o zonas endémicas? • R = CIPROFLOXACINO, levofloxacino, ceftriaxona
  • 38. SALMONELOSIS • Cual es el mejor método diagnostico en un paciente que se automedico en fiebre tifoidea? • R = MIELOCULTIVO por que el medicamento no penetra a esa zona • Tratamiento para fiebre tifoidea? • R = CLORAMFENICOL 50/mg/kg por 2 SEMANAS • Tratamiento para anemia + fiebre tifoidea? • R = AMOXICILINA o ampicilina, NO DARLE CLORAMFENICOL por ke causa aplasia medular. • Tx fiebre tifoidea en embarazada? • R = AMOXICILINA o ampicilina
  • 39. SALMONELOSIS • Tx fiebre tifoidea en niño con resistencia? • R = QUINOLONAS NO, cefotaxima o CEFTRIAXONA • En que casos se utiliza la dexametasona en fiebre tifoidea? • R = En el ESTADO TIFOIDICO CON SHOQUE • Que se le da al portador de salmonella? • R = CIPROFLOXACINA por 3 MESES • Que tx das a un px portador crónico que no respondió a ciprofloxacino? • R = COLECISTECTOMÍA, por que ahí vive la salmonella.
  • 40.
  • 41. BRUCELOSIS • Cuales son los principales vectores de la brucelosis y a quienes afecta comúnmente? • R= ORDEÑADORES, rastros, CARNICERO, veterinarios son vectores. DERIVADOS DE LA LECHE como el queso o tejidos del animal afectado. • Cuales son las cepas de brucella que produce enf en humano/ FIEBRE ONDULANTE? • R = MELITERSIS DE CABRA + frecuente a nivel mundial, SUIZ DEL CERDO +, ABORTUS DE GANADO VOVINO +++. • Microbiologicamente que es la brucella? • R = COCOBACILO GRAM -, crece a 37 grados, es inmóvil a pesar de TENER FLAGELO. • Cual es la prueba de laboratorio para brucelosis? • R = 2- MERCAPTOETANOL, es una inmunoglubulina IgG Vs BRUCELLA
  • 42. BRUCELOSIS • Cual es el medio de cultivo de elección para brucelosis? • R = RUIZ CASTAÑEDA medio doble y ROSA DE BENGALA (fines epidemiológicos) • Que tratamiento utilizas de primera elección contra brucella? • R = Combinado DOXICICLINA + GENTAMICINA de 3-6 semanas. • Que tx utilizas en brucelosis en hueso y SNC? • R = DOXICICLINA Y RIFAMPICINA • Cual es el tratamiento de brucelosis en embarazada? • R = TMP/SMZ+ RIFAMPICINA + ACIDO FÓLICO • Cual es el tratamiento de niños menores de 12ª con brucelosis? • R = TMP/SMZ + RIFAMPICINA • Cual es el tratamiento en pacientes alérgicos a sulfas en brucelosis? • R = RIFAMPICINA Y CEFTRIAXONA.
  • 43.
  • 44. BOTULISMO • Cual es el cuadro clínico del CLOSTRIDIUM BOTULINUM transmitido por alimentos? • R = Los síntomas comienzan en 6 HRS DESPUÉS A 2 SEMANAS, se manifiesta con DIPLOPÍA, VISIÓN BORROSA, PTOSIS, disfagia, sensación de sequedad de mucosa oral, DEBILIDAD MUSCULAR que afecta únicamente a los hombros, miembros torácicos y mas tarde a los pélvicos. • Cual es el manejo? • R = Administración de TOXINA BOTULÍNICA con PREVENCIÓN DE INSUFICIENCIA RESPIRATORIA que amerite ventilación mecánica
  • 45. CASO CLINICO • - An 18-year-old woman has eaten homemade preserves. Eighteen hours later, she develops diplopia, dysarthria, and dysphagia. Which of the following is the most likely causative organism? • (A) Clostridium botulinum toxin • (B) staphylococcal toxin • (C) salmonellosis • (D) brucellosis • (E) shigellosis
  • 46. PESTE BUBONICA • Cual es el agente causal por la peste bubónica? • R = YERSINIA PESTIS • Cual es el cuadro clínico de la peste bubónica? • R = Es la forma mas frecuente, se transmite por la PICADURA DE PULGA. Inicio con FIEBRE ALTA, nauseas, MIALGIAS y ADENOPATÍAS DOLOROSAS REGIONALES, el PACIENTE EVOLUCIONA A ESTUPOR, COMA Y MUERTE. • Que formas clínicas tiene la peste? • R = Peste bubónica, septicémica y neumónica • Como diagnosticas peste? • R = BIOPSIA DE GANGLIOS o EXUDADO FARÍNGEO • Cual es el tratamiento de la peste? • R = ESTREPTOMICINA, CLORANFENICOL O TETRACICLINAS. • Cual es la prevención de la peste? • R = Hay una vacuna con CEPA INACTIVADA F1
  • 47. NAEGLERIA • Cual es el agente causal de la enfermedad transmitida en los manantiales? • R = Naegleria • Cuale es el tratamiento de naegleria? • R = Anfotericina B • Cual es el pronostico de naegleria? • R = Muerte
  • 48. RABIA • Cual es el agente causal de la rabia? • R = Virus de la familia Rabdovirus • Cual es el cuadro clínico de la rabia? • R = Dolor en el sitio de la mordedura, fiebre, malestar general, nausea y vomito. Diez días mas tarde hay manifestaciones del SNC pudiendo ser encefálica o paralitica y las 2 formas evolucionan a coma - muerte. A. ENCEFALICA: Se caracteriza por delirio, somnolencia, hidrofobia (espasmos laríngeos dolorosos al beber agua) B. PARALITICA: Da una parálisis ascendente. • Cual es el manejo de la rabia? I. Es necesario observar al animal los siguientes 10 DÍAS tomando UNA BIOPSIA DEL CUERO CABELLUDO y analizarla con Ac fluorescente o PCR. II. Atención de la LESIÓN LAVANDO CON AGUA Y JABÓN ABUNDANTE A CHORRO DURANTE 10 MINUTOS, para MUCOSA ORAL O NASAL CON SOLUCIÓN FISIOLÓGICA DURANTE 5 MINUTOS. Desinfectar la herida con agua oxigenada o tintura de yodo. III. Si se requiere SUTURAR LA HERIDA se debe APLICAR PRIMERO INMUNOGLOBULINA ANTIRRÁBICA HUMANA y se aproximan los bordes o se dan puntos temporales en caso que la herida sea profunda, IV. En exposición leve se administra vacunación antirrábica en la región deltoidea los días 0-3- 7-14 Y 28. V. Se APLICARA INMUNOGLOBULINA INTRALESIONAL LA PRIMER ½ Y LA OTRA MITAD VÍA IM. VI. Se usara SUERO HETEROLOGO EN CASO EXTREMO DE NO CONTAR CON INMUNOGLOBULINA.
  • 49. ADENOVIRUS • Donde se replica el adenovirus? • R = Faringe, conjuntivas, intestino delgado, ganglios linfáticos cervicales, preauriculares o mesentéricos. • Cual es el cuadro clínico de adenovirus? • R = CONJUNTIVITIS aguda, QUERATOCONJUNTIVITIS, FIEBRE CONJUNTIVAL, FARINGITIS LARINGOTRAQUEITIS (CRUP), bronquiolitis, neumonía, gastroenteritis, linfadenitis mesentérica y CISTITIS HEMORRÁGICA.
  • 51.
  • 52. INFLUENZA • Cuales son las proteínas que distinguen al virus de la influenza? • R = H: HEMAGLUTININA y N: NEURAMINIDASA • La influenza aviar que nomenclatura viral la representa? • R = H5N1 y es altamente mortal
  • 53. DENGUE Y FIEBRE AMARILLA • Cual es el mosquito transmisor del dengue? • R = AEDES AEPYPTI. • Cual es el cc del dengue? 1) El DENGUE CLÁSICO se inicia con FIEBRE, BRADICARDIA, pulso lento, EXANTEMA MACULAR PUNTIFORME, tos, ardor faríngeo, MIALGIAS, ARTRALGIAS. 2) El DENGUE HEMORRÁGICO se caracteriza por aumento en la permeabilidad vascular con clasificación del I-IV. • Que es lo que distingue a la fiebre amarilla del dengue laboratorialmente? • R = CUERPOS DE INCLUSIÓN VIRAL llamados CUERPOS DE COUNCILMAN EN LA SANGRE. • Cual es el cuadro clínico de la fiebre amarilla? • R = FIEBRE, cefalea, mialgias, DOLOR LUMBOSACRO, ERITEMA OCULAR Y FACIAL, PETEQUIAS EN ENCIAS Y MUCOSA NASAL, HEMATEMESIS, MELENA, ALBUMINURIA, encefalitis o meningoencefalitis. • Cual es el manejo de la fiebre amarilla y del dengue? • R = SINTOMÁTICO
  • 54.
  • 55. VIH • Que células ataca el VIH? • R = CD4 • Cual es el ciclo vital del VIH? • R = A través de la proteína gp41, se realiza la fusión, posteriormente la RNA del VIH se descubre e interna en la célula afectada, la enzima transcriptasa inversa del virion cataliza la transcripción inversa del RNA en DNA, este se transfiere hacia el núcleo en el cual se integra en los cromosomas por medio de la integrasa. Los macrófagos actúan como reservorio del VIH y lo diseminan a otros sistemas • Que se debe realizar como prevención en una persona que halla sido puncionada de manera accidental con una aguja utilizada previamente en un paciente con VIH? • R = INHIBIDORES NUCLEOSIDOS DE LA TRANSCRIPTASA REVERSA • Como se manifiesta el síndrome retroviral agudo SRA en VIH? • R = SÍNTOMAS SIMILARES AL RESFRIADO O MONONUCLEOSIS, fiebre, escalofríos, sudores nocturnos y erupciones en la piel que dura de 1-3 SEMANAS. • De acuerdo a la CDC como se realiza dx de SIDA? • R = Recuento <200/MM DE CD4 + la presencia de ENFERMEDAD OPORTUNISTA
  • 56. VIH • Como dx VIH? • R = ELISA y confirmatorio con WESTERN BLOOD • Cuales son parte del grupo de fármacos que inhiben la TRANSCRIPTASA INVERSA en VIH? • R = ZIDOVUDINA, didanosina, zalcitabina. • Cuales son las indicaciones actuales para uso de tratamiento retro vírico en VIH? 1. Síndrome de infección aguda 2. INFECCIÓN CRÓNICA: Enfermedad sintomática o asintomática con T CD4 <350/NL o RNA DEL VIH CON > 50, 000 COPIAS 3. PROFILAXIS POST- EXPOSICIÓN: Se recomienda utilizar combinaciones como ZIDOVUDINA + LAMIVUDINA + NEVERIPINA • Cuales son las indicaciones para cambiar el tratamiento antirretrovirico? 1. Disminución <1 log del RNA del VIH con plasma a las 4 semanas de haber iniciado el tratamiento 2. Incremento DEL TRIPLE O MAS DEL VALOR PLASMÁTICO DEL RNA NO ATRIBUIBLE A INFECCIÓN concomitantes o vacuolas. 3. DISMINUCIÓN DEL PORCENTAJE DE CÉLULAS TCD4 4. Deterioro clínico 5. Efectos adversos
  • 57. VIH • Como se manifiesta la neumonía por Pneumocystis Jiroveci? • R = CD4 < 200/MM mas AUMENTO DE LA DHL, el diagnostico definitivo se obtiene por la TINCIÓN DE ESPUTO CON WRIGHT-GIEMSA • Cual es la causa mas frecuente de retinitis en pacientes con VIH? • R = CITOMEGALOVIRUS • Mecanismo por el que el VIH penetra a la célula? • R = Pinocitosis • Cual es la neoplasia maligna mas común en pacientes con VIH? • R = Sarcoma de Kaposi • Cual es el manejo del Sarcoma de Kaposi? • R = Para la FORMA CUTÁNEA SE DA DOXORRUBICINA, para el INTESTINAL O VISCERAL SE DA DOXORRUBICINA, BLEOMICINA Y VINBLASTINA. • Como se maneja la profilaxis en pacientes con VIH de acuerdo al conteo de las células CD4? 1) CD4 < 200: VS P. JIROVECI con TMP/SMZ 1Tab/24 hrs C/3 DIA 2) CD4 75-100: VS M. AVIUM con CLARITROMICINA y se SUSPENDE CUANDO AUMENTAN LOS CD4 3) CD4 < 50: VS CITOMEGALOVIRUS con GANCICLOVIR 4) PPD con induración > 5 MM iniciar profilaxis VS M. TUBERCULOSIS con ISONIACIDA a 300 mg al día + PIRIDOXINA durante 9-12 MESES.
  • 58. VIH • Tratamiento de primera elección para la retinitis por CITOMEGALOVIRUS? • GANCICLOVIR • Tratamiento para el herpes tipo 2 en pacientes con VIH? • ACICLOVIR • De que manera se puede inhibir el síndrome de desgaste? • TALIDOMIDA • Agente infeccioso de la lengua vellosa? • Virus de EPSTEIN BARR ( relacionada con virus de mononucleosis y linfomas) VIRUS TIPO 4 • Sitio mas común de lesión dermatológica, “SARCOMA DE KAPOSI” manchas color de piel? • El sitio de afectación mas frecuente es la CARA EN PARPADOS Y PUENTE NASAL RELACIONADO CON EL VIRUS TIPO 8 • Tratamiento de toxoplasmosis? • PIREMITAMINA o sulfas + CLINDAMICINA en dosis altas
  • 59. VIH • Posibilidad de contagio por coito anal receptivo en el VIH? • 1:100 1:30 • Zona donde se alojan el virus de la inmunodeficiencia es alta y útil para seguimiento del tx de VIH? • Biopsia de mucosa rectal otra opción es PCR • Transmisión perinatal del VIH?: • 13% al 40% • Coito con inserción anal: 1: 1,000 • Transfusión: 1: 100,000 • Coito inserción vaginal: 1: 10,000 varia si la mujer se encuentra en periodo menstrual o si tiene una ETS • Aguja infectada?: • 1-300 • Compartir agujas?: • 1-150 (drogadictos) • A que se refiere la regla de los 3 con relación a aguja infectada? • Hepatitis B : 30% Hepatitis C: 3% HIV: .3%
  • 60. VIH • Diagnostico para valorar el tratamiento “respuesta terapéutica” • RCP se realiza al mes y medio y a los 3 meses y posteriormente CADA 6 MESES (no se usa para diagnostico) • Que pasa con una persona que tiene VIH y las siguientes enfermedades Concomitantes? • Hepatitis B: Indiferente (no pasa nada) • Hepatitis C: Agresividad • HEPATITIS G: FRENA LA PROGRESIÓN DEL VIH • Con respecto a la estatificación de etapas A-B-C y grados I al III? • A: asintomáticos B: Síntomas Constitucionales (fiebre, perdida de peso, diarrea) C: Infecciones Oportunistas (neumocistis jirovecci, linfoma del SNC, Criptosporidiasis, Toxoplasma del SNC, Micovacterium Avium intra celular) • I.- 500 copias II.-MAS DE 200 Y MENOS DE 500 III.-MENOS DE 200 copias • INHIBIDORES DE TRANSCRIPTASA INVERSA -no análogo de los Nucleosidos- NNRTI • De la ZIDOVUDINA AZT cual es el efecto indeseable? PROBLEMAS MEDULARES, anemia severa, trombocitopenia, leucopenia • DE LA EFEVIRENZ cual es el efecto indeseable? • Insomnio y pesadillas (DEPRESIÓN) • INHIBIDORES DE PROTEASAS • Del Indinavir efecto indeseable mas frecuente: FORMADOR DE CÁLCULOS cólico renoureteral • Ritonavir + lopinavir : Es el caletra efecto sinérgico
  • 61. VIH • Profilaxis (exposición en trabajadores de la salud)(exposición SEXUAL) que maniobras o procedimientos? • Tiene una posibilidad de infección de 1:300 SE REALIZA UN ELISA con fines legales, ( AL MES)… si se considera EXPOSICIÓN DE BAJO RIESGO dar AZT+3TC X UN MES se REALIZA ELISA SI ES NEGATIVO SE SUSP… si es exposición de alto riesgo AZT+3TC +UN TERCER MEDICAMENTO…. NO DAR TX A LAS 72 POSTEXPOSICION • Copias de CD4? • Entre 50 y 100 copias en el momento del dx: MVAIC, CMV, Linfomas del SNC • MENOS DE 200: Criptocococis, toxoplasmosis, NEUMOCISTIS JIROVECCI • Entre 200 y 500: LENGUA VELLOSA, Sarcoma de Kaposi, TB Pulmonar, Infección por virus herpes, candidiosis • Px con HIV y tiene exposición con px con tb? • Profilaxis: PPD MAS DE 5MM ISONIACIDA + RIFAMPICINA + PIRAZINAMIDA POR 12 MESES + PIRIDOXINA • En que consiste el síndrome retroviral agudo? 1) Paciente con infección de VIH con SÍNTOMAS A CORTO PLAZO (de 2 A 3 SEMANAS y sucede en el 40%de los casos) Y CONSISTE EN FIEBRE ADENOPATÍA, RASH CUTÁNEO COLOR SALMON Y HEPATOMEGALIA… CUADRO MUY PARECIDO AL DE LA MONONUCLEOSIS INFECCIOSA, 2) DX ELISA A LAS 6 SEMANAS Y 3 MESES si sale positivo se realiza el WESTERN BLOT PARA CONFIRMARLO
  • 62. VIH • Combinaciones aprobadas: I. Tenofovir (TDF)+ emtricitabina (FTC) y efavirenz (EFV) II. Atazanavir (ATV) + ritonavir (rtv)+tenofovir (TDF) / emtricitabina (FTC) (si no funciona el primer esquema) III. Caletra +tenofovir (TDF) / emtricitabina (FTC) (El caletra se puede cambiar con AZT + 3TC) • Efectos colaterales? I. Zidovudina….. ANEMIA por hipoplasia medular II. Nerfinavir…. DIARREA III. Didanosina…. PANCREATITIS IV. Zalcitabina….. NEUROPATÍA PERIFÉRICA V. Estaduvidina…. NEUROPATÍA PERIFÉRICA (lamivudina +zidovudina) 3TC….. Neuropatia periférica VI. Abacavir….. FIEBRE RASH, HIPERSENSIBILIDAD FATAL VII. Tenofovir…. TOXICIDAD RENAL VIII. Saquinavir…. LÍPIDOS ELEVADOS • Px con embarazo + VIH Cual es el tx? I. ZIDOVUDINA (2do y 3er trimestre)… se continua en el trabajo de parto (para evitar la infección vertical) II. POSPARTO (NO LACTANCIA 10-20 tiene posibilidad de infectar) 2da opción LAMIVUDINA
  • 63. VIH • Cuales son parte del grupo de fármacos INHIBIDORES DE PROTEASA en VIH? • R = SaquinAVIR, ritonAVIR, indinAVIR. • Cuales son parte del grupo de fármacos INHIBIDORES DE LA FUSIÓN? • R = Enfubitida, que entre sus efectos adversos se encuentra la hipersensibilidad y neumonía bacteriana • Cuales son parte del grupo de fármacos que inhiben la TRANSCRIPTASA INVERSA en VIH? • R = ZIDOVUDINA, didanosINA, zalcitabINA.
  • 64.
  • 65.
  • 66.
  • 67.
  • 68.
  • 69.
  • 70.
  • 71. CASO CLINICO -A 19-year-old man has donated blood for the first time. Despite having no risk factors for human immunodeficiency virus (HIV) infection, his blood tests positive for HIV by enzyme immunoassay (EIA). Which of the following statements is correct? • (A) EIA is currently the most specific test for HIV • (B) he might have a false-positive secondary to an unsuspected collagen-vascular disease • (C) he has a 75% chance of truly being infected with HIV • (D) EIA is an excellent screening test • (E) a Western blot test would be more sensitive
  • 72. -EIA is an excellent screening test for HIV infection as it is positive in over 99.5% of cases. However, it lacks specificity, and in low risk populations, only about 10% of EIA positive results are true positives. Recent influenza vaccination, acute viral infections, and liver disease are common causes for false positives. The Western blot test is more specific and is the usual confirmatory test, although even more specific tests are now available.
  • 73. ASPERGILOSIS • Como se adquiere la aspergilosis? • R = Por inhalación de ESPORAS DEL HONGO, las cuales se encuentran en HOJAS SECAS, GRANEROS DE MAIZ, ESTIÉRCOL Y VEGETALES EN DESCOMPOSICIÓN. • En los pacientes con rinitis crónica por aspergilus, que datos de laboratorio encuentras en el moco? • R = Rico en EOSINOFILOS y cristales de CHARCOT-LEYDEN • En que consiste la aspergilosis pulmonar endobronquial saprofita o aspergiloma? • R = En el CRECIMIENTO DEL HONGO dentro de las cavidades pulmonares, SECUNDARIAS GENERALMENTE A TB, SARCOIDOSIS, HISTOPLASMOSIS O BRONQUIECTASIAS. • Como dx aspergilus? • R = CULTIVO DE SABOURAD, detección de ANTICUERPOS, galactomanano en suero (chbts en su pared celular) y biopsia • Cual es el tratamiento de aspergilus? • R = Esteroides, ANFOTERICINA B, LOBECTOMÍA EN CASO DE ASPERGILOMA o debridacion de tejido infectado. • Cual es el tratamiento de elección en caso de toxicidad secundaria al DESOXICOLATO de ANFOTERICINA B en caso de aspergilosis invasiva? • R = VORICONAZOL 6mg/kg/dia.
  • 74.
  • 75. BLASTOMICOSIS • Cual es el agente causal de blastomicosis? • R = BLASTOMYCES DERMATITIDIS • Cuales son las manifestaciones clínicas de blastomicosis? • R = PULMONAR Y CUTÁNEA. NEUMONÍA aguda que no responde al tratamiento y CURA ESPONTÁNEAMENTE. Las LESIONES CUTÁNEAS predominan en las ZONAS EXPUESTAS, característicamente son PAPULAS, NÓDULOS O PLACAS BIEN DELIMITADAS, únicas o múltiples, NO DOLOROSAS NI PRURIGINOSAS, que evolucionan a lesiones verrugosas, costras o ulceras. • Como diagnosticas blastomicosis? • R = De ELECCIÓN CULTIVO pero tarda mucho, se pueden identificar organismos con la TINCIÓN DE PLATA, METENAMINA O ACIDO PERIODICO DE SCHIFF en la biopsia de los tejidos infectados o en la citología de esputo. Donde se observan CÉLULAS LEVADURIFORMES CON YEMAS DE BASE AMPLIA. • Cual es el tratamiento de blastomicosis? • R = En la forma LEVE - ITRACONAZOL, en la forma GRAVE - ANFOTERICINA B
  • 76. CANDIDOSIS • Como se manifiesta la CANDIDIASIS DIGESTIVA y como se trata? • R = Se ASOCIA a lesiones ORALES Y PERIANALES, produce DIARREA CON MOCO, el diagnostico se realiza con el EXAMEN DIRECTO DE LAS HECES CON KOH y se trata con NISTATINA ORAL. • Como se manifiesta la CANDIDIOSIS URINARIA y como se trata? • R = Se presenta como CISTITIS, pielonefritis, BOLAS FÚNGICAS RENALES, ABSCESOS RENALES o necrosis de las papilas. Se trata con fluconazol o ANFOTERICINA B. • Como se diagnostica CANDIDIOSIS PULMONAR y como se trata? • R = Cuando realizas HEMOCULTIVOS POSITIVOS o ANTÍGENO MANAN DEL SUERO. El tratamiento se realiza con ANFOTERICINA B, en los pacientes que no la toleran se utiliza FLUCONAZOL. • Como se manifiesta la ENDOCARDITIS por candidiosis? • R = FIEBRE persistente, SOPLOS CARDIACOS y ESPLENOMEGALIA, en algunos casos hay embolización a grandes arterias, como la iliaca o femoral. LA VÁLVULA MAS AFECTADA ES LA MITRAL.
  • 77. CANDIDOSIS • Como diagnosticas ENDOCARDITIS por cándida? • R = Ecocardiagrafia, hemocultivos seriados y detección de ANTÍGENO MANAN en suero? • Cual es el tratamiento de endocarditis por cándida? • R = RESECCIÓN DE VÁLVULA afectada y ANFOTERICINA B • Como se presenta la candidiasis diseminada aguda o sepsis? • R = Se presenta en PACIENTES NEUTROPENICOS, CON FIEBRE, LEUCOCITOSIS, TROMBOCITOPENIA, inestabilidad hemodinámica y choque.
  • 78. CRIPTOCOCOCIS (AVES, PALOMAS) • Cual es el agente causal de la criptococosis y donde se alberga? • R = CRYPTOCOCCUS NEOFORMANS, se encuentra en HECES DE PALOMAS (CUIDADORES DE AVES) O NIDOS • Como diagnosticas criptococosis? • ANTÍGENO MANAN en suero o biopsia. • En LCR con TINCIÓN DE TINTA CHINA • Como se trata la criptococosis? • R = ANFOTERICINA B y 5-fluorocitosina
  • 79. HISTOPLASMOSIS (CAVERNAS) • Donde se alberga el Histoplasma capsulatum? • R = En forma de moho en suelo RICO EN NITRÓGENO, como el de los GALLINEROS, o donde duermen las aves y CAVERNAS DE MURCIÉLAGOS • Como diagnosticas histoplasmosis? • R = El método de elección es el CULTIVO, pero tarda mucho y haces frotis de los fluidos o tejidos infectados, las muestras para cultivo en HISTOPLASMOSIS DISEMINADA SE OBTIENEN DE MEDULA ÓSEA, HÍGADO, GANGLIOS LINFÁTICOS O LESIONES MUCOCUTANEAS. • Cual es el manejo de la hispoplasmosis? • R = Anfotericina B
  • 80. MUCORMICOSIS • Cual es el agente etiológico de la mucormicosis? • R = MUCOR, ABSIDIA, RIZOPUS denominado ficomicetos. • Como se manifiesta la infección de los senos paranasales en mucormicosis? • R = Predominantemente a PACIENTES CON DM mal tratada, con FIEBRE, congestión nasal, SECRECIÓN NASAL SANGUINOLENTA, DOLOR SINUSAL Y DIPLOPÍA, a la exploración física se observa DISMINUCIÓN DE LOS MOVIMIENTOS OCULARES, quemosis y PROPTOSIS y SI INVADE EL SENO FRONTAL PRODUCE COMA. • Como se manifiesta la mucormicosis gastrointestinal? • R = Se disemina al peritoneo produciendo la muerte en 70 días • Como se diagnostica mucormicosis? • R = FROTIS EN FRESCO donde se ven HIFAS HIALINAS o en BIOPSIAS donde se observa NECROSIS ISQUÉMICA o hemorrágica • Cual es el manejo de la mucormicosis? • R = El tratamiento consiste en altas dosis de ANFOTERICINA B, ASEO QUIRÚRGICO DE LOS TEJIDOS.
  • 81.
  • 82. COCCIDIOIDOMICOSIS • Cuales son los signos y síntomas de la coccidioidomicosis? • R = TOS, RINORREA, febricula, dolores musculares y en la RADIOGRAFÍA DE TÓRAX CON CAVITACIÓN. • - A 43-year-old man developed a cough shortly after returning from a 1- month hiking trip in California. While there, he was hiking in the central California valleys. During his trip, he had developed a “flu-like” illness consisting of fever, cough, and muscle pains, which resolved spontaneously. A CXR shows a thin-walled cavity in the right upper lobe, and the sputum reveals fungal elements. Which of the following is the most likely causative organism? • (A) ringworm • (B) Cryptococcus neoformans • (C) Candida albicans • (D) mycobacteria • (E) coccidioidomycosis
  • 83. - Coccidioidomycosis is the usual cause of pulmonary cavitation resulting from fungal infection. A rarefaction may be demonstrable in a pneumonic lesion within 10 days of onset
  • 84. PALUDISMO • Cual es el agente causal de paludismo o malaria? • R = PARASITOSIS intracelular eritrocitaria por PLASMODIUM, que se transmite por el MOSQUITO ANOPHELES • Cuantas especies de plasmodium existen? • R = 4, P. falciparum, P. vivax, P. ovale y P. malariae • Cual es el ciclo biológico del plasmodium? • R = Los mosquitos se infectan al ingerir sangre infectada con micro y macrigametocitos. Dentro del mosquito, se multiplica sexualmente en su intestino, produciendo los ooquistes, y al madurar se rompen liberando ESPOROZOITOS, que migran a las glándulas salivales del mosquito y son la forma infectante en los seres humanos. Ya en el TORRENTE SANGUÍNEO viajan a hígado donde se reproducen y forman los MEROZOITOS que tienen la capacidad de infectar los eritrocitos. • Que produce la lisis eritrocitaria al contraer paludismo? • R = La PLASMEPSINA, que es una proteasa del acido aspartico que DEGRADA LA HEMOGLOBINA.
  • 85. PALUDISMO • Cual es el tipo de genero en paludismo mas frecuente? • R = P. FALCIPARUM, es la mas frecuente y de peor pronostico con fiebre > 40 • Como se diagnostica el paludismo? • R = OBSERVACIÓN DE PARASITO EN FROTIS SANGUÍNEO O GOTA GRUESA. La TINCIÓN DE GIEMSA permite observar las granulaciones de Schuffner. • Como se trata el paludismo? • R = QUININA Y CLOROQUINA. El SULFATO DE QUININA se usa en RESISTENCIA A CLOROQUINA
  • 86.
  • 87. BORRELIOSIS O ENFERMEDAD DE LYME • Cual es el agente causal de la enfermedad de Lyme? • R = Espiroqueta BORRELIA BURGDORFERI GRAM (-) de color violeta con tinción Giemsa que se CULTIVA EN MEDIO NOGUCHI. Se adquiere por MORDEDURA DE GARRAPATA del genero IXODES. • Cual es el cuadro clínico de la enfermedad de Lyme? • R = Se divide en 3 etapas I. ERITEMA CRÓNICO O MIGRANS, pápulas eritematosas que se EXTIENDEN DE FORMA CENTRIFUGA DEJANDO LA PARTE CENTRAL DEL CUERPO SANA, acompañada de síntomas generales como astenia, adinamia, fiebre, mialgias y cefalea. II. Alteraciones neurológicas, cardiacas y reumáticas III. POLIARTRITIS MIGRATORIA, alteraciones dermatológicas y neurológicas • Como se diagnostica la enfermedad de Lyme? • R = ELISA o PCR
  • 88. BORRELIOSIS O ENFERMEDAD DE LYME • Tratamiento de elección para enf de Lyme? • R = DOXICICLINA por mes y medio • Cual es el tratamiento para borreliosis en su etapa precoz? • R = DOXICICLINA. • Cual es el tratamiento para borreliosis en su etapa tardía? • R = CEFTRIAXONA
  • 89. LEPTOSPIROSIS • Cual es el vector de la leptospirosis, que enzimas produce y que nutrientes necesita? • R = Leptospira interrogans en ORINA DE RATA. PRODUCE CATALASA Y HIALURODINASA y requieren TIAMINA B1 Y VITAMINA B12 PARA SU DESARROLLO • Cuales son las manifestaciones clínicas de la Leptospirosis? • R = De inicio súbito, con FIEBRE, derrame conjuntival, epistaxis, MIALGIAS, cefalea intensa, EXANTEMA Y HEPATOMEGALIA. • Que es el síndrome de Weil en Leptospirosis? • R = Al cursar la primera fase, viene una ASINTOMÁTICA para posteriormente PRESENTARSE LEPTOSPIROSIS ANICTERICA. • Como se manifiesta el síndrome de Weil? • R = CON FIEBRE, CEFALEA, MIALGIAS, EXANTEMA, miocarditis, ICTERICIA, insuficiencia hepática, INSUFICIENCIA RENAL, FENÓMENOS HEMORRÁGICOS, LEPTOSPIURIA y daño al SNC (MENINGITIS ASEPTICA). • Cual es la triada clásica del síndrome de Weil? • R = Daño en HÍGADO, RIÑÓN y SNC. • Cual es el tratamiento de la Leptospirosis? • R = PENICILINA, estreptomicina o tetraciclinas
  • 90. TULAREMIA • Cual es el agente causal de Tularemia? • R = FRANCISELLA TULLARENSIS POR PICADURA DE GARRAPATA • Cual es el cuadro clínico de Tularemia? • R = De inicio súbito con FIEBRE, ASTENIA, ADINAMIA, hiporexia, ataque al estado general, posteriormente hay diversas formas clínicas: ULCEROGANGLIONAR, manifestada con adenomegalias y ulceras cutáneas; TIFOIDE GLANDULAR, donde se afecta el pulmón, ganglios, hígado y bazo. SE PRESENTA CON NEUMONÍA, NECROSIS VISCERAL Y SEPSIS. • Como diagnosticas Tularemia? • R = En medio de CULTIVO DE AGAR SANGRE cisteína o ELISA. • Como tratas la Tularemia? • R = ESTREPTOMICINA, tetraciclina.
  • 91.
  • 93.
  • 95.
  • 96. LEISHMANIASIS • Que vectores transmiten leishmaniasis? • R = MOSQUITOS del genero LUTZOMYA Y PHLEBOTOMUS, LEISHMANIA pertenece a la familia del tripanosoma • Cual es el ciclo biológico de la leishmaniasis? • R = Al picar el mosquito a un mamífero infectado, ingiere los macrófagos parasitados, que posteriormente se destruyen, dejando libre al parasito en su forma flagelar e infectante (promastigote), se reproducen por fision binaria longitudinal, y migran hacia la porción anterior del aparato digestivo, hasta la probocide (boca). La infección se produce al alimentarse de un nuevo huésped, inocula los PROMASTIGOTES, que al ser fagocitados se transforman en AMASTIGOTES, donde se multiplican, causan la lisis de las células que los contienen e infectan a otras células. • En la leishmaniasis viceral o Kala-Azar a que células afecta el parasito? • R = A los MACRÓFAGOS del sistema retículo endotelial • Como se manifiesta la leishmaniosis cutánea? • R = En el sitio de inoculación, PAPULA ERITEMATOSA/PRURIGINOSA – NODULO – ULCERA PIEL/TEJ. CEL. SUB – EXUDA LIQ. SEROHEMATICO – ADENOPATÍA REGIONAL. • Como se manifiesta la leishmaniosis mucocutanea o espundia? • R = ULCERA CUTÁNEA – pasan years – LESIÓN OBSTRUCTIVA ÚNICA/MULTIPLE – LARINGE/TABIQUE NASAL/ANO Y VULVA.
  • 97. LEISHMANIASIS • Como se manifiesta la leishmaniasis viceral o Kala-Azar (fiebre negra)? • R = Hay HIPERPIGMENTACIÓN CUTÁNEA DE LAS EXTREMIDADES, mal estar general, DOLOR ABDOMINAL, HEPATO- ESPLENOMEGALIA, LINFADENOPATIAS, DIARREA es fatal si no se trata adecuadamente. • Como diagnosticas leishmaniasis? • R = VER EL AMASTIGOTE EN BIOPSIAS DE PIEL, lesiones mucosas, hígado, ganglios linfáticos, medula ósea tenidas con Giemsa. En los CULTIVOS de los tejidos se busca el PROMASTIGOTE. • Con que prueba de laboratorio diagnosticas leishmaniasis? • R = INTRADERMORREACCIÓN DE MONTENEGRO y ELISA. • Cual es el tratamiento de la leishmaniasis viceral? • R = Para la FORMA VISCERAL el tratamiento DE ELECCIÓN ES ESTIBOGLUCONATO DE SODIO O ANTIMONIATO DE GLUTAMINA IM POR 50 DÍAS. • Que medicamentos empleas en caso de que la leishmaniasis visceral no responda a los antiamoniales? • R = ANFOTERICINA LIPOSOMAL POR 5 DÍAS Y EL ISETHIONATO DE PENTAMIDINA. • Que medicamento utilizas en leishmaniasis cutánea? • R = ISETIONATO DE PENTAMIDINA, paromomicina APLICACIÓN TÓPICA 2 VECES AL DÍA POR 15 DÍAS.
  • 98.
  • 99. GIARDIASIS • Cual es el agente causal de giardiasis y a que gpo de edad afecta comúnmente? • R = GIARDIA INTESTINALIS de la clase MASTIGOPHORA, de la FAMILIA HEXANITIDAE y afecta a menores de 5 años • Cual es el ciclo biológica de la giardiasis? • R = Los TROFOZOITOS presentes en el duodeno, avanzan por el intestino y debido a la escasa cantidad de agua en el colon, cambian de forma a su estado quístico. El quiste permanece como la forma infectante y es ELIMINADO CON LA MATERIA FECAL. Al ser INGERIDO POR EL HUÉSPED, pasa por el tubo digestivo y los ácidos gástricos rompen la pared liberando al trofozoito, que nuevamente pasa al duodeno, donde madura, se fija a la pared intestinal y se replica por fision binaria longitudinal y es arrastrado con la materia fecal. • Que déficit nutricional puede ocasionar la giardiasis? • R = Déficit de vitaminas KADE, B12 Y ACIDO FÓLICO. • Como se manifiesta la giardiasis? • R = En niños cursa asintomática. NAUSEA , VOMITO, DIARREA CON MOCO Y ESTEATORREA.
  • 100. GIARDIASIS • Como diagnosticas giardiasis? • R = Observación de QUISTE EN HECES, ELISA. • Que componente de los trofozoitos de giardiasis te causa hipotrofia de las vellosidades intestinales • R = El DISCO SUCTOR • Cual es el tratamiento de elección para la giardiasis? • R = El TINIDAZOL con dosis única de 2 gr. METRONIDAZOL. La furazolidona causa hemolisis en pacientes con déficit de 6- GDP.
  • 101. AMEBOSIS • Cual es el agente causal de la Amebosis? • R = Entamoeba dispar y ENTAMOEBA HYSTOLITICA. • Como se desplaza el quiste de la E. Hystolitica? • R = Pseudopodos • Cual es el ciclo biológico de la entamoeba hystolitica? • R = Ingestión de quistes maduros, desenquistamiento en intestino delgado y los TROFOZOITOS liberados tetranucleados multiplican sus núcleos para formar una ameba de ocho núcleos, que posteriormente se fragmenta en ocho amebas, llamadas amébulas meta quísticas, que maduran a trofozoitos y migran a colon, donde se alimentan de bacterias y restos celulares; se reproducen por fisión binaria y producen quistes que se eliminan por las heces. • Como se unen los parásitos del intestino grueso de E. Hystolitica? • R = Mediante la lectina
  • 102. AMEBOSIS • Cual es el cuadro clínico abdominal de la amebiasis por E. Hystolitica? 1. COLITIS AMEBIANA AGUDA manifestada por dolor abd tipo cólico, tenesmo y EVACUACIONES DIARREICAS CON MOCO Y SANGRE. 2. COLITIS FULMINANTE con mortalidad mayor al 50% se presenta con diarrea sanguinolenta intensa, ataque al estado general, fiebre, dolor abdominal intenso y PERFORACIÓN INTESTINAL con choque séptico secundarios a la extensión de la ulcera hasta la serosa. 3. AMEBOMA O GRANULOMA AMEBIANO que cursa con DIARREA SANGUINOLENTA Y OBSTRUCCIÓN INTESTINAL. • Cual es el cuadro clínico de la amebosis cutánea? 1. ULCERAS FUGEDENICAS (de COLOR PURPURA, de crecimiento rápido, MUY DOLOROSAS y ABUNDANTE MATERIAL NECRÓTICO), 2. LESIONES VEGETANTES (friables, de evolución subaguda, LOCALIZADAS EN LOS PLIEGUES CUTÁNEOS), 3. ENTAMOEBOSIS (se presenta por una reacción de hipersensibilidad, CON LESIONES SEMEJANTES A URTICARIA. • Como se manifiesta el absceso hepático amebiano? • R = De inicio agudo, CON HEPATOMEGALIA Y DOLOR EN HIPOCONDRIO DERECHO QUE SE IRRADIA HACIA EL HOMBRO.
  • 103. AMEBOSIS • Como diagnosticas amibiasis? • R = COPROPARISOTOSCOPICO EN FRESCO, hematoxilina férrica o TINCIÓN DE GOMORI, en busca de TROFOZOITOS, ELISA. En la piel se busca trofozoito. • Cual es el tratamiento de amibiasis? 1) METRONIDAZOL, TINIDAZOL. 2) Dehidrohemetina y emetina son amebicidas de la pared intestinal, pero no sobre la luz intestinal siendo estos YODOQUINOL y paramomicina. • Que medicamento es eficaz para la amebiasis hepática? • R = CLOROQUINA
  • 104.
  • 105. TRIPANOSOMIASIS AFRICANA O ENFERMEDAD DEL SUENO • Cual es el agente etiológico de la TRIPANOSOMIASIS AFRICANA O ENFERMEDAD DEL SUEÑO y cual es su vector? • R = Tripanosoma BRUCEI, Tripanosoma RHODESIENSE y Tripanosoma GAMBIENSE. Su vector es la MOSCA TSE TSE. • Cual es el agente causal de la tripanosomiasis americana o ENFERMEDAD DE CHAGAS y cual es su vector? • R = TRIPANOZOMA CRUZI y su vector es la chinche • Cual es el ciclo biológico de los tripanosomas? • R = Al alimentarse de un individuo con la infección, los vectores ingieren el amastigote, que se multiplica dentro del tracto digestivo y las FORMAS INFECCIOSAS (TRIPOMASTIGOTE) son eliminadas en las heces; la infección se produce al depositar las heces en una herida de la piel antes de picar en las membranas mucosas o en la conjuntiva. Posteriormente, se multiplican en el sitio de la entrada, y entran a la sangre como tripomastigotes circulantes e invaden a las células, CON PREDILECCIÓN POR EL MIOCARDIO, MUSCULO LISO Y SNC.
  • 106. TRIPANOSOMIASIS O ENFERMEDAD DE CHAGAS • Como se manifiesta la tripanosomiasis o ENFERMEDAD DE CHAGAS POR T. CRUZY? • R = En la piel, en el sitio de entrada se observa CHAGOMA que es PARECIDO A LA FURUNCULOSIS CON LINFADENOPATIA REGIONAL. En conjuntiva, edema bipalpebral unilateral, CONJUNTIVITIS Y LINFADENITIS PRE AURICULAR (SIGNO DE ROMANA) CON FIEBRE Y ESPLENOMEGALIA LEVE. CRONICAMENTE se manifiesta por INSUFICIENCIA CARDIACA PREDOMINANTE DEL LADO DERECHO, con arritmias ventriculares y episodios de tromboembolia sistémica o pulmonar. MEGA ESÓFAGO Y MEGACOLON con síntomas de disfagia, constipación intensa, hipertrofia parotidea. • Como se diagnostica la tripanosomiasis americana o ENFERMEDAD DE CHAGAS? • R = En la FASE AGUDA se observa con la TINCIÓN DE GIEMSA de sangre anti coagulada. Diagnostico serológico se basa en hemaglutinación indirecta o ELISA. • Como se diagnostica la TRIPANOSOMIASIS AFRICANA? • R = Para T. GAMBIENSE por medio de PRUEBAS DE AGLUTINACIÓN y para T. RHODESIENSE es mediante la observación del parasito en la tinción de GIEMSA. • Cual es el tratamiento de elección para la TRIPANOSOMIASIS AMERICANA O CHAGAS? • R = NIFURTIMOX por 30-90 DÍAS o BENZIDIMAZOL de 30-90 DIAS. • Cual es el tratamiento de la TRIPANOSOMIASIS AFRICANA? • R = Para T. GAMBIENSE ES PENTAMIDINA POR 10 DÍAS. Para T. RHODESIENSE ES LA SURAMINA POR 7 DÍAS.
  • 107. BABEIOSIS • Cual es el agente causal de la babeiosis y cual es su vector? • R = Enfermedad INTRAERITROCITARIA, causada por PROTOZOARIO BABEZIA MICROTI Y DIVERGENS, transmitida por el vector que es una GARRAPATA de nombre IXODES SCAPULARIS • Cual es el ciclo biológico de la babesiosis? • R = Cuando las GARRAPATAS I. SCAPULARIS se alimentan de un animal infectado, ingieren los parásitos en el intestino, se produce la esquizogonia y posteriormente los merozoitos que infectan las células epiteliales del vector, y son transmitidas a glándulas salivales por la vía linfática y alimentarse del nuevo huésped le transmiten la infección. • Como se observan los parásitos de la babeiosis en los eritrocitos? • R = INCLUSIONES EN FORMA DE CRUZ lo que causa ANEMIA HEMOLÍTICA • Cual es el cuadro clínico de la babeiosis? • R = Clínicamente se manifiesta por FIEBRE, ANEMIA HEMOLÍTICA, cefalea, diaforesis, MIALGIAS, artralgias, fatiga, ictericia, ESPLENOMEGALIA. • Como diagnosticas la babeiosis? • R = Identificación del PARASITO INTRAERITROCITARIO EN FROTIS CON TINCIÓN DE GIEMSA. ANTICUERPOS ENTRE 1ERA Y 4TA SEMANA. En bh se observa ANEMIA, TROMBOCITOPENIA y LEUCOCITOSIS leve. • Cual es el tratamiento de la babeiosis? • R = Puede resolverse sin tratamiento. Para LA BABEIOSIS SINTOMÁTICA ES CON QUINIDINA MAS CLINDAMICINA. • Que tipo de babeiosis tiene peor pronostico? • R = B. DIVERGENS con 40% de muertes.
  • 108. TENIASIS • Cual es el agente causal de la teniasis? • R = T. saginata y T. solium • Cuales son las partes estructurales que componen a la tenia? • R = En su extremo anterior tienen una estructura llamada ESCÓLEX (CABEZA), seguida de un cuello. En su escólex presentan cuatro ventosas orales. En sus ventosas tiene un róstelo, con una corona doble de ganchos (escolices). El CUERPO (ESTRÓBILO) se conforma por varios segmentos llamados PROGLOTIDOS, que en su interior tienen ÓRGANOS SEXUALES. • Cuanto miden los adultos de T. SAGINATA? • R = 2-7 METROS de largo • Cuanto puede llegar a medir la T. SOLIUM? • R = 15-18 METROS • Cual es el ciclo biológico de la teniasis? • R = La teniasis intestinal es causada por la INGESTA DE CISTICERCOS. Al legar a intestino delgado, se unen a la pared intestinal. De los 2 a los 4 meses de la infección, alcanzan su madurez, caracterizada por la presencia de proglotidos gravidos que liberan huevos y se excretan por las heces. La cisticercosis ocurre por la ingestión de los alimentos o agua contaminados con excremento humano que contiene huevos o proglotidos. Los huevos se abren en el intestino, liberando a las oncoesferas, que invaden la pared intestinal pasando al torrente sanguíneo y linfático para migrar hacia musculo estriado, cerebro e hígado.
  • 109. TENIASIS • Cual es el cuadro clínico de la teniasis? • R = Normalmente es asintomática. Cuando cursa con síntomas son: PLENITUD ABDOMINAL, FATIGA, DOLOR LEVE OCASIONAL E HIPOREXIA. • Que déficit vitamínico cursa con teniasis? • R = Déficit de VITAMINA B12 • Cual es el cuadro clínico de CISTICERCOSIS en el SNC? • R = CRISIS CONVULSIVAS, cefaleas, vomito, HIPERTENSIÓN INTRACRANEAL. • Como diagnosticas la teniasis? • R = COPROPARASITOSCOPICO SE OBSERVAN HUEVOS, ELISA. • Cual es el tratamiento para teniasis? • R = PRAZICUANTEL a DOSIS ÚNICA DE 10 MG/KG. Ninguna droga mata a los huevos de tenia solium. NICLOSAMIDA para TENIA SAGINATA
  • 110. ASCARIASIS • Cuanto mide el macho y la hembra de áscaris? • R = La HEMBRA es mas grande mide 25-30 CM y el MACHO 25 CM. • Cual es el ciclo biológico de la ascariasis? • R = La infección ocurre por la ingestión de huevos maduros. Los huevos al ser eliminados por las heces, no tienen envoltura y son infecciosos a las 2-3 semanas en el suelo o el agua. Al ser ingeridos eclosionan en el intestino, las larvas inmaduras penetran la pared intestinal y SE DIRIGEN HACIA LOS ALVEOLOS A TRAVÉS DEL TORRENTE SANGUÍNEO, posteriormente, ASCIENDEN POR LOS BRONQUIS HASTA LA FARINGE, DONDE SON DEGLUTIDOS COMPLETANDO SU DESARROLLO EN EL INTESTINO DELGADO. • Cual es el cuadro clínico de la ascariasis? • Pulmonar: SÍNDROME DE LOFFLER (NEUMONÍA EOSINOFILICA) POR MIGRACIÓN ALVEOLAR, CON TOS PRODUCTIVA, DISNEA, sibilancias, dolor retroesternal, FIEBRE. • Intestino, cursa asintomático pero puede ocasionar APENDICITIS, OBSTRUCCIÓN DE CONDUCTOS BILIARES.
  • 111. ASCARIASIS • Como diagnosticas ascariasis? • R = Hallazgo de HUEVOS POR COPROPARASITOSCOPIO POR CONCENTRACIÓN DE FLOTACIÓN (FAUST) O POR SEDIMENTACIÓN (RITCHIE) QUE ES CUALITATIVO. Método cuantitativo de Stoll y Kato-Katz/ Kato-Miura para determinar numero de parásitos. • Cual es el tratamiento de la ascariasis? • R = ALBENZAZOL Y EL PAMOTATO DE PIRANTEL son los medicamentos de elección. • En que momento del embarazo puedes tratar ascariasis? • R = En el 3ER TRIMESTRE • Como actúa el pamotato de pirantel? • R = Inhibe la acetilcolinesterasa y bloquea la transmisión de la placa neuromuscular. • Cuales son algunas de las complicaciones extraintestinales de la ascariasis? • R = Pancreatitis, colangitis, colecistitis, absceso y perforación hepática, obstrucción de vías respiratorias y rotura del conducto onfalomesenterico.
  • 112. OXIUROSIS • Cual es el agente causal de la Oxiurosis? • R = Nematodo ENTEROBIUS VERMICULARIS, tiene la forma de alfiler. • Cual es el ciclo biológico de la Oxiurosis? • R = Los parásitos adultos SE ENCUENTRAN EN EL CIEGO, donde copulan y una vez que han terminado LA HEMBRA MIGRA HACIA EL ANO, depositando DURANTE LAS NOCHES SUS HUEVOS EN LA REGIÓN PERIANAL, secretando una sustancia que PRODUCE PRURITO INTENSO, lo que ocasiona el rascado y la contaminación de las manos. • Cual es el cuadro clínico de la oxiurosis? • R = PRURITO PERIANAL O VULVAR NOCTURNO. • Como diagnosticas oxiurosis? • R = CON CINTA ADHESIVA PERIANAL observando los parásitos CON TINCIÓN GRAM • Cual es el tratamiento de la oxiurosis? • R = Se debe tratar a toda la familia con ALBENDAZOL 400 MG/DOSIS ÚNICA
  • 113. ESTRONGILOIDOSIS • Cual es el agente causal de la estrongiloidosis? • R = Helminto STRONGYLOIDES STERCORALIS • Cual es el ciclo biológico de la estrongiloidosis? • R = Los adultos copulan en el yeyuno del huésped, y las hembras fecundadas entran a la pared intestinal y permanecen en la submucosa, los huevos larvados se desprenden de la submucosa y en la luz intestinal liberan a la larva rabditoide, pudiendo transformarse en larvas filariformes las cuales pueden penetrar el intestino pasando a la circulación donde digieren a diferentes tejidos. • De donde se obtiene la estrongiloidosis? • R = De CAMINAR DESCALZO, atraviesan la piel causando infección • Cual es el cuadro clínico de la estrongiloidosis? 1) En el sitio DONDE PENETRA LA LARVA filariforme, se PRESENTAN LESIONES PAPULOSAS, ERITEMATOSAS Y PRURIGINOSAS CON EDEMA LOCAL E INFLAMACIÓN. 2) Si es en LA REGION PERIANAL SE PRESENTA COMO DERMATITIS PERIANAL RADIADA. 3) Intestinales: se manifiestan por ulceras, DIARREA, DOLOR EPIGÁSTRICO URENTE, meteorismo, melena. 4) Pulmonar: con SÍNDROME DE LOFFLER, con tos, irritación traqueal, fiebre y hemoptisis. • Como diagnosticas la estrongiloisosis? • R = El COPROPARASITOSCOPICO por concentración- flotación (Faust) se OBSERVAN LAS LARVAS. • Cual es el tratamiento de la estrongiloidosis? • R = IVERMECTINA o ALBENDAZOL
  • 114. TRIQUINOSIS • Cual es el agente causal de la triquinosis? • R = TRICHINELLA SPIRALIS la cual se obtiene al consumir CARNE DE CERDO • Cual es el ciclo biológico de la triquinosis? • R = Consumiendo carne de cerdo, los jugos gástricos liberan las larvas enquistadas, maduran en el intestino delgado donde se reproducen. Posteriormente atraviesan la mucosa intestinal y se diseminan por la vía linfática/sanguínea hacia otros tejidos, especialmente musculo estriado pobre en glucógeno y de mayor actividad (diafragma, intercostales, bíceps, cuádriceps, lengua y maseteros) provocando una reacción inflamatoria. • Cual es el cuadro clínico de la triquinosis? • R = Se manifiesta en 3 fases. 1) Fase intestinal: con dolor mesogastrico tipo cólico, mal estar general, nausea, vomito, cefalea y diaforesis. 2) Fase de migración: edema, fiebre, diaforesis, fotofobia, conjuntivitis, 3) Fase muscular: DOLOR MUSCULAR QUE EXACERBA CON EL EJERCICIO. 4) A la ex fis se observan hemorragias en forma de astilla en unas retina y sub conjuntivales. • Como se realiza el diagnostico la triquinosis? • R = Coproparasitoscopio, biopsia de músculos dolorosos tenidos con hematoxilina y eosina con elevación de la CPK. • Cual es el tratamiento de la triquinosis? 1) Fase intestinal se da ALBENDAZOL 400 mg/12 x 15 días. 2) Fase muscular es intrahospitalario con antiparasitarios mas AINES.
  • 115. FASCIOLOSIS • Cual es el agente causal de la fasciolosis? • R = FASCIOLA HEPÁTICA • Cual es el huésped intermediario de la fasciola hepática? • R = Los CARACOLES • Cual es el ciclo biológico de la fasciola hepática? • R = Al ingerir las metacercarias, llegan al intestino delgado eclosionando, dejan salir las alrcas que atraviesan la pared intestinal y se dirigen a hígado, hasta llegar a los conductos biliares, donde se establecen para desarrollarse. Al fecundar en el hígado los huevos pasan al duodeno y se expulsan con la materia fecal, posteriormente el opérculo se abre para dejar salir al miracidio, el cual es una larva ciliada, que le permite desplazarse y penetrar a los caracoles pulmonados donde se desarrolla el esporoquiste. • Cual es el cuadro clínico de la fasciolosis? • R = DOLOR INTESTINAL, FIEBRE ALTA, HEPATO-ESPLENOMEGALIA, ICTERICIA con presencia de microabscesos y necrosis. • Como diagnosticas la fasciolosis? • R = EOSINOFILIAELEVADA, COPROPARASITOSCOPIO SERIADO • Cual es el tratamiento de elección para la Fasciolosis? • R = El BIOTINOL, se puede usar DEHIDROHEMETINA, METRONIDAZOL Y PRAZICUANTEL.
  • 116. ELEFANTIASIS O FILARIOSIS LINFÁTICA • Cuales son los agentes etiológicos de la elefantiasis o filariosis linfática? • R = WUCHERERIA BANCROFTI y B. malayi • Cual es el agente causal de la Filariosis? • R = Microfilaria • Cual es el vector de la filariosis? • R = MOSQUITOS O MOSCAS. • Cual es el ciclo biológico de la filariasis? • R = Las microfilarias son ingeridas por insectos hematófagos y transmitidas a los huéspedes donde se ALOJAN EN SANGRE Y TEJIDO LINFÁTICO • Cual es el cuadro clínico de la filariasis? • R = FILARIASIS LINFÁTICA: FIEBRE CON/SIN INFLAMACIÓN DE GANGLIOS Y VASOS LINFÁTICOS, PUEDE HABER EPIDIDIMITIS U ORQUITIS. En la infección crónica causa OBSTRUCCIÓN LINFATICA CAUSANDO EDEMA, HIDROCELE, LINFEDEMA ESCROTAL, VARICES LINFÁTICAS Y ELEFANTIASIS.
  • 117. ELEFANTIASIS O FILARIOSIS LINFÁTICA • Que te provoca la QUILURIA de la filariasis? • R = La ROTURA DE LOS VASOS LINFÁTICOS EN EL TRACTO URINARIO • Como se manifiesta la oncocercosis transmitida por la inoculación de larvas en la picadura de mosca en filariasis? • R = El parasito se aloja en la piel formando nódulos con edema y eritema local. • Cuales son los agentes causales en la oncocercosis o ceguera de los ríos en filariasis? • R = ONCHOCERCA VOLVULUS • Como diagnosticas la filariasis? • R = Se observan las MICROFILARIAS EN SANGRE Y LINFA. Se toma biopsia de piel. • Como tratas la filariosis? • R = DIETILCARBAMACEPINA
  • 118. ESQUISTOSOMIASIS • Cual es el ciclo biológico de la esquistosomiasis? • R = Los huevos son eliminados por las heces, al tener contacto con el agua se liberan las larvas sin la espina (miracidios), que nadan hasta encontrar un caracolinvadiendo los músculos donde se transforman en esporocitos, estas salen del caracol en forma de cercarías mismas que penetran en la piel del hombre cuando nada y se alimentan de eritrocitos. • Cual es el cuadro clínico de la esquistosomiasis? 1) DERMATITIS CERCARÍA: DONDE HAY PAPULAS ERITEMATOSAS Y PRURIGINOSAS EN EL SITIO DE LA INOCULACIÓN. En algunos casos se presentan manifestaciones alérgicas caracterizadas por fiebre, urticaria y eosinofilia. 2) FIEBRE DE KATAYAMA: FIEBRE, MIALGIAS, URTICARIA, tos no productiva, dolor abd, diarrea, eosinofilia y HEPATOESPLENOMEGALIA. 3) CRONICIDAD POR SCHISOTOMA MANSORI O JAPONICUM: DIARREA SANGUINOLENTA y hepatoesplenomegalia. 4) CRONICIDAD POR SCHISOTOMA HEMATOBIUM HAY DISURIA, HEMATURIA Y PROTEINURIA, POSTERIOREMENTE HAY PÓLIPOS EN VEJIGA, GLOMERULONEFRITIS, CISTITIS Y URETRITIS. • Como diagnosticas esquistosomiasis? • R = COPROPARASITOSCOPICO identificando los huevos, o en la ORINA, se puede realizar biopsia o tinción de ELISA. • Cual es el tratamiento de elección de la esquistosomiasis? • R = PRAZICUANTEL 20 MG/KG C/12 HRS
  • 119. TRICOCEFALOSIS/ TRICHURIS TRICHIRIA • Cual es el agente causal de la tricocefalosis y a quienes afecta principalmente? • R = TRICHURIS TRICHURIA y afecta principalmente a niños. • Cual es el ciclo biológico de la tricocefalosis? • R = Ingestión de alimentos con huevos larvados. Los huevos eclosionan en el intestino delgado y migran hacia el ciego, se introduce en la mucosa y deposita sus huevos en el lumen siendo excretados con las heces. • Cual es el cuadro clínico de la trichuris trichiria? • R= DOLOR ABDOMINAL TIPO CÓLICO, pujo, tenesmo, diarrea, distensión abdominal y PROLAPSO RECTAL. • Cual es el cuadro clínico de trichuris en niños? • R = Síndrome disentérico, ANEMIA FERROPENICA, palidez, PROLAPSO RECTAL Y RETARDO EN EL CRECIMIENTO. • Como se diagnostica la trichuriasis o tricocefalosis? • R = COPROPARASITOSCOPIO por concentración-flotacion (Faust). También se pueden utilizar los CRISTALES DE CHARCOT LEIDEN que son los productos de degradación de los eosinofilos. • Cual es el tratamiento de elección de tricocefalosis? • R = ALBENDAZOL Y HIERRO
  • 120. TOXOCARIASIS • Cual es el agente causal de la toxocara? • R = TOXOCARA CANIS, TOXOCARA CATI. • Cuanto mide el parasito de toxacara? • R = Menor al de Ascaris lumbricoides 5-10 CM • Cual es el ciclo biológico de la toxocarosis? • R = Huevos eliminados en la materia fecal, son ingeridos por el hombre, intestino delgado, penetran a pared intestinal y migran por torrente sanguíneo a hígado, pulmones, SNC, corazón y los ojos provocando la FORMACIÓN DE GRANULOMAS EOSINOFILOS. • Cual es el cuadro clínico de la toxocariasis? • R = El órgano MAS AFECTADO ES EL HÍGADO, donde se producen GRANULOMAS y los PULMONES MANIFESTADA COMO SÍNDROME DE LOFFLER, cursando con tos, fatiga, disnea, prurito, dolor abdominal, hepatomegalia, esosinofilia, hiperglobulinemia (IgE). La toxocariasis ocular donde hay estrabismo y deterioro visual unilateral donde se observa un granuloma eosinofilico. • Como realizas el diagnostico de toxocariosis? • R = ELISA. Hipergamaglobulinemia IgE. En LESIONES HEPÁTICAS se observan lesiones HIPOECOICAS. • Cual es el tratamiento de elección la toxocariasis? • R = ALBENDAZOL 400 MG CA/12 X 21 DIAS.
  • 121. CASO CLINICO • Cual es el cuadro clínico en RN o lactantes de toxoplasmosis cerebral y que resultados en TAC se observan? • R = The infection has a predilection for the CNS and the eye, and produces encephalitis in utero. Symptoms can be evident in the first few days of life. Infants born with active disease may have fever, rash, seizures, and hepatosplenomegaly at birth. En la TAC se observan calcificaciones cerebrales • - A 6-month-old child presents with recurrent seizures and poor development. The evaluation reveals a baby with hydrocephalus, impaired movement of the extremities, hypotonia, and retinal abnormalities. A computerized tomography (CT) scan demonstrates large ventricles and calcified lesions. Which of the following is the most likely diagnosis? • (A) Tay-Sachs disease • (B) congenital hydrocephalus • (C) kernicterus • (D) toxoplasmosis • (E) congenital neurosyphilis
  • 122. HERPES VIRUS SIMPLE 2 Cual es la manifestación clínica de herpes virus simple 2? R = VESÍCULAS CON BASE ERITEMATOSA con hipertrofia ganglionar local -A 23-year-old woman develops vesicular lesions on an erythematous base on her vulvar area. She has tender lymphadenopathy and dysuria as well. Which of the following is the most likely causative organism? • (A) cytomegalovirus (CMV) • (B) gonococcus • (C) herpes simplex virus type 2 (HSV-2) • (D) Treponema pallidum • (E) varicella zoster
  • 123. -HSV-2 genital infections may be associated with fever, malaise, and anorexia. Vesicular lesions usually ulcerate rapidly and become covered with exudate. There is a 90% chance of recurrent symptoms in the first year following a primary infection. HSV-1 genital infections are similar, but the chance of recurrence is less.
  • 124. OOFORITIS • Cual es la complicación mas común de parotiditis en las mujeres? • R = Ooforitis -An 18-year-old woman presents with headache, anorexia, chilly sensations, and discomfort on both sides of her jaw. She has also noticed discomfort in both lower abdominal quadrants. Physical examination reveals bilateral enlarged parotid glands that are doughy, elastic, and slightly tender; with a reddened orifice of Stensen’s duct. Her abdomen is soft with bilateral lower quadrant abdominal tenderness; a temperature of 38.5°C; and a pulse rate of 92/min. Laboratory data show hemoglobin 13 g/dL; hematocrit 40%; white blood cells (WBC) 9000/mL, with 35% segmented neutrophils, 7% monocytes, and 58% lymphocytes. Which of the following is the most likely cause for her abdominal pain and tenderness? • (A) mesenteric lymphadenitis • (B) oophoritis • (C) gonorrhea • (D) peritoneal metastases • (E) intestinal hyperperistalsis
  • 125. -Pain referring to either or both lower quadrants is common when oophoritis is present. Fever usually accompanies oophoritis. Sterility is not a consequence of mumps oophoritis.
  • 126. CASO CLINICO - A 23-year-old woman visits your office because of headache, malaise, anorexia, pain in both sides of her jaw, and discomfort in both lower abdominal quadrants. Physical examination reveals enlarged parotid glands; bilateral lower quadrant abdominal tenderness; a temperatura of 38.7°C; and a pulse rate of 92/min. Serologic testing (IgM) confirms the diagnosis of mumps. Which of the following is the most appropriate treatment for this condition? • (A) symptomatic • (B) immunization • (C) broad-spectrum antibiotics • (D) sulfonamides • (E) steroids
  • 127. - Antibiotics, sulfas, steroids, and mumps convalescent sera are of no value. Mouth care, analgesics, and a bland diet are usually recommended. Glucocorticoids are usually prescribed for orchitis, although definite evidence of their effectiveness is lacking. Prevention via vaccination is the preferred strategy for mumps.
  • 128. ETS
  • 129.
  • 130.
  • 131.
  • 132. ENFERMEDAD POR RASGUNO DE GATO • Cual es el agente causal de la enfermedad por rasguño de gato? • R = BARTONELLA HENSELEAE • -A 30-year-old man develops a pustular lesion at the site of a cat scratch on his forearm. This is followed 1 week later by malaise, fever, and lymphadenopathy. On examination, he has tender axillary lymph nodes. Which of the following is the most likely causative organism? • (A) Bartonella henselae • (B) Bartonella bacilliformis • (C) Bartonella quintana • (D) Coxiella burnetii • (E) Borrelia burgdorferi
  • 133. SARAMPION • Cuales son los signos y síntomas de Sarampión? • R = TOS, CORIZA Y CONJUNTIVITIS CON FOTOFOBIA. MANCHAS DE KOPLIC (Lesión blanquecina sobre base eritematosa en mucosa subyugal). Exantema que inicia cabeza, progresión céfalo-caudal RESPETA PALMAS Y PLANTAS.
  • 134. CASO CLINICO -A 7-year-old child, unvaccinated because of his parents’ religious beliefs, develops malaise, cough, coryza, and conjunctivitis with a high fever. Examination of his mouth reveals blue white spots on a red base beside his second molars. The next day he develops an erythematous, nonpruritic, maculopapular rash at his hairline and behind his ears, which spreads over his body. Which of the following is the most likely diagnosis? • (A) hand-foot-and-mouth disease (coxsackievirus) • (B) measles (rubeola) • (C) rubella (German measles) • (D) mumps • (E) pertussis
  • 135. - This is a typical case of measles. The Koplik’s spots in the mouth are easily missed with poor illumination. They are white blue spots of 1 mm on a red background and are not seen in any other infectious disease. The rash of measles becomes confluent while that of rubella does not. Pneumonia is an infrequent complication but accounts for many measles deaths. Giant cell pneumonia is also seen, most commonly in children suffering with a severe disease such as leukemia or immunodeficiency. Aerosolized ribavirin has been used to treat severe pneumonia secondary to measles, but its efficacy is still unclear. The other potentially lethal complication of measles is encephalitis
  • 136. CASO CLINICO - An 8-year-old boy from an impoverished innercity area has never been vaccinated appropriately. He develops fever, cough, and coryza. The next day, blue white spots develop on the buccal mucosa. On the third day, an erythematous, nonpruritic maculopapular rash develops on the face and spreads over the entire body. Which of the following is the most likely complication? • (A) pneumonia • (B) encephalitis • (C) otitis media • (D) bronchitis • (E) mastoiditis
  • 137. - The most common complication of measles is otitis media, other complications include mastoiditis, pneumonia, bronchitis, encephalitis, and lymphadenitis. Otitis media is usually a bacterial superinfection, and should be treated with antibiotics.
  • 138. CASO CLINICO - A6-year-old boy develops symptoms of cough, fever, and malaise followed by a generalized maculopapular rash that has spread from the head downwards. A clinical diagnosis of measles is made. A few days after the onset of the rash he is drowsy, lethargic, and complaining of headache. A lumbar puncture, electroencephalogram (EEG), and computerized tomography (CT) of the brain exclude other etiologies and confirm the diagnosis of encephalitis. Which of the following is the most likely delayed neurologic complication of measles virus encephalitis? • (A) meningitis • (B) pure motor paralysis • (C) autonomic neuropathy • (D) mental retardation or epilepsy • (E) “stocking-glove” peripheral neuropathy
  • 139. - Subacute sclerosing panencephalitis causes involuntary spasmodic movements and progressive mental deterioration, frequently ending in death within a year. It usually occurs in children whose measles occurred at an early age (= 2 years). It occurs 6–8 years after the primary infection. It presents with nonspecific symptoms such as poor school performance or mood and personality changes. It then progresses to intellectual decline, seizures, myoclonus, ataxia, and visual disturbances. Continued deterioration results in inevitable death.
  • 140. MONONUCLEOSIS • Cual es el cuadro clínico de un paciente con CMV/Mononucleosis? • R = Faringo- amigdalitis, ADENOPATÍAS CERVICALES- AXILARES ESPLENOMEGALIA. • - A 21-year-old woman visits her physician because of 3 weeks of a “flu-like” illness. She reports symptoms of malaise, fever, fatigue, and a sore throat. There is no weight loss or night sweats, and she has not traveled out of country. Her past medical history is not significant and she is not taking any medications. Physical examination is normal except for enlarged cervicallymph nodes. Laboratory data show hemoglobin 13.2 g/dL; hematocrit 42%; platelets 380,000/mL; WBC 8500/mL, with 35% segmented neutrophils, 1% eosinophils, and 64% lymphocytes, of which 36% were atypical. A heterophil antibody (sheep cell agglutination) test is negative. Which of the following is the most likely causative organism? • (A) herpes simplex • (B) echovirus • (C) CMV • (D) coxsackievirus • (E) reovirus
  • 141. -The most common cause of non-EBV mononucleosis-type syndrome is CMV. It is the most common presentation of CMV in nonneonates with normal immune function.
  • 142. MONONUCLEOSIS INFECCIOSA O ENFERMEDAD DEL BESO • Cual es el agente causal de la mononucleosis infecciosa o enfermedad del beso? • R = EPSTEIN BARR y es un herpes virus • A que patologías se asocia el virus del Epstein Bar? • R = LINFOMA DE BURKITT, linfoma de células T, LEUCOPLASIA ORAL VELLOSA, carcinoma nasofaríngeo. • Como dx al virus de Epstein bar? • R = AGLUTININAS HETEROFILICAS, MONO test. • Cual es el tx de Epstein Bar? • R = ACICLOVIR
  • 143. CASO CLINICO • -A 17-year-old man presents with new symptoms of fatigue, malaise, fever, and a sore throat. He has no significant past medical history and is not on any medications. Physical examination is entirely normal except for enlarged, palpable cervical, lymph nodes. He reports no weight loss or night sweats. Laboratory investigations include a normal chest x-ray, negative throat swab, but abnormal blood film with atypical lymphocytes. The hemoglobin is 15.5 g/dL; hematocrit 42%; platelets 290,000/mL; WBC 10500/mL, with 45% segmented neutrophils, 1% eosinophils, and 54% lymphocytes, of which 36% were atypical. Which of the following is the most appropriate initial diagnostic test? • (A) lymph node biopsy • (B) bone marrow • (C) erythrocyte sedimentation rate (ESR) • (D) heterophil antibody (sheep cell agglutination) test • (E) hepatic biopsy
  • 144. -The presence of IgG antibodies by the indirect immunofluorescence test indicates recent or prior EBV infection. IgM antibodies indicate recent infection only. Heterophil antibodies are present in 50% of children and 90–95% of adolescents and adults with infectious mononucleosis. Monospot tests are the best diagnostic tools but may not turn positive until the second or third week of the illness. Specific EBV antibodies and cultures are rarely used.
  • 145.
  • 146. HERPEANGINA • Que enfermedades causa el VIRUS COXSACKIE? • Grupo A : HERPEANGINA, ENFERMEDAD MANO-PIE-BOCA y conjuntivitis hemorrágica aguda, el • Grupo B:pleurodinia, miocarditis, pericarditis y meningoencefalitis. • Cuales son las características de la herpeangina? • R = LESIONES VESICULOSAS Y ULCEROSAS EN FARINGE, PARTICULARMENTE EN LOS PILARES ANTERIORES, las amígdalas y el paladar, vomito, anorexia, fatiga y ataque al estado general.
  • 147. CASO CLINICO - A previously healthy 19-year-old female university student develops myalgia, headache, fever, and malaise. Blood tests reveal lymphocytosis, with 20% of the lymphocytes being atypical. She remains tired and unwell for 6 weeks, but repeated tests for heterophil antibody are negative. Which of the following is the most likely diagnosis? • (A) Epstein-Barr virus (EBV) infection • (B) primary HIV infection • (C) human herpes virus type 7 (HHV-7) • (D) CMV infection • (E) toxoplasmosis
  • 148. VALVULAS NATIVAS • Cual es el agente causal de endocarditis en caso de reemplazo valvular con válvulas nativas? • R = STAPHYLOCOCCUS EPIDERMIDIS - 60-year-old man presents with fever and malaise 6 weeks after mitral valve replacement. On examination, his temperature is 38 C, blood pressure 130/80 mm Hg, pulse 80/min, and a loud pansystolic murmur at the apex, which radiates to the axilla. He has no skin or neurologic findings. Which of the following is the most likely causative organism? • (A) Staphylococcus aureus • (B) a fungus • (C) Staphylococcus saprophyticus • (D) pneumococcus • (E) Staphylococcus epidermidis
  • 149. - About half of all early-onset (<60 days after surgery) prosthetic endocarditis is caused by staphylococcal infection, with S. epidermidis predominating. Early-onset prosthetic endocarditis is generally the result of intraoperative contamination of the prosthesis or a bacteremic postoperative complication.
  • 150. CASO CLINICO - A 56-year-old man is having intermittent fevers and malaise for the past 2 weeks. He has no other localizing symptoms. Two months ago, he had valve replacement surgery for a bicuspid aortic valve. Amechanical valve was inserted and his postoperative course was uncomplicated. On examination, his temperature is 38 C, blood pressure 124/80 mm Hg, pulse 72/min, and head and neck are normal. There is a 3/6 systolic ejection murmur, the second heart sound is mechanical, and a 2/6 early diastolic murmur is heard. The lungs are clear and the skin examination is normal. Three sets of blood cultures are drawn and an urgent echocardiogram is ordered. Which of the following is the most likely causative organism? • (A) Staphylococcus aureus • (B) S. epidermidis • (C) S. viridans • (D) enterococci • (E) Candida
  • 151. - S. epidermidis is still the leading cause of prosthetic valve endocarditis in the early postoperative period (usually defined <2 months). It is usually the consequence of intraoperative contamination or postoperative bacteremia. Endocarditis occurring 12 months after surgery is usually due to the same organisms that cause native valve endocarditis.
  • 152. ENDOCARDITIS • Cuales son los agentes infecciosos involucrados en la endocarditis sub aguda? • R = Estreptococos del grupo viridans • Cual es el tratamiento para los estreptococos del grupo viridans? • R = PENICILINA + AMINOGLUCOSIDO POR 30 DÍAS • Cual es el agente causal de endocarditis de Litman Sax? • R = AUTOINMUNE, común en LUPUS. • Cual es el procedimiento terapéutico profiláctico para intervención cardiaca o dental? • R = AMOXICILINA 2 GR 1 HR ANTES DEL PROCEDIMIENTO, si es ALÉRGICO CLINDAMICINA 600 MG, AZITROMICINA 500 MG 1 HR ANTES DEL EVENTO. • En endocarditis meticilino resistente que medicamento das? • R = TEICOPLANINA, en gram (-) cefalosporinas de 3era
  • 153.
  • 154.
  • 155.
  • 156. NEUMONIA • Cual es el principal agente patógeno en neumonía adquirida en la comunidad? • R = Streptococcus pneumoniae • -A 73-year-old man from a nursing home develops headache, fever, cough, sore throat, malaise, and severe myalgia during a community outbreak affecting numerous other residents at the home. The symptoms gradually resolve after 3 days, and he starts feeling better but then there is a reappearance of his fever,with cough and yellow sputum production. On examination, his temperature is 38.5 C, pulse 100/min, respiration 24/min, oxygen saturation 88% and crackles in the right lower lung base, bronchial breath sounds and dullness on percussion. CXR reveals a new infiltrate in the right lower lobe. Which of the following is the most likely causative organism? • (A) primary viral pneumonia • (B) an autoimmune reaction • (C) Mycoplasma pneumoniae • (D) Streptococcus pneumoniae • (E) Neisseria catarrhalis
  • 157. - This man has a complication of viral influenza; a secondary bacterial pneumonia has developed. This usually occurs 2–3 days after the initial viral symptoms resolve. Primary viral pneumonia with influenza is not common. S. pneumoniae, Staphylococcus, and H. influenzae are the most common bacterial invaders in pulmonary complications of influenza. Pneumonia is the leading cause of death and may also be due to S. pneumonia and H. influenzae. Mixed viral and bacterial pneumonia is common; pure viral pneumonia in influenza is uncommon (but can be very severe).
  • 158. NEUMONIA • Cual es el patógeno mas frecuente en la comunidad? • 1. Streptococo pneumonie 2.Micoplasma pneumonie 3.Clamidia pneumoniae • Agente etiológico de neumonía hospitalaria? • 1.Pseudomona 2. Kliebsella 3.E.Coli • Agentes etiológicos de neumonía adquirida en la comunidad con bronco aspiración? • Px ALCOHÓLICOS y/ o PX QUE CONVULSIONAN y los agentes BACTEROIDES ORALIS, streptococo • Agentes etiológicos para neumonía en px atípica? • INFLUENZA H1N1 MICOPLASMA PNEUMONIE • Caso clínico: paciente que estuvo en contacto con aire acondicionados o torre de enfriamento o nebulazadores que durante un tiempo no funciono… desarrolla neumonía • cual seria el agente causal? • Legionella pneumonie • Tx: • ERITROMICINA o CLARITROMICINA si viene
  • 159. NEUMONIA • Enfermedad ocupacional que desarrollan personas que cuidan pájaros? • PSITACOSIS agente causal: CHLAMYDIA PSITTACI • Tx : • TETRACICLINA (doxiciclina) • Caso clínico de MICOPLASMA PNEUMONIE caract y tx? • Es una neumonía atípica FENOMENO DE REYNAUD, ERITEMA MULTIFORME. Hemolisis, MIRINGITIS BULOSA Rx: Pneumoniae en Parches • Tx: • ERITROMICINA (si vienen azitromicina o claritromicina contestar esas) • Forma en que se realiza el dx: • Prueba de fijación de complementos (la de mas especificidad y sensibilidad )ANTICUERPOS FRÍOS • En que consiste el sx de austria? • ENDOCARDITIS, NEUMONÍA y MENINGITIS…. Se presenta en px ALCOHÓLICOS • Agente causal: • STRPTOCOCO PNEUMONIE • Cual es la imagen radiográfica de la neumonía adquirida en la comunidad: Broncograma aéreos dentro de la consolidación pulmonar.
  • 160. NEUMONIA • Que neumonía se relaciona con neumatocele? • Neumonía estafilococica (burbujas de aire en el parénquima pulmonar)? • La neumonía adquirida en el hospital cual es su imagen radiológica? • Patrón necrotizante, de focos múltiples • Neumonía adquirida en comunidad por bronco aspiración con pérdida del conocimiento que partes está afectada? • Esta afectado el PULMÓN DERECHO, el LÓBULO MEDIO y el SEGMENTO POSTERIOR • Neumonia en un px con vih cual es su agente causal? • Neumonía por PNEUMOCYSTIS JIROVECI
  • 161. NEUMONIA • Px con VIH con sospecha de neumonía por pneumocystis jiroveci que tinción se realiza para corraborar dx? • R: metenamina argéntica por NITRATO DE PLATA • Px con micoplasma el mejor estudio es? • 1era pruebas de FIJACIÓN DE COMPLETOS • 2do opción ANTICUERPOS FRÍOS • Tratamiento de neumonía adquirida en la comunidad? 1) Si no tiene enfermedad concomitante: 2) MACROLIDO (azitromicina o claritromicina) tx de 1era elección 3) Con enfermedad concomitante: 4) FLUOROQUINOLONAS (levo, moxi, gatifloxacina) • Tx de px con neumonía adquirida en la comunidad pero que por sus caract. Es internado? • CEFALOSPORINA DE 3RA GENERACIÓN MAS MACROLIDO: ej cefotaxima + claritromicina • Tx de neumonía por neumococo? • FLUOROQUINOLONA LEVOFLOXACINO (NOponer PENICILINA… ojo)
  • 162.
  • 163. NEUMONIA • Px con neumonía adquirida en comunidad pero VIENE GRAVE que debe ser INTERNADO EN TERAPIA intensiva? • Fluoroquinolona + ceftriaxona o fluoroquinolona + cefotaxima • Tx paciente con neumonía ADQUIRIDA INTRAHOSPITALARIA, internado en terapia intensiva? • Pseudomona, ecoli, kliebsella FLUOROQUINOLONA + CEFALOSPORINA antipseudomonica (CEFTAZIDIMA, cefoperazona) o (carbepenem o iminpenem ) solo • Neumonía hospitalaria por staphilococos aureus cual es el tx? • VANCOMICINA (útil incluso para meticilino resistentes)
  • 164.
  • 165.
  • 166.
  • 167.
  • 168. CASO CLINICO - A 4-year-old boy is sent to the emergency room because of clinical suspicion of meningitis. He has been ill for 2 days with fever and lethargy. On examination, he is febrile, the neck is stiff, and papilledema is present. There is no rash, the lungs are clear, and heart sounds normal. Which of the following is the most likely causative organism? • (A) Neisseria meningitidis • (B) Streptococcus pneumoniae • (C) Haemophilus influenzae • (D) Staphylococcus • (E) Listeria species
  • 169. - Since the introduction of H. influenzae type B vaccine, S. pneumoniae has become the most common type of meningitis in infants and toddlers
  • 170. TUBERCULOSIS • Cuales son las manifestaciones extrapulmonares de la Tb? • R = Meningitis, pericarditis, invasión ósea, gastrointestinal y urinaria • Cual es la tinción especifica para Tb y reacción PPD positiva? • R = Tinción de ZIEHL NEELSEN. En paciente inmunocomprometido ES + SI ES > 5 MM • Como se muestra el LCR con Tb? • R = CON PROTEÍNAS Y CELULARIDAD AUMENTADAS E HIPOGLUCORRAQUIA. • Como se da el tratamiento para Tb? • FASE INTENSIVA: Se dan 3 FÁRMACOS MAS ETAMBUTOL Y ESTREPTOMICINA hasta completar 60 DOSIS • FASE DE SOSTEN: ISONIAZIDA Y PIRAZINAMIDA 3 VECES A LA SEMANA DANDO 45 DOSIS. • Como das quimioprofilaxis a contactos de pacientes con Tb? • R = 6M EN CONTACTOS <5ª Y EN LOS CONTACTOS DE 5 – 14ª NO VACUNADOS CON ISONIAZIDA SIN EXCEDER 300 MG. • Como se le llama a la disminucion de la respuesta tuberculinica con el paso de la edad? • R = Efecto Spink
  • 171. TUBERCULOSIS • Cual es el mecanismo de acción de la RIFAMPICINA? • R = Es un BACTERICIDA que inhibe la subunidad beta de la RNA POLIMERASA, IMPIDIENDO LA SÍNTESIS PROTEÍNICA. • Cual es el mecanismo de acción de la ISONIACIDA? • R = Es un ANÁLOGO DE LA NICOTINAMIDA que inhibe la SÍNTESIS DE ACIDO MICOLICO y el metabolismo de los lípidos que CAUSA LA ROTURA DE LA PARED, el cual DEBE ADMINISTRARSE CON PIRIDOXINA YA QUE PUEDE CAUSAR NEUROPATÍA PERIFÉRICA. Se usa la Estreptomicina en caso de resistencia a Isoniacida. • Cual es el mecanismo de acción de la PIRAZINAMIDA? • R = ANALOGO DE LA NICOTINAMIDA que actúa como BACTERICIDA en un medio acido, lo que le permite actuar contra los bacilos intracelulares. • Cual es el mecanismo de acción del ETAMBUTOL? • R = Tiene actividad BACTERIOSTÁTICA. En menores de 12ª puede causar NEURITIS RETROBULBAR.
  • 172. TUBERCULOSIS • Grupo de edad en que es mas frecuente la micobacteriosis ganglionar? • Es mas frecuente en los niños (en segundo lugar la tb pulmonar) se divide en escrofulacia y la tuberculosa • Grupo de edad en que es mas frecuente la micobacteriosis pulmonar? • En adultos: la primera causa es la tb pulmonar y la #2 la tb ganglionar • Causa mas frecuente de Síndrome de piura estéril? • 1era. Tuberculosis Renal • 2da. Micoplasma Ominis • Características de TB Renal? 1) Dolor lumbar, disuria, Febricula, 2) Urografía excretor: URETEROS ARROSARIADOS • Cuando afecta ala columna vertebral se llama? • Mal de Pott • Cuanto tiempo se debe mantener en aislamiento a alguien que padece tuberculosis activa? • R = 2-3 meses
  • 173. TUBERCULOSIS • Diagnostico diferencia de tb intestinal? • Enfermedad de Cronh y cáncer intestinal • Cual es la utilidad de la BCG? • Proteger contra MENINGITIS TUBERCULOSA en los niños (su protección no dura mas de 10 años) • Características de una meningitis tuberculosa? • Evolución subaguda, tiende a obstruir los agujeros de lushka y maggendi, CARACTERÍSTICAMENTE EL 3-4-6 PAR CRANEAL AFECTADO • Cuales son las dos micobacteriosis, en los cuales en el esquema de tratamiento se agrega corticoesteroides? • Meningoencefalitis tuberculosa y pericarditis tuberculosa • Causa mas frecuente de enfermedad de adison? • 1era.AUTOINMUNIDAD 2da:TUBERCULOSIS (mas frecuente en países subdesarrollados)
  • 174. TUBERCULOSIS • Cual es la causa mas frecuente de origen infeccioso de calcificación de suprarrenales? • Enfermedad de Adison (astenia, adinamia, HIPERPIGMENTACION CUTANEA elevada, NA BAJO, K ELEVADO, acidosis elevada, hipotensial arterial) • Diagnostico definitivo de TB? • Cultivo LOWENSTEIN-JENSTEIN-HOLTZ • Que es el complejo de Ranke? • Nódulos linfático hiliares o parahiliares afectados por tb + complejo primario de Gonh • Cual es el sitio mas frecunte del complejo primario de Gonh? • Lóbulo inferior del lóbulo derecho • Como se interpreta Intradermoreaccion de Monteaux? • Se lee a las 72 HRS DE HABERLO APLICADO.. SI TIENE MAS DE (10 MM) 1 CM DE INDURACIÓN SE CONSIDERA POSITIVO
  • 175. TUBERCULOSIS • Si es un px con VIH, a cuanto se considera positiva? • .5 cm (5MM) • Que medicamentos constituye los 4 meses restantes de tx de px con tb? • Isoniacida + rifampicina • Que antifimico queda contraindicado en la mujer embarazada? • ESTREPTOMICINA…. El tx es con ISO RIFA ETAMBUTOL + PIRIDOXINA POR 9 MESES (para disminuir el efecto indeseable de la isoniacida –polineuropatia y hepatitis-) • Cuales son las tuberculosis que se da tx por 9 meses? • En la tb MILIAR, tb SNC, tb ARTICULAR u OSEA • Los px con resistencia solo a isoniacida que tx se deberá dar? (Se llama RESISTENTE si lo es a isoniacida) 1) RIFAMPICINA, PIRASINAMIDA Y ETAMBUTOL O RIFAMPICINA, PIRASINAMIDA ESTREPTOMICINA POR 6 MESES 2) Rifampicina + etambutol por un año (se llama MULTIRESISTENTE si es a iso y rifampicina)